Uworld consolidated

Ace your homework & exams now with Quizwiz!

When is septic arthritis considered in a child? Aka Kocher criteria

1. non - weight bearing 2. fever 101.3 3. ESR >40 or crt > 2 4. Leukocytosis Pt with >3 have 93 % chance of septic arthritis --> immediate joint aspiration (helps confirm and decompress joint) is done on BOTH joints then given empiric parenteral abx with drainage and debridement

What are the indication of ICD placement in NON OBSTRUCTIVE HCM?

1. non sustained V-tach on holter, 2. LVH with septal thickening >3mm, 3. hypotension w exercise, 4. history of family syncopy due to arrhythmia, 5. FHX of sudden cardiac death (scd) syncope or 6. exertion dyspnia should be considered for ICD placement

What are the 3 CHD RISK EQUIVLENTS? and why?

1. non-coronary atherosclerotic disease 2. DM 3. CKD DM is a significant predictor of adverse CV outcomes especially in WOMEN!

What 3 conditions have multiple skin tags?

1. pregnancy 2. Cron disease (perianal) 3. insulin resistance

What is the ideal modality to screen for osteolytic bone lesions and which 3 cancers cause it?

1. prostate 2. small cell 3. hodgkin lymphoma radionuclide bone scan

Evaluation of hypercalcemia

1. repeat and correct for albumin (vs ionized calcium) 2. measure PTH 3. if low <20 indicative of PTH INDEPENDeNT process 4. measure 25 hydroxyvitamin D, 1,25 dihydroxyvitamine D, PTHrP

Wegener's granulomatosis P/w

1. saddle nose, rhinitis, 2. hemoptysis and other upper and lower respiratory symptoms. 3. c-ANCA + 4. CXR with cavitary nodules.

What traits do Marfan's and ED have in common? What are classic to ED and Marfan's?

1. scoliosis 2. pectus excavatum 3. autosomal dominant 4. Mitral prolapse 5. joint hypermobility ED+rose90day (col5a mutation) distinct features: 1. Skin: hyperextensibility, bruising, atrophic scar healing 2. high arched palate 3. abdominal and inguinal hernia's 4.Uterine prolapse Marfan's (fbn1 mutation) 1. normal skin except striae 2. pectus carinatum 3. TALL LONG EXT 4. Lens + retinal detachment 5. Spontaneous pneumothorax 6. Progressive aortic root dilatation

cutting behavior is a PSYCH. symptom in adolescents with distressing negative affective states. Initial MGMT of cutter w/ nonsuicidal self-injury should include WHAT ?

1. screening for SI 2. comprehensive psychiatric evaluation. NO HOSPITALIZATION DESPITE INC RISK OF SUICIDE IF CHICK SAID SHE DIDNT COMIT ITS COOL

Cutaneous manifestations of infectious endocarditis:

1. splinter hemorrhage 2. petechiae 3. janeway lesion 4. osler nodes

Causes of fall include?

1. syncope 2. mechanical 3. seizure Ask about number of episodes, prodrome, surrounding event, associated sympt., duration, time until recovery, witness, other medical problem, drugs, seizure history.

name the side effects of aminoglycosides?

1. teratogenic (deafness in newborn) 2. ATN 3. contra-indicated in MG due to post NMJ Blockad

name the clinical features of TSH secreting pituitary adenoma?

1. thyrotoxicosis 2. diffuse goiter 3. mass effect (optional)

Vasovagal syncopy is associated with what prodromal symptoms (4)?

1. tunnel vision 2. dia4 3. nausea 4. palor

What are the 6 clinical risk factors that would lead to a high risk for RCRI?

1. vascular high risk surgery (breast or cataract is low risk) 2. hx of ischemic heart dx. 3. HF 4. STROKE 5. DM on insulin 6. PREOP CRT >2 2RISK -- MODERATE ≥3 HIGH RISK 5%

What are the non-cardiogenic eitiology of syncope?

1. vasovagal 2. carotid sinus hypersensitivity 3. orthostasis 4. seizures 5. TIA

What is the most effective nonpharmacologic measure to decrease BP those with HTN?

1. weight loss: reduction of BMI <25 results in 20 SBP drop for every 10kg loss! 2. DASH diet: high in fruits veggins and low saturated/ total fats -> 10point drop SBP 3. Exercise 30min x 5 days = 9 point drop 4. diet sodium <3 g/day = 6drop sbp 5. ≤2 drinks/day 3 drop

The treatment of C difficile-associated diarrhea (CDAD) requires:

10 days of oral vancomycin or fidaxomicin is generally curative

stone removal size

10 mm gall stone and kidney 5-10mm

When should cleft lip repair be done in an infant?

10 weeks old, 10lbs, and 10grms of Hgb

What is the interval for f/u colonoscopy s/p polypectomy

10 years if small hyperplastic polyp 5 years if ≤2 small (<1 cm) adenomatous polyps 3 years if 3-10 adenomatous polyp or one with villous ft. 2-3 months if large >2 cm or adenocarcinoma: exclude synchronous lesion (tattooed) and reoccurrence with f/u @ 1, 4, and 9 years s/p initial resection.

What is the risk of getting down syndrome if the mother is a carrier of chromosome 21 robertsonian translocation?

10%

cd4 for toxo hiv - (HINT: 100 BJ)

100 BJ benjamin

what age baby cruises, walks alone, uses 2-finger pincer grasp, says mama/dada specific, imitates actions

12 month old

What should a newborn get if the mother has active Hep B with high viral titers and HepBeAG during pregnancy?

12hours after delivery should have HepB IG and HepB vaccine followed by routine immunization (series vaccination) with 3mo follow up to assess for HepB serology after 3rd vaccine is given

How long should you wait before starting a SSRI in someone on previous MOI?

14 days minimum washout UNLESS they were on a long acting ssri like fluoxetine then there's a 5 week minimum, otherwise there is risk of SS.

at what age does a kid: walk backward, uses a cup, uses 4 to 6 words, temper tantrums

15 months

Since serum ferritin less than 15ng/ml is considered absolute iron deficiency what is the range in which FE deficiency is still likely? What does anisocytosis represent?

15-30 is the cut off that indicates Fe deficiet (eg males with gi blood loss) Anisocytosis means unequal RBC volume; indication of anemia

How much smoking would one have to do to have a significnat increase in CHD risk?

1ppd!

What is the treatment for acute gout? What if they have a contraindication to the 1st line?

1st line for acute gout is to treat with NSAIDs specifically those with a short half life eg indomethacin, ibuprofen If unable to tolerate NSAID treat with colchicine?

pt w cluster headache should receive preventive treatment at the onset of a cluster ha _____ is considered the agent of choice for the prevention of episodic and chronic cluster headaches.

1st line prevention: Verapamil - prevent; 240 mg and titrated up as needed 1st line abortive treatment: 100% oxygen inhalation >15min: aborts HA in >70% of patients. 2. IF NOT ABORTED 2ND LINE W/ sumatriptan -

What is the first and 2nd line treatment for someone with dexa t score ≤-2.5?

1st line: bisphosphonate 2nd line: terapartide Parathyroid hormone fragment (PTH 1-34, teriparatide): improves bone density and reducing the risk of fractures. high cost and for pt a. high risk of fractures, b. failed bisphosphonate

UTI empiric treatment

1st line: trimethoprim-sulfamethoxazole (TMP-SMX) for 3 days cures >90% of cases; 2nd line: 5 days of nitrofurantoin is another common first-line agent. - cipro in pylo

Treatment of inflamatory nodular acne 1st line 2nd line 3rd line?

1st topical retinoids 2nd topical erythromycin or clindamycin 3rd oral tetracyclines: with systemic doxycyline (treat diffuse spread)

Class II is associated with

750-1,500 ml of blood loss, HR > 100, hypotension, and anxiousness.

ppi trial

8 weeks - per lady

chronic uticaria cases

80-90% idiopathic

what usually presents with mild elevations in amylase, transaminases, an aspartate aminotransferase to alanine aminotransferase ratio of > 2:1, and marked elevations in bilirubin. It can present with all the signs and symptoms of portal hypertension.

: Alcoholic hepatitis is an acute inflammation of the liver linked to chronic, heavy alcohol consumption. amylase can be elevated with persistant vomiting !

___is a rare condition characterized by acute glomerulonephritis and pulmonary alveolar hemorrhage and is associated with positive antibodies to the glomerular basement membrane.

: Goodpasture syndrome

The goal of management of hypertensive urgency is to lower the blood pressure to _____ .

<160/100 mmHg over several hour to days For previously treated patients, you can restart or adjust the dosing of the existing medication regimen. For untreated patients, you can start a calcium channel blocker, beta-blocker, or angiotensin converting enzyme (ACE) inhibitor. You should not start a diuretic alone.

Platelet transfusion is generally indicated in patients with acute GI bleeding with platelets <____ /mm2.

<50,000/mm2.

length time bias detects

= more benign cases rather than those that are rapidly progressive die off quickly and thus never benefit from screen

Laparoscopy is the treatment of choice for a suspected non-ruptured ectopic pregnancy greater than ___ cm.

> 3.5cm --> laproscopy rather than repeat bhcg but if pt is hemodynamically stable, closely f/u serial beta-hCG levels is recommended with or without ultrasound.

Procedural risk factors that can cause pulm compleations post operatively?

> 3hr long surgery, emergent sugery, head neck, aortic vascular, thoracic upper abd, neurosurger, or use of long acting NM blockade (pancuronium)

High-risk patients for colorectal cancer include those with first-degree relatives diagnosed with colorectal cancer or advanced adenomatous polyps (size ____, with ______ features, or high-grade dysplasia) before age 60.

>1 cm, VILLOUS OR HIGH-GRADE DYSPLASION Screening colonoscopy @ 40 or 10 years prior (whichever sooner).

Recurrent pregnancy loss is defined as

>3 consecutive spontaneous abortions (eg, <20 weeks gestation).

When do you taper oral glucocorticoids for an asthma exacerbation refractory to bronchodilator therapy?

>3 weeks daily use or with Cushingoid appearance. Glucocorticoid cessation is warranted after therapeutic benefit is achieved or if HTN, or psychosis.

What confirms volume of urinary retention for bladder scan?

>300ml if urinary retention MUST DECOMPRESS BLADDER before giving any other treatement for either UTI, prostatitis, BPH etc Kidney function will deteriorate if decompression SUPRAPUBIC CATH is not done ASAPPPPPPPPPP

Weight loss surgery is appropriate for patients with BMI of _____or with obesity-related comorbidity and BMI ___kg/m2.

>40 kg/m2 >35 kg/m2.

Tobacco Abuse - Perioperative Period, Benefits / Smoking Cessation For____ Improves Outcome

>6 Weeks

Rheumatoid arthritis p/w morning stiffness in the bilateral metacarpophalangeal and proximal interphalangeal joints. The onset is insidious, and symptoms must be present for ____ weeks to establish the diagnosis

>6 weeks of symptoms

In non-diabetic pt statins are indicated for those with what estimated 10-year ASCVD risk or what LDL level?

>7.5-10%, ≥190 mg/dL.

What injury should be suspected on somone who landed on a pivot and now has difficulty with passive knee extension and unable to extend knee against force?

ANTERIOR PATELLAR INJURY: likely due to jump and pivot landing as the quads insert onto the lower patella and thus can cause pain and swelling like ACL but have negative Lachman's and difficulty with extension. Surgical intervention is needed.

STATS: What is synonymous with attributable risk increase?

ARI

What are the most common and important extraarticular manifestations of ankylosing spondylitis?

AS -- 4 As and 1S 1. anterior uveitis 2. aortic regurg 3. apical pulm. fibrosis 4. IGA nephropathy 5. Restictive lung disease

Name the 3 Left to right shunts

ASD VSD PDA

When is acute stress disorder unable to be diagnosed and thus will turn into PTSD

ASD 3days-1month of symptoms and can include depersonalization/derealization symptoms with amnesic symptoms PTSD >1month

Diastolic flow "rumble" murmur with a loud 1st heart sound and CXR: Large pulmonary arteries; increased pulmonary markings ECG: RBBB and right axis deviation

ASD murmurs -TEE is diagnostic; bubble study injecting microbubbles and watching them cross the defect via a right-to-left shunt. This is often used to aid in the diagnosis of ASDs.

What 2 drugs must be given to someone who is planning on getting percutaneous intervention for ACS and why?

ASPRIN AND LOAD WITH PLAVIX ( GP2B3A receptor inhibitor to prevent platelet aggregation as cath can sheer the endothelium of coronary artery.

What LFT abnormality is seen in HELLP S/O?

AST elevation!

Transaminases usually not > 300, T- bilirubin > >5 mg/dL)

AST:ALT ratio 2:1, ETOH HEPATITIS

Toxic megacolon should be suspected in someone with IBD colitis who develops toxic signs and has a DISTENDED, tympanitic abdm. What imaging helps diagnosis?

Abdm. X RAY CT scan is best for early detection of COMPLICATIONS

How is hypertrophic pyloric stenosis diagnosed and what is the common metabolic derangement and treatment of disease?

Abdominal ultrasound 1st line and UGI contrast study Commonly have hypokalemic and hypochloremic metabolic alkalosis, that is why electrolyte correction is done prior to pylorotomy during treatment.

AAA diagnosis in asymptomatic vs symptomatic presentation (eg limb ischemia)

Abdominal ultrasound in asymptomatic patients who are suspected to have an AAA. Surgical repair of an abdominal aortic aneurysm is recommended when the diameter is greater than or equal to 5.5 cm. If confirmed or if showing symptoms for AAA then get abdominal CT WITH CONTRAST

What is someone with subchorionic hematoma at risk for?

Abortion PPROM preclampsia IU. Growth restriction abruptio placenta

HTN and cocaine use put pregnant mom at risk for what?

Abruptio placentae

Pt hospitalized for acute variceal bleed needs what prophy?

Abx prophy b/c bacterial infections develop in 50% - FQN (ofloxacin, norflox or cipro) for 10d

Keratitis is an inflammation of the cornea, the clear tissue covering the pupil and iris and bordered by the bulbar conjunctivae. Infectious keratitis can be due to a variety of organisms, including bacteria (eg, Pseudomonas aeruginosa), viruses (eg, herpes simplex virus, varicella-zoster virus), fungi, or what parasites (eg, ????).

Acanthamoeba

What is acute diarrhea and when is it caused by virus or bacteria?

Acute diarrhea <14 usually due to VIRAL pathogen causing bland. watery, diarrhea and self-limited course Bacterial infection --> acute diarrhea has visible blood or mucus (dysentery) causing abdominal pain and fever, malaise!

______which presents 2-4 weeks after bacterial infection, causes migratory polyarthritis involving the larger joints (eg, knee, elbow).

Acute rheumatic fever, group A Streptococcus pharyngitis Skin findings include erythema marginatum and subcutaneous nodules.

When is a Contrast enhanced CT (CECT) absolutely indicated in pancreatitis?

Acute severe pancreatitis pt ≥72hrs after patient's low-grade fever, leukocytosis, and recurrence of abdominal pain and nausea are concerning for a possible early complication of AP. 1. pancreatic necrosis, 2. acute necrotic collection, 3. acute peripancreatic fluid collection, or 4 . pancreatic infection (eg, infected necrosis).

How is pylo treated in pregnant pt?

Admit and start BSA ceftriaxone

What IV medication is used to treat gram negative septicemia

Aminoglycosides (eg gentamycin) used for systemic gram negative infection eg Septicemia - nosocomial RTI - complicated UTI - intrabdominal infection

What medication(s) are recommended for the prophylactic treatment of penetrating cat bites?

Amoxicillin/clavulanate and second-generation cephalosporins

What are some indicators of hemolysis?

An elevated reticulocyte count indicates hemolysis as the cause of anemia; along with - elevated indirect bilirubin level, - decreased haptoglobin bc rbc binds to it. - increased LDH, - splenomegaly)

G2 P1 W/ blood work shows that anti-D antibody titers are 1:32. Which is the most likely explanation for the positive antibody results in this patient?

An inadequate dose of anti-D immune globulin postpartum is the most likely cause of anti-D alloimmunization (eg, positive antibody screen) in this patient. Inadequate dose of anti-D immune globulin postpartum 2/2 fetomaternal hemmorage (abruptio placentae)

is a common cause of sudden cardiac death in young athletes. Patients may have premonitory symptoms of exertional angina, lightheadedness, or syncope. ECG and transthoracic echocardiogram can be nondiagnostic.

Anomalous aortic origin of a coronary artery (AAOCA)

What diagnosis should be suspected in someone with 3 miscarriages and positive Lupus anticoagulant, Anticardiolipin antibody and Anti-beta-2 glycoprotein antibody

Antiphospholipid syndrome diagnosed with at least 1 lab and clinical finding. Labs: 1. + lupus anticoagulant 2. anticardiolipin ab (MARKER FOR CLOT FORMATION) 3. AB2GP AB Clinical: >/3 miscarriages (prior to 10th week) venous and arterial thrombosis

diastolic heart murmur causes and associated condition?

Aortic insufficiency or Mitral stenosis (AIMS) associated with CVA

What improves long term survivability in those with AS

Aortic valve replacement reduces mortality even in a 70 year old man! Ballon valvulotomy is only done as a bridge to valve replacement.

How does thyrotoxicosis/hyperthyroidism manifest in older individuals

Apathy, depression, fatigue, proximal myopathy weakness, afib, weight loss and decrease appetite without the characteristic hyperadrenergic symptoms

What should be done for someone with anogenital wart due to condyloma acuminata?

Application of Trichloroacetic acid

What are the cardiac causes of syncopy and what test should be ordered?

Arrythmia and blood flow obstruction (eg in aortic stenosis) Order of EKG and PE for orthostatics

PAD treatment and stroke prevention PAD symptomatic treatment meds?

Asa (or clopidogrel/plavix) and statin symptomatic = ciloztazole after smoking stopped and walking started

____ is often treated with intravenous voriconazole and may occasionally present with a clinical profile similar to that of rhino-orbital mucormycosis; however, patients are typically neutropenic, not diabetic.

Aspergillosis

What are the autonomic prodromal symptoms of syncope? Which 2 causes of Cardiac syncope do not show any preceding syptoms?

Autonomic: N, pallor, diaphoresis, warmth Absent in Vtach, TDP or benign syncopal event

When does Tertiary HPT result in as far as PTH secretion? What is the treatment?

Autonomous PTH secretion THAT IS UNRESPONSIVE TO HIGH CA LEVELS! TERTIARY (3 HIGHS) HIGH P04 retention HIGH SERUM FREE CA SUPER HIGH PTH LOW 1,25 DIHYDROXYV VD levels --> Indication for Parathyroidectomy = persistent high CA or PO4; HIGH PTH; calciphylaxis, intractable bone pain.

Cat-scratch disease generally presents with a cutaneous lesion and regional lymphadenopathy following a cat scratch or bite. ____ treatment can reduce the length and severity of symptoms and what other meds can be given for lympadenitis?

Azithromycin along with antistaphylococcal and streptococcal coverage (eg, clindamycin) added empirically treat the most common organisms causing lymphadenitis.

Management of shoulder dystocia (BE CALM)

B- Breathe; do not push E- Elevate legs & flex hips, thighs against abdomen (McRoberts) C- Call for help A- Apply suprapubic pressure L- EnLarge vaginal opening with episiotomy M Maneuvers: Deliver posterior arm Rotate posterior shoulder (Woods screw) - apply pressure to anterior aspect of the posterior shoulder - Adduct posterior fetal shoulder (Rubin) - apply pressure to the posterior aspect of the posterior shoulder Mother on hands & knees -"all fours" (Gaskin) Replace fetal head into pelvis for cesarean delivery (Zavanel

Kawasaki (aaa iiii aka kAwAsakIII IVIG ASA TREATMENT IN THE NAME) disease what should be done in a person with kd that was treated w ivig

B/L conjunctivitis Unilateral cervical LAD Strawberry tongue Rash of hands and feet ***Tx with ASA*** avoid live vaccine if treated with IVIG 11 MO AFTER IVIG

The Schilling test can be used for diagnosis to determine if the

B12 deficiency is caused by pernicious anemia. by administering oral radiolabeled vitamin B12 f/b injected B12 that is unlabeled. The unlabeled B12 then saturates the receptors in liver and body tissues, and allows the radiolabeled B12 to be absorbed into the GI tract and pass into the urine. pernicious anemia have impaired absorption of vitamin B12 and therefore the level of radiolabeled B12 in the urine will be low (<10%).

Someone with ataxia, dementia, delerium and with pancytopenia and inc MCV warrents treatment with what first? What triad is seen w/ thiamine deficit due to etoh consumption present as? What treats do they have in common with the above?

B12 first then folic acid (B before F in alpha) Triad of encephalopathy, oculomotor dysfunciton, gait ataxia

pt s/p pth adenectomy with hemifacial paralysis has what disorder?

BELLS PALSY DUE TO VIRAL INFEXN signs of hypocalcemia are bilaterally symmetrical. Hemifacial involvement is not typically seen; therefore, with a preceding history of upper respiratory infection and facial asymmetry, the most likely diagnosis of this patient is Bell's palsy (lower motor nuclear involvement of the facial nerve).

which thyroid cancer is vascular and hematogenously seeds mets to BLBL?

BLBL bone lung brain and liver hematogenous spread is characteristic of follicular § iodine deficit maybe. Hurthel cell varient is resistant to iodine ablation. (falling may be a hurdle )

What is the difference btw breast milk jaundice and breastfeeding failure jaundice?

BMJ: presents after 3 days and peaks at 2 weeks BFJ: occurs in first week of life!

What is the gold standard for diagnosis osteomyelitis?

BONE BX

What is the treatment of hepatorenal syndrome and when should you suspect it? HINT MOAN

BUN/Cr ratio of 20:1 - failure to respond to volume challenge would be consistent with hepatorenal syndrome. MOAN MIDODRINE OCTREOTIDE NE ALBUMIN tx: octreotide and midodrine or norepinephrine alone Albumin should also be continued for at least 2-3 days.

In a SSRI partial responder what is a good add on drug that can activate antidepressent effect without causing weight gain or sexual dysfunction?

BUPROPRION

ACS Intervention

Basic life support measure Oxygen to keep SpO2 >90 ASA 325 Nitroglycerin Start Beta blocker and statin STAT (unless acute HF, or Liver dysfunction) P2Y12 blocker - plavix or ticagrelor unfractionated heparin

What should be done for someone with dark steaks in vision floaters and visual haze? no red reflex and hemorrhage on exam

Bed rest elevate bed 45 degrees for vit. hemorrhage.

What is the preferred initial treatment for preschool-age children with attention-deficit hyperactivity disorder vs those older than 6?

Behavioral therapy is the preferred initial treatment for preschool-age children with attention-deficit hyperactivity disorder. older child can get pharma 1st line

What is the difference btw Bells and the sign present in hypocalcemia?

Bell's is a LMN hemifacial paralysis while Chovosek sign is due to tapping causing hemifacial spasm (bilateral ) dont confuse. Trousous- is carpal spasms in hypocalcemia HyperParathyroid dx: low Po4, high serum CA s/p parathryoidectomy: Hypoparathyroid occurs = increased posphate and LOW SERUM CA promoting hungry bone s/o --> BONES BECOME HUNGRY TO SUCK UP CA AND LOWER SERUM CA

When is MHT menopausal tx helpful

Beneficial Menopausal symptoms (eg, hot flashes, vaginal atrophy) Bone mass/fractures Colon cancer Type 2 diabetes mellitus All-cause mortality (age <60) MHT can be safely used for a short period (3-5 years) in younger, low-risk women (ie, nonsmokers; no history of breast cancer, venous thromboembolism, or CHD).

Metformin's major side effects besides GI upset are? What is a serious consideration ?

Besides n, abdominal pain and diarrhea, metformin causes: 1. b12 malabsorption 2. lactic acidosis --> can be fatal especially if crt >1.5 or GFR <60 3. Those who will have Iodine contrast (heart cath) should dc metformin day of procedure and restart 2 days after prevent sever Lactic acidosis (or when crt stable) contraindications include renal insufficiency, hepatic dysfunction, ETOH abuse, sepsis, or CHF.

What drug improves short term morbidity ( REINFARCTION ) and long term MORTALITY after MI?

Beta blockers decrease M&M after recent MI they also improve survival in pt with CAD and LVSD

____ is an inherited blood disorder that causes marked microcytic anemia with codocytes (target cells) on a peripheral smear and is nearly always asymptomatic.

Beta-thalassemia minor

What p/w anterior shoulder pain, worst near the bicipital groove. Pain exacerbated with supination and/or flexion of the elbow. ____ and speeds tests may be positive.

Bicipital tendonitis: Diagnosis: + Yergason's and Speed's Treated: 1st: conservative: rest, activity modification, physical therapy, and oral nonsteroidal anti-inflammatory drugs (NSAIDs). 2nd: corticosteroid injection into the bicipital groove is indicated.

What is the treatment for Pagets ? what is common with disease?

Bisphosphonates, cotton wool spots incidentally found on head skull imaging c/b hearing loss

Bone metastases and hypercalcemia of malignancy have what level of vit d and pth?

Bone metastases Breast Multiple myeloma ↑ Osteolysis ↓ PTH & PTHrP ↓ Vitamin D

Helicobacter pylori infection in patients with peptic ulcer disease, affecting ____. These regimens are also known as triple therapies and have reported cure rates from 85% to 90%. The treatment regimens are _____ (hint 3 times BOL)

Both gastric and duodenum OAC x10 days OMEP, ammoxicil, clarithromycin BMT: bismuth subsalicylate metro and tetracycline x 14d LAC lansoprazol amox clarithromycin x14d

Does combined OCP decrease risk of ovarian or endometrial cancer?

Both.

Baby <12mo with poor feed, decreased salivation and ptosis needs what treatment ig gag reflex is intact?

Botulism immune globulin Intravenous botulism immune globulin (BIG-IV) should be given ASAP to reduce the severity/ duration of symptoms. Intubation can be done after Big IV

contraindication to breastfeed in infant vs mom?

Breastfeeding contraindications Maternal 1. Active untreated tuberculosis 2. HIV infection* (unless 3rd world) 3. Herpetic breast lesions 4. Active varicella infection 5. Chemotherapy or radiation therapy 6. Active drug use Infant Galactosemia

__ is considered the gold standard to diagnose tracheomalacia.

Bronchoscopy

Amaurosis fugax is a sudden and transient monocular blindness and is associated with what PE finding?

CAROTID BRUITS Marker of carotid artery atherosclerotic disease that is usually advanced.

What is the treatment with someone with symptoms of echolalia , mutism, immobillity or purposeless activity?

CATATONIA associated with waxy flexibility is treated with LORAZEPAM BENZO and ECT to prevent malnutrition hyperpyresia injury etc

Systolic heart murmur CAUSES AND ASSOCIATION?

CAUSES: MRmASTR MR AS TR associated with CHF!!!

What initial labs should be ordered in a 3 year old who is found eating weird stuff outside who has mild periumbilical tenderness?

CBC serum FE ferritin levels retic count if suspecting lead poisoning to detect if anemia or fe deficiency is present Measure of blood lead levels is needed for treatment plan however

What is the management for someone with typical biliary colic symptoms and no gallstones on imaging?

CCK (CHOLECSTOKININ-stimulated cholescintigraphy) to evaluate for functional gall bladder d/o. Cholecystectomy in those with low gallblader ejection

Patients with severe acute pancreatitis, signs of sepsis, or evidence of deterioration + abdominal pain >72 hours after presentation should undergo what test? What if they do not have abdominal pain?

CECT abdominal CT scan with contrast to evaluate for potential complications or evidence of infection. PIP TAZO if no pain and hypotension as it is more likely due to infection.

Which condition affects the nerve roots and causes LMN signs ?(options = CES vs CMS Cauda equina Syndrome or conus medularis syndrome)

CES presents solely with lower motor neuron signs because it affects the nerve roots themselves and not the spinal cord.

What level of CK in an asymptomatic patient on statins warrent discontinuation of therapy?

CK >10x ULN (upper limit of norm)

What is used to treat diaper rash and what are the 2 causes?

CLotrimazole cream

How does conus medullaris syndrome present?

CMS can present with UMN signs such as spasticity and hyperactive reflexes are present BELOW the lesion in addition to LMN signs ( muscle flaccidity and loss of reflexes ) at the level of the lesion.

What is one key feature that distiguishes rubella from CMV congenital torches infection?

CMV has periventricular calcification on head CT (toxo also has calcifications) also german measules rubella has microcephally and PDA

What nerve is damaged during carotid endartarectomy?

CN 12- lick wounds CN 7 mandibular bbr assymetric smile (obicularis ori) RLN- hoarseness from vocal cord paralysis.

What childhood disorder is termed antisocial personality after 18yo

CONDUCT d/o: characterized by antisocial behavor, common in adhd child and characterized by: 1. stealing 2. fighting 3. destroying objects 4. forced sexual activity

Acute painless monocuar vision loss with fundoscopic exam showing ischemic retinal pallor and CHERRY RED MACULA IS ?

CRAO treat with urgent optho. consult and lower IOP cherry red spot is due to preserved circulation via posterior cilliarry artery occurs in age >60 with carotid atherosclerosis --> opthalmic artery occlusion

Corticosteroids are the treatment of choice for what 2 liver conditions?

CST for autoimmune hepatitis and acute alcoholic hepatitis.

When would CT be waranted in abdominal pain (2 causes) and when would abdominal ultrasound be a good initial test?

CT abdm. done for pancreatitis and ischemic coitiis Abdm, ultrasound for gallbladder disease

What is the best modality to diagnose patients with suspected AAOCA- Anomalous aortic origin of a coronary artery?

CT coronary angiography or coronary magnetic resonance angiography provide the best visualization of coronary anatomy, and are the diagnostic tests of choice

____ of the head is a fast and cost-effective method for detecting subdural hematomas in infants with abusive head trauma.

CT scan: subtle overlooked symptoms of irritability and vomiting 2/2 increased ICP skeletal survey may be performed to evaluate for other injuries suspicious for abuse only in stable child: 1. posterior rib fractures, 2. metaphyseal corner fractures, and 3. fractures in different stages of healing.

Once the diagnosis of SEA is confirmed on imaging what is needed to guide antibiotic therapy? and what intervention?

CT-guided aspiration and culture are needed to guide abx (Staphylococcus aureus accounts most SEA) - immediate surgical decompression ( 24 hours) to avoid complications of cord compression or cauda equina syndrome.

gold standart dx for PE?

CTPE or ct angio of the thorax which requires contrast if renal failure pt then VQ scan

____is a craniosacral treatment that can enhance the amplitude of the CRI. It is done by first resisting the flexion phase and encouraging the extension phase of the CRI until a "still point" is reached, then allowing the restoration of normal _____ to occur.

CV4 flexion and extension

In a person with gaurding and acute abdominal pain and history of gerd what diagnostic imaging should be ordered? What is the likely dx and how do pt like this present?

CXR and abdominal x-ray to dx bowel perforation with peritonitis eg in someone with a perforated PUD Pneumoperitoneum/ Peritonitis pt tend to lie flat and motionless to limit peritoneal irritation

does NF1 and 2 both have cafe au lait macules (CALMs)

Calms is pathonmoumoic for NF1. NOT IN NF2. Also nf2 require audio-logic eval for sensorineural hearing loss

What are differentials for non resolving pneumonia?

Cancer: - Bronchogenic carcinoma, - bronchoalveolar cell carcinoma, - lymphoma, Infection: eosinophilic pneumonia, Systemic: -bronchiolitis (BOOP), -vasculitis, drugs (amiodarone).

ultrasound of the thyroid showing hypoechoic, microcalcifications, internal vascularity and serum TSH is high/ normal indicates what type of thyroid nodule?

Cancerous: high/norm tsh levels have a higher risk of malignancy and should be evaluated with FNA. -low TSH levels should be evaluated using thyroid scintigraphy, usually with iodine 123

What tb testing can differentiate between active and latent TB ?

Cannot differentiate between latent & active TB - Nucleic acid amplification - Sputum culture - Smear microscopy TB skin test only diagnoses latent TB (not active) Quant gold: cannot differentiate btw latent and active

What is the first-line agent for temporal lobe epilepsy, and trigeminal neuralgia.

Carbamazepine

When are serial ekg and enzymes needed in a suspicious ekg?

Cardiac enzymes and serial ECGs are appropriate for patients with suspected but undiagnosed ACS.

____ shock: Decreased CO, Increased PCWP, Increased PVR

Cardiogenic shock

dose related cardiotoxicity is due to which chemo drug?

Cardiotoxicity with anthracyclines is strongly related to cumulative doses but trastuzumab-associated cardiotoxicity is not dose-related.

What is the leading cause of mortality in SLE and why?

Cardiovascular events due to atherosclerosis as SLE patients are prone to premature development of coronary atherosclerotic disease and coronary artery disease in younge women! This is due to chronic inflammation with GCorticoid use superimposed upon usual HLD and HTN risk compared to other 35-44yo women those with SLE have a 50 fold increase in CAD risk! coronary vasculitis is rare sle complication and doesnt lead to death

Amaurosis fugax requires what evaluation is necessary to evaluate the extent of the disease and to assess the need for a carotid endarterectomy.

Carotid Doppler

What type of study is used to ID an outbreak of something?

Case control

Acne treatment during pregnancy which ones are category c d and x and what do each mean ? hint think of human studies as C and D c - cohort d - demolition x - no sex

Category C: (animal risk or human risk cohort) Adapalene, tretinoin, topical dapsone, benzoyl peroxide, sodium sulfacetamide Category D: (fetal risk in postmarket human) Oral tetracyclines Category X: Tazarotene, isotretinoin

what syndrome can be distinguished by isolated lower motor neuron signs with associated polyneuropathy which sometimes presents with perineal paresthesia and bowel and bladder incontinence.

Cauda equina syndrome = isolated LMN signs with , polyneuropathy which sometimes presents with perineal paresthesia and bowel and bladder incontinence and can be seen in IVDU epidural absecess

Name the 4 causes of subclinical hyperthyroidism?

Causes 1. Exogenous thyroid hormone 2. Graves disease 3. Nodular thyroid disease (thyroid adenoma or toxic multinodular goiter) 4. inflammatory thyroiditis (eg, silent thyroiditis, subacute [de Quervain] thyroiditis).

CAP vs HAP MGMT ?

Ceftriaxone with azithromycin = community-acquired pneumonia. hospital-acquired pathogens IF suspected, empiric treatment should include an antipseudomonal agent (eg, piperacillin-tazobactam).

What GI syndrome results in microcytic anemia and which results in Macrocytic anemia?

Celiacs - microcytic SIBO- small intestinal bacterial overgrowth- macrocytic anemia due to B12 deficit

TCA overdose CNS effects and Cardiovascular

Central nervous system drowsiness, delirium, coma, respiratory depression-- inhibit central GABA receptors and can cause seizures. Cardiovascular Sinus tachycardia, hypotension Prolonged PR/QRS/QT intervals Arrhythmias (eg, ventricular tachycardia, fibrillation)- why NA bicarb is given

How long after treatment is HG checked for empiric treatment of iron deficit anemia in a child?

1 mo of empiric iron supplementation, an increase in hemoglobin ≥1 g/dL supports the diagnosis of iron deficiency anemia. - Ferrous sulfate should be maintained for 2-3 months after hemoglobin normalization to replenish iron stores. Within 1-2 weeks of initiating supplementation, the reticulocyte count increases quickly.

When should child be screened for Iron deficiency

1 year especially in premi, lead exposed, breast feed for >6mo, drinking >24 oz of mild

Hypercalcemia of malignancy with increase vitamine D indicates which cancer?

1,25-dihydroxyvitamin D Lymphoma ↑ Calcium absorption ↓ PTH ↑ Vitamin D

small cell cancer in lung causes what 3 paraneoplastic s.o

1- LEMS 2- CUSHING ACTH LIKE SUBSTANCE 3- SIADH 4- ENCEPHALOMYELITIS AND CEREBELLAR DEGENERATION

power =

1-beta

What is considered a risk factor during preconception counseling?

1. AMA ≥ 35yo 2. chronic dx 3. poor ob hx 4. FH genetics 5. Social mental health (domestic violence) Interventions : 1. Medication changes, 2. specialist referral's, 3. vaccine, 4. abstention from etoh and drug, 5. smoking cessation , 6. healthy eating habit, 7. folic acid supplements General use of seatbelts during driving

Ascending aortic dissection complications include?

1. AR 2. RCA occlusion --> ACS 3. HEMOPERICARDIUM = TAMPONADE 4. stroke carotid artery occlusion 5. horners (sg compression) 6. vocal cord paralysis (RL NERVE PARALYZED)

What are the post-op pulmonary complication risk factors in a patient?

1. Age >50 2. Severe COPD 3. cigarrette smoking (unless stopped for 1mo) 4. OSA 5. Pulm. HTN 6. CHF 7. poor health and metabolic health eg serum alb ≤3g/dl

What are the clues to diagnose the following cardiogenic syncopal episodes?

1. Aortic stenosis HCM: exertion syncope + systolic murmur 2. VTach: sudden syncope; cardiomyopathy or past MI 3. SSS: fatigue/dizziness before syncope; sinus pause on ECG 4. Advanced AV block: Bifascicular block or Inc PR; Dropped beat 5. TDP: sudden syncope; QT-prolonging meds, LOW K / MG

Name the 5 causes of cardiogenic syncopy?

1. Aortic stenosis or HCM 2. Vtach 3. Sick sinus syndrome 4. Advanced AV block 5. TDP

Pt with positive stress testing in the initial evaluation for stable CAD should be treated with what?

1. Asprin 2. high intensity statin 3. beta blocker 4. optimizing CV risk factors - smoking - BP control - glucose control

Lifestyle therapy for postmenopausal porosis (3 things) VS Pharmacologic therapy: First-line_____

1. Avoid smoking & excessive alcohol 2. Regular weight-bearing exercise 3. Total intake (diet + supplement): - calcium ≥1200 mg/day, - vitamin D ≥800 IU/day Indicated for osteoporosis or high-risk osteopenia*: oral bisphosphonates

Anemia, thrombocytopenia in the setting of CLL (or any bone marrow infiltrating disease) requires what further management?

1. Bone Marrow Biopsy 2. Refer to hematology and oncology - Bx: to see if B cell infiltration responsible for bone marrow failure in the setting of anemia where reticulocyte count is inappropriately low in the setting of elevated WBC, Anemia, thrombocytopenia not responsive to treatment

Progestins prevent pregnancy by?

1. CERVICAL MUCUS THICKENING hence preventing sperm migration + low tubal motility Suppression of ovulation by decreasing FSH and LH Progestin subdermal implant is the MOST EFFECTIVE CONTRACEPTIVE METHODE

B12 deficiency is also seen in what 4 disease states?

1. Crohn disease 2. terminal ileum rescection 3. chroic pancreatitis 4. Diphyllobothrium latum consumption.

Absence of what 3 features in someone who has elevated bilirubin and ALK phose out of proportion to ast alt along with biliary outflow obstruction warrants MRCP?

1. ERCP done if choledocholitiasis is visulaized 2. high-risk features (eg dialated CBD on imaging elevated serum bili 3. evidence of acute cholangitis (fever RUQ pain jaundice hypotention and confusion_ - these patients require IV abx and treatment of sepsis

Most common bacteria causing acute cholangitis are

1. Escherichia coli and 2. Klebsiella. both GM negative enteric rods

What predisposes infant to develop hypertrophic pyloric stenosis?

1. Firstborn male 2. Use of macrolide (erythromycin and azithro.) for pertussis treatment in first few weeks of life. 3. bottle feeding (4 fold increase)

What agents (3) inhibit peripheral deiodination of t4 to t3?

1. GC 2. CC 3. PTU thus they are used in treatment of thyrotoxicosis and thyroid storm

What are the absolute indication for immediate Coronary angio despite TIMI score?

1. Hemodynamic instability 2. HF or new MR 3. recurrent CP 4. Ventricular arrythmia

Pathophys of CVD? Why would depression worsen outcomes M&M?

1. Increased inflammation 2. endothelial cell dysfunction 3. abnormal plt. 4. inc cortisol 5. inc sympathetic tone Depression worsens outcome due to less motivation to medication adherence, lifestyle change, and dec. mobility.

What testing needed for Turners baby?

1. Karyotype if key physical findings 2. 4 extremity BP measure 3. Echo to check for aortic coarctaiton (can cause cardiogenic shock in bby) 4. US for horseshoe kidney can cause UTI Child screen hypothyroid and celiac due to inc risk of these and neuropsych for learning disabilities (IQ NORMAL THO)

Describe leriche syndrome? Hint its a triad.

1. LE claudication, absent or diminished femoral pulse and ED

When do you have to get a medical abortion after being knockedup because it is contraindicated to be preggers??

1. LV EF<40 2. class 3-4 HF 3. prior cardiomyopathy from pregnancy 4. severe obstructive heart lesion 5. severe pulm HTN (EISENMENGER S.O) 6. unstable aortic dilation >40mm

What is an absolute contraindication to using combined OCPs?

1. MIGRAINE +aura (inc. risk for ischemic stroke) 2. BP ≥160/100mmHg 3. Age ≥35Yo who smoke ≥15cig./day 4. DM with end-organ damage

Name the DMARDS

1. MTX: folate antimetabolite 2. Leflunomide: pyrimidine synthesis inhibitor 3. HCQ: TNF IL1 suppression 4. Sulfasalazine: TNF and IL1 suppression 5. TNF inhibitors: adalimumab, certolizumab, etanercept, golimumab, infliximab (aka biologics)

Adverse effects of the DMARS include?

1. MTX: hepatotoxic, stomatitis, cytopenia 2. Leflunomide: hepatotoxic and cytopenia 3. HCQ: retinopathy 4. Sulfasalazine: hepatotoxic, stomatitis, hemolytic anemia (sulfa drugs maha) 5. TNF inhib: infection, demyelation, CHF, malignancy

What is the management of unstable angina and NSTEMI

1. NITRATE 2. BB: cardioselective metoprolol and atenolol, IV for HTN pt 3. ANTIPLATELET: asa and p2y12 receptor blocker (clopidogrel) 4. ANTICOAGULATION: UFH, enoxaparin, bivalirudin, fonduparanox 5. STATIN: high intensity 6. coronary reperfusion: perc within 24 hrs

ADHD is associated with development of what condition during childhood? How is the IQ?

1. ODD oppositional defiant d/o 2. conduct disorder 3. tourettes syndrome GOOD IQ

What are 4 relative contraindications to lymphatic treatments:

1. Osseous fractures 2. Bacterial infections with a temperature >102F 3. Abscess or localized infection 4. Certain stages of carcinoma

Besides serum sickness what are other extra-hepatic manifestations of HEP B ?

1. PAN 2. Glomerulonephritis

In pheochromocytoma during operation what is given if hypertensive? What is given if hypotensive?

1. Phentolamine for htn 2. FLUID BOLUS NS if hypotensive

What are the causes of PTH dependent hypercalcemia?

1. Primary or tertiary HPTH 2. familial hypocalciuric hypercalcemia 3. lithium these people have both elevated ca and high normal pth.

What are the etiologies leading to ED and which is sudden onset ED, and which is associated with loss of libido, hair loss, and testicular atrophy

1. Psycological (ABRUBT ONSET) vs Organic Organic causes: 2. Vascular 3. Neurologic: MS, Cauda equina, lumbar herniation - present with incontinence, LBP, weakness leg (red-flags) 4. Endocrine: Hypogonadism, also have loss of libido

What is THE MOST common cause of oligohydramnios

1. RENAL AGENESIS 2. rupture of membranes. In this situation, as soon as oligohydramnios is diagnosed on ultrasound, one should also evaluate the fetal kidneys during ultrasonic exam

Indication for valve replacement?

1. Severe AS and ≥1 of the following: Onset of sx angina syncope etc LVEF ≤50% Undergoing other cardiac surgery (eg CABG)

How is HCM managed stepwise?

1. Start on negative ionotropic agent - beta blockers: metoprolol -verapamil - disopyramide 2. if unmanaged with beta blocker may addo on verapamil or disopyramide 3. last line is alcoho septal ablation this is only for those with persistant symptoms

Factors that will decrease the rate and quality of the CRI AKA <8?

1. Stress (emotional or physical) 2. Depression 3. Chronic fatigue 4. Chronic infections

What is the best order to treat wernike's encephalopathy

1. Thiamine 2. Thiamine followed by Iv crystalloid and dextrose never vice versa!

Restrictive transfusion is associated with less risk of what complicaitons?

1. Transfusion reactions 2. fewer rebleeding episodes 3. dec. mortality

What would be considered primary prevention of disease?

1. VACCINATION 2. DIET 3. NO SMOKING

Factors that will increase the rate and quality of the CRI AKA >12

1. Vigorous physical exercise 2. Systemic fever 3. Following OMT to the craniosacral mechanism

what are the 4 notable lab findings of autoimmune hepatitis and how is it trated

1. ana 2. anti smooth muscle ab asm- ab 3. anti liver kidney microsomal ab - alkm 4. anti mitochondrial ab - ama treatment: steroids

respiratory alkalosis include hyperventilation and what 3 other major causes

1. anxiety states 2. early aspirin ingestion 3. sepsis.

Tuberous sclerosis complex is a genetic syndrome that leads to what manifestations? What screening is needed for TSC?

1. benign tumors throughout the body hypopigmented 2. central nervous system involvement. Tumor screening Regular skin & eye examinations Serial MRI of the brain & kidney Baseline echocardiography & serial ECG Baseline electroencephalography Neuropsychiatric screening The initial screening: 1. skin examination, 2. central nervous system imaging 3. and abdominal imaging, and 4. electroencephalography if seizures are suspected.

In pt with WBC 18500, colonic dilatation with air fluid levels and no pneumoperitoneum what should be the management?

1. bowel rest admit to ICU 2.NG tube placement 3. avoid meds that decrease peristalsis (opioids or anticholinergic) 4. fluid and electolyte repletion (eg postassium )

Administration of 1. ____ antibiotics and 2. correction of ______ constitutes essential medical care for cholangitis. These patients will usually require ______ to diagnose the cause of the obstruction of the bile ducts.

1. broad-spectrum intravenous 2. fluid and electrolyte imbalances an endoscopic retrograde cholangiopancreatography (ERCP)

what 3 labs would you order for pt with - anemia, - thrombocytopenia, and - icterus or jaundice in an otherwise asymptomatic patient should be concerning for some kind of hemolytic process. veronica

1. cbc with diff 2. liver profile 3. PBS ~~ haptoglobin, LDH, and retic count: may be obtained to check hemolysis

First-line antibiotic therapy for acute cystitis during pregnancy includes (3 drugs).

1. cephalexin, 2. fosfomycin, and 3. amoxicillin-clavulanate.

Shockable rhythms include pulseless ventricular tachycardia or ventricular fibrillation. If the patient is stable, you can attempt ____ If the patient is unstable (abnormal vital signs, unresponsive), treatment is with _______

1. chemical cardioversion using amiodarone or lidocaine. 2. unstable = defibrillation, using 360 J of energy with a monophasic defibrillator, or 200 J with a biphasic defibrillator.

When should screening endoscopy for metaplasia (barrets) be done?

1. chronic gerd with ≥2 alarms 1. ≥50yo 2. male 3. white 4. hiatal 5. obesity waist 6. smoker 7. 1st deggre relative with BE/EAC

The initial step in management of OE otitis externa swimmer ear

1. clean ear with curette to visualize and irrigate with dilute hydrogen peroxide - mild OE: acetic acid 1 week - moderate OE with more pain and puritis: topical cipro, neomycin/polymyxin) - severe presents with systemic signs and periauricular redness - treat with systemic abx cipro (brand cortisporin = poly.neo with steroid)

What could prevent cellulitis in pt who has a repeat infection?

1. compression stocking 2. abx prophy for the duration of predisposing risk factors

What are the red flags of LBP?

1. constitutional symptoms ( lbs, age >50, IVDU, nocturnal pain, malignancy hx 2.

Untreated OSA has what serious adverse outcomes, including drowsy driving, (name 4)

1. coronary artery disease --> heart failure, 2. group 3 pulmonary hypertension, prominent S2 (P2) on exam 3. metabolic syndrome, and 4. nonalcoholic fatty liver disease (NAFLD).

How would a pt with sicca syndrome present?

1. dry mouth (xerostomia) 2. dry eyes (keratoconjunctivitis sicca) 3. common in elderly (prevalence may be as high as 35-38%)

What is virchows triad?

1. endothelial injury 2. abnormal blood flow 3. hypercoagulability

Clostridium difficile is the most common cause of nosocomial diarrhea. Appropriate preventive measures include

1. facility infection rate monitoring, 2. hand hygiene, 3. contact precautions, and 4. limiting antibiotic use. 5. Hand washing with soap and water is more effective at spore elimination than alcohol-based hand sanitizers. *NO Repeat stool testing. (>60%) of patients can remain positive for C difficile even after successful treatment. Stool culture and antibiotic susceptibility are not routinely recommended.

Biophysical profile is ordered for what fetal findings? What of what 5 components of the BPP ultrasound ?

1. fetal movement, 2. tone, 3. breathing, and 4. amniotic fluid volume, 5. evaluates for fetal hypoxia after an abnormal nonstress test (eg, decelerations, no accelerations).--- think of swimming pool per mom

Diagnosis of cholangitis requires ____ with laboratory evidence of cholestatic liver enzymes.

1. fever and evidence of systemic inflammation (elevated white count) 2. labs with cholestatic liver enzymes 3. imaging demonstrates ductal dilation or an underlying etiology (eg, stone, stricture).

what is the most common cause of pna in baby up to 20days old? what about after 3 weeks and up to 3 months =?

1. gbs, 2. e-coli, listeria (leg) 2. viral (eg rsv, parainf, influenza a, b, adeno), strep pna, chlamidia trachomatus, hib )

What is commonly done to prevent a catheter related blood stream infection? Does position of catheter or replacement of catheter help decrease infection? How about prophylaxis w/ abx?

1. hand hygine 2. sterile barrier during placement 3. chlorhex prep of skin prior to insertion 4. cath site care 5. removal only when cath is not needed Antibiotics only add to resistant organism cath placement location is not a big deal guidewire use or replacing cath worsens infection risk

Hyperventilation syndrome leads to respiratory alkalosis secondary to hypocarbia, with resultant ___ calcium and causes what physical finding?

1. hypocalcemia is secondary to the hypocarbia; 2. Carpopedal spasm occurs when acute hypocarbia causes LOW ionized calcium and phosphate levels, -CO2 levels drop from tachypnea --> H ions released from Albumin. - Cationic Albumin binds calcium --> hypocalcemia.

AIN is caused by what 5 factors? 1. what disease state 2. what analgesic 3. what 2 antibiotics 4. which AED

1. infection 2. NSAID 3. penicillin 4. cephalosporins 5. phenytoin

What are the major causes of PTH independent hypercalcemia

1. malignancy 2. vitamine D toxicity (inc CA absorption), vit. A 3. thiazide (drugs) 4. milk alkali 5. thyrotoxicosis 6. granulomatous disease

Which pathologies possess psammoma bodies? ( hint PSaMMoma)

PSaMMoma pneumonic PSaMMoma: Papillary (thyroid), Serous (ovary), Meningioma, and Mesothelioma.

How does central retinal vein occlusion present?

Painless, acute, or subacute 1 eye vision loss due to NON embolic source. Cotton wool spots.

What type of care should be given to someone with stage 4 cancer that was responsive to therapy and wishes to spend time with family with quality of life.

Palliative care can occur with life prolonging measures and leads to better mental health outcomes, satisfaction, and quality of life

_____are the most common cystic lesions of the pancreas, accounting for 75-80% of such masses.

Pancreatic pseudocysts are the most common cystic lesions of the pancreas, accounting for 75-80% of such masses.

pediatric condition treated with IM coriticosteroids?

Parainfluenza infxn of larynx and trachea Croup = CST (group of loud laryngotracheitis) - if mild can be suportive as it resolves in 1 week and humidifier is good to help with symptoms that are worst at night Mild: humidified air and cst moderate/sev. CST plus nebulized epi! Nebs added with CST in severe upper airway obstruxn. Clinical diagnosis requires featurs of 1. Insp. stridor (brassy cough on UWOrld) barking cough 2. hoarseness 3. fever precede 4. rhinorrhea precede X ray is not needed for dx!

(__ type of molar pregnancy ____) may show a fetal heartbeat on ultrasound and is associated with fewer symptoms (e.g hyper-emesis gravidarum, bHCG elevation)?

Partial mole presents with fewer symptoms. - viable heartbeat - normal karyotype (46XX or 46XY)

Previously vaccinated individuals who are potentially re-exposed to rabies should receive ____ ? Unvaccinated need:

Past vaccine: only a booster course of rabies vaccine (2 doses) because passive immunization with immune globulin blunts the antibody that is still circulating with prior vaccine! unvaccinated: both immune globulin and a full vaccination series (4 doses).

____ causes a characteristic continuous murmur that is heard best in the left infraclavicular area.

Patent ductus arteriosus (PDA) Moderate to large PDAs with left-to-right shunting cause volume overload of the left atrium and ventricle, resulting in left atrial/ventricle dilation and LV dysfunction.

Gestational diabetes mellitus PATHO? IS HGA1C ACCURATE?

Pathophysiology Human placental lactogen secretion Screening 24-28 weeks gestation 1-hr 50 g GCT 3-hr 100 g GTT NOT HGA1C due to high rbc turnover when pregnant

Patients who are not able to undergo PCI in a timely fashion should be considered for ____ therapy.

Patients who are not able to undergo PCI in a timely fashion should be considered for fibrinolytic therapy.

how many days off after concussion? what if returns while symptomatic?

Patients who are symptomatic should not be permitted to return to play if they are still exhibiting signs of a concussion otherwise 2nd hit causes cerebral edema --> death . An adequate amount of physical as well as cognitive rest is recommended before returning to play. (avg around 10 days )

does the nature of the triggering event or degree of weakness at presentation does not affect prognosis in guillain barre ?

Patients with GBS tend to have impaired neurologic recovery if the illness was preceded by a Campylobacter infection, they had severe motor weakness on admission, or required ventilatory support.

TREATMENT for pt with rectal examination warm, very tender prostate fever, dysuria, urinary frequency, and cloudy urine.

Patients with acute bacterial prostatitis usually have treatment with urgent bladder decompression (eg, suprapubic catheter) and antibiotics is required.

Patients with insulin-dependent type 1 or 2 diabetes mellitus typically require ---- during the postpartum period due to decreased insulin resistance after delivery of the placenta.

half of insulin regimen during the postpartum An insulin sliding scale can be added for additional glucose control.

How do you treat bipolar mania during pregnancy?

haloperidol Lithium has a relative risk of ebsteins anomoly but its absolute risk is low * and thus can be used in SEVER mania during pregnancy

A patient who presents with respiratory distress following neck surgery must be immediately evaluated for a possible _____.

hematoma Commonly tested postsurgery complications are bleeding, infection, and injury of relevant anatomy.

Listeria, vibrio and yersinia infections are most common in what hereditary condition

hemochromatosis

Urgent synchronized cardioversion is appropriate for what arrhythmia?

hemodynamically unstable supraventricular tachycardia

_____ should be ordered to confirm the diagnosis and distinguish α- from β-thalassemia minor.

hemoglobin electrophoresis and is normal in patients with α-thalassemia; β-thalassemia causes abnormally elevated hemoglobin A2 levels.

______ is given in hemorrhagic stroke to keep SBP at what goal?

hemorrhagic stroke, for whom target systolic blood pressure is around 140 mm Hg Nicardipine. - given IV for severe hypertension.

Pt with insomnia, altered mentation, asterixis and ATAXIA has what?

hepatic enceph. = inc ammonia levels due to an underlying trigger. 1. HE from excessive diuresis, which reduces intravascular volume and - results in hypokalemia and - metabolic alkalosis.

Causes of transudative effusion?

hepatic hydrothorax nephrotic s/o heart failure constrictive pericarditis

What is the early manifestation of hemochromatosis

hepatomegally is early while later on they get cirrhosis (HCC RISK TOO)

A rightward shift of the oxyhemoglobin dissociation curve indicates that the oxygen is being unloaded to the peripheral tissues and thus assists in the healing process and occurs when ____?

high temp hyperthermia, metabolic acidosis, increased 2,3 diphosphoglycerate, and hypercapnia.

RTA type 1 is associated with hypokalemia and ___ urine pH, with a predisposition for _____. and causes for nonanion gap metabolic acidosis.

high urine pH making predisposed for kidney stones!

Stimulant pharmacotherapy for children with attention-deficit hyperactivity disorder can be initiated if ___ and changed to what other med --name-- in no improvement

history and physical examination do not suggest cardiac disease. In the absence of positive cardiac history, further evaluation, including ECG, is unnecessary. disorder who have an inadequate response or fail to tolerate a stimulant can be tried on another. Other options include switching to a nonstimulant medication such as atomoxetine or an alpha-2 adrenergic agonist. References

Hypocalcemia following parathyroidectomy is caused by relative hypoparathyroidism and is called ____ sydrome? *Extremely important question for USMLE step-3

hungry bone syndrome. The signs of hypocalcemia are typically bilateral. Unilateral signs may suggest an alternative diagnosis. Hungry bone syndrome usually develops 2 to 4 days following surgery.

If pt SBP remains above 120 in aortic dissection what is the next drug given. Can hydralazine be used?

hydralazine causes reflex tachy and worsens dissection nitroprusside is a arterial and venous vasodilator and can be used AFTER adequate beta blockade if sbp remains above 120 in dissection.

A combination of ___therapy has been shown to provide additional symptomatic and mortality benefit in AA pt with persistent NYHA class III or IV symptoms due to left ventricular systolic dysfunction (LVEF <40%) not responding to optimal medical therapy. (ACE BB DIURETIC ALDOANTAG)

hydralazine plus nitrate espectially if they are EUVOLEMIC

How does MAHA present and what are lab findings? What is an association during pregnancy

hyperbilirubinemia, scleral icterus, Found in TTP with fever seizure coma and GI sx

HTN urgency management e.g who have severe hypertension (>179/119 mmHg) without symptoms and signs of end-organ damage are diagnosed with

hypertensive urgency. there is no proven benefit from rapid reduction of blood pressure in patients with hypertensive urgency. 1. myocardial or cerebral ischemia can be precipitated by overly rapid blood pressure lowering. 2. BP spontaneously declines with rest in a quiet room.

Metabolic findings in hypothyroidism?

hyponatremia

What would cause a low reticulocyte count?

hypoproliferative state due to: - renal disease, - hypothyroidism and - aplastic anemias

Common culprit of hepatic encephalopathy?

hypovolemia (over diuresis most commonly) hypokalemia - promotes ammonium-- ammonia conversion Metabolic alkalosis - decreases urinary loss of ammonia

1. ______ shock: Decreased CO, Decreased PCWP, Increased PVR

hypovolemic shock

How does one differentiate between SJS and TEN?

if greater than 30% involved = TEN <10 = SJS Lesions covering 10% to 30% of body surface area are referred to as SJS/TEN

How do you RO CAD in someone with OA

if severe and prevents walking for stress test get pharma stress testing 1. adenosine myocardial perfusion imaging 2. dobutamine echocardiography ( think of cardio dude giving isotope to chick)

In a cross sectional study how can one SUGGEST (NOT INFER) causality

if there is a dose dependent response with OR increasing with increased dose. Otherwise known as a biological gradient.

classic anticholinergic manifestations, such as ____GI MANIFESTATION___ and urinary retention.

ileus dry skin and mouth mydriasis and tachycardia (eg in addy similar)

G1PO 24 weeks presents with right side weakness, dull HA x 1 week, vision loss with valsalva Neuroimaging reveals intraluminal thrombus in the sagittal sinus and a small area of intraparenchymal hemorrhage. Which of the following is the best next step in management of this patient?

Patients with central venous sinus thrombosis P/W: gradually worsening headache due to elevated ICP, - focal neurologic deficits / seizure 2/2 cerebral venous congestion. Although focal cerebral hemorrhage may be present on imaging, T/W: heparin or low-molecular-weight heparin even if focal cerebral hemorrhage! it is still safe and and the standard of care

niacin (vitamin B3) deficiency causes? hint 3 things

Pellagra (ie, photosensitive dermatitis, diarrhea, dementia) - can be seen with

When is prasugrel as antiplatelat contraindicated and why?

Per TRITON TIMI trial pt with prior stroke should not be put on prasugrel or in those >75 YO, <60kg with h/o stroke since they are more prone to bleeds

intensely pruritic papules and vesicles between the fingers as well as on the volar wrists, elbows, axillae, lower abdomen, or genitalia. Crusted _____ can cause large, scaly patches but is not pruritic and occurs most commonly in -----?

Permethrin is used to treat scabies and crusted scabies, immunocompromised patients (eg, lymphoma, AIDS).

What other DM medication helps NASH, which cause weight gain?

Pioglitazone and Metformin help with nash, hyperTG, and HDL Pioglitazone (may be hepatotoxic), insulin and sulfonureas = weight gain.

--- is indicated when HELLP syndrome is complicated by fulminant hepatic disease (eg, aspartate aminotransferase >2,000 U/L or lactate dehydrogenase >3,000 U/L).

Plasma exchange

Management of TTP?

Plasma exchange

What does plasmapheresis treat?

Plasmapheresis: reduces levels of circulating autoantibodies in conditions like 1. Guillain-Barré syndrome, 2. myasthenia gravis 3. thrombotic thrombocytopenic purpura TTP

How is HELLP diagnosed and what is the most important prognostic indicator?

Platelet count § prognosis HELLP criteria: 1. Hemolysis (at least 2) on peripheral smear (schistocytes, burr cells), serum bilirubin (≥1.2 mg/dL), low serum haptoglobin, anemia, 2. Elevated liver enzymes AST or ALT ≥ twice upper level of normal, LDH ≥ twice upper level of normal and 3. low platelets (< 100,000/mm3).

What condition commonly affects those >50Yo who have pain in large muscles of shoulder and pelvis? What associated complication can occur? How is it diagnosed and treated?

Polymyalgia rheumatica (PMR) diagnosed clinically with ESR elevation used as confirmation Commonly co-occurs with GCA! which needs high dose GC (PREDNISONE 40-60MG QD) and considered for temporal artery bx.

Polypharmacy is defined as:

Polypharmacy is defined as the use of multiple (generally ≥5) drugs in a single patient, typically to manage different medical conditions.

When is rabies treatment effective? What are the treatment modalities? What is the prognosis ?

Poor prognosis, pending death, rabies treatment is effective PRIOR to symptom manifestation PEP includes Immune Globulin and Rabies vaccine series

HEP C is associated with what derm symptoms?

Porphyria cutanea tarda, leukoclastic vasculitis and lichen planus if advanced liver dx is due to hep C virus

SAAG >1.1 means what?

Portal hypertension is present likely 2/2 three causes 1. CHF 2. ETOH hepatitis 3. Cirrhosis

A fistula between what two vessels is created in a TIPs procedure? What is the end goal of TIPS?

Portal vein and hepatic veins to decrease hydrostatic pp in portal venous system.

How is acute hemoglobin transfusion reaction treated? What causes it?

Presentation: Fever, Flank pain, hemoglobinuria, DIC, +coombs Cause: Due to ABO mismatch Timing: 1-6hrs after transfusion Treat: Aggressive NS hydration - prevents renal failure Testing: Positive direct Coombs test, pink plasma free hg >25, mismatch cross.

What is pylephlebitis and what causes it ?

Presents with F, RUQ pain, jaundice, and hepatomegally. Portal vein thrombosis after having an untreated intra-abdm infection.

What does erythromycin eye ointment do for infant?

Prevent GHONOCCOCCAL eye infexn no chlamydia If prevention doesnt work treat with IM cef x1dose (presents at 2-5day of life)

Hyperpigmented lips, fatigue, muscle weakness, and loss of lbs in an adolescent girl Labs: 132NA, 5.1K, 107CL, 20Bicarb, increased white count t cells What are the dx and management

Primary adrenal insufficiency (addisons) - increased ACTH mucosal pigmentation - low aldosterone - low cortisol Get serum morning serume cortisol initially - if low then get ACTH stimulation test treat with steroids dual

What medication for HCP health care professionals exposed to HIV ?

Primary exposure prophylaxis with 4week COMBINATION 3 drug combo ideally started 1-2 hours after exposure. 2 nrti (tenofovir +1 other Ziduvodine alone is not as efficacious as 3 drug combo Followed by 4th generation HIV testing at 6 weeks and 4months to assess for seroconversion.

Coronary angiography demonstrating transient coronary spasm is the diagnostic hallmark of this condition and needs what treatment?

Prinzmetal, or variant, angina Nitrates and calcium channel blockers are the mainstays of treatment.

INITIAL -What is the initial investigation for OMyelitis in clinical setting and if infection is <2weeks (specific not sensitive)

Probe to bone clinical Cortical periosteal changes evident on XRAY IF <2W

When is progestin use contraindicated?

Progesterone receptor positive breast cancer.

What is the cause of ecythema gangrenosum? is it painful?

Pseudomonal bacteremia commonly in immunocompromized host ichhyyy treat dual pip tazo and gentamycin Painless but often has fever and systemic signs of bacteremia Violatious border

In someone with CF and respiratory infection what organisms should be covered and what abx?

Psudomonal: + amikacin + cefepime MRSA: Vanco

If pt is s/p cath 2 weeks ago when can he resume sexual activity?

Pt can resume 2 weeks after PCI or major cardiac event given that he had a successful revascularization and or is now low risk for a cardiac event!

Someone with jaundice RUQ pain fever and tachycardic should have what intervention and why?

Pt has cholangitis Management: 1. FLUID RESUCITATION 2. antibiotics 3. ERCP followed by 4. cholecystectomy ERCP done first since it is diagnostic and therapeutic for CBD obstruction. Cholecystectomy will NOT relieve the CBD obstruction but only PREVENTS future episodes!

In an elderly patient who is being treated for CHF s/p ablation for A-fib presents c/o fatigue and edema, labs show hyponatremia, bradycardia, elevated JVP 2cm, and LE sensation loss. What metabolic derangement is the culprit, how can this be prevented and monitored?

Pt has hypothyroidism due to amiodarone (which can also cause hyperthyroidism is the predisposed). Amiodarone prevents T4 conversion to T3 thus causing her fatigue, hyponatremia..etc via the Wolf chaicof effect. TSH should be monitored every 3 months while on Amiodarone and treated with high dose of Levothyroxine. HASHI HYPOTHYROID: SLOWED RELAXATION OF DTR BRADY CARDIA AND GOITER; increase risk of thyroid carcinoma and lymphoma; high TSH most sensitive for hashi!

What should be suspected in cirrhotic who fails to respond to a fluid bolus and has signs of renal failure ?

Pt likely has hepatorenal syndrome: is a common cause of acute renal failure in patients but is considered a diagnosis of exclusion. A combination of midodrine and octreotide along with albumin is the treatment of choice after the diagnosis is confirmed (after failing to respond to fluid bolus)

What. 3 PE findings would support evidence of cholestasis and most likely has a stone obstructing her common duct?

Pt with ascending cholangitis p/w 1. fever, 2. (RUQ) pain, 3. jaundice.

STATS: What does a funnel plot tell you? What is on the x and y axis?

Publication bias, effect of treatment on x axis plotted against a measure of study size or precision using SEM on y axis.

When is gamma glutamyl transferase and alk phos elevated?

Pylephlebitis, get imaging to see thrombus and treat for polymicrobial infection to prevent bowel ischemia, portal HTN, hepatic abcess.

Pt with a tender papule that is bluish, violaceous ulcer has what dermatitis condition? What disease process is this condition associated with?

Pyoderma gangrenous = rare neutrophilic dermatitis associated with IBD (chrones) and inflammatory arthritis.

During pregnancy which drug should be avoided if pt has active TB?

Pyrazinamide (possible teratogen) Otherwise pt on 3 drug regimine for 2 months followed by INH and RIF for 7 months

_____ is an anticholinesterase inhibitor used in the treatment of myasthenia gravis.

Pyridostigmine

STATS: Common tests for heterogeneity include the ____ statistic (no heterogeneity if P >0.05) and the ____ index (25%, 50%, and 75% are traditionally considered cutoffs for low, moderate, and high heterogeneity, respectively).

Q statistic (no heterogeneity if P >0.05) a I2 index (25%, 50%, and 75% are low, moderate, and high heterogeneity

In someone with Decompensated cirrhosis with ascities what finding on xray is indicative of hepatic hydrothorax? And how is it managed given that a theraputic/ diagnostic tap is preformed.

R SIDED PLEURA EFFUSION mgmt with furosemid spirno and NA restriction Tap is transudadive based on lights criteria

In someone with PE what Echo finding should you expect given the EKG shows a RBBB

R- atrial dialation leading to tricuspid insufficiency

What testing is needed in someone with myopia, and increased arm span prior to sport participation

R/o Sudden cardiac death prior to sports in those with tall stature and marfan s/o appearance high risk for aortic anyrism ECHO

Rib number and their associated muscle attachments that can be used for treatment are as follows: Rib 1: --- scalenes Rib 2: --- scalene Ribs 3-5: Pectoralis minor Ribs 6-9: Ribs 10-11: Rib 12: Quadratus lumborum

R1 Anterior and middle R2 Posterior R6-9 Serratus anterior R10-11 Latissimus dors

If diverticulitis pt has not improved after 3 days of treatment what is the next step?

REPEAT CT scan with contrast of the abdm. and pelvis. Colonoscopy is contraindicated until 8weeks of resolution to evaluate for CRC

When is Rho gam needed

RH+

Why is MRI/MRA needed in someone with ED what is common in those with ED?

RO brain anyrism

What is RRR? What does it determine?

RRR determines which 2 interventions is more beneficial to reduce the risk of disease based on incidence densities (person year data ef)

succynilcholine is used for

RSI (rapid seq. intubation) is a depolarizer metabolized by achESTERASE (enzyme blocked in OP poisoning)

Biliary colic presents with constant epigastric pain that can radiate to ________ and is associated with N, V, and _______. Abdm. exam is ______?

Radiates to back and R shoulder Associated with NV and diaphoresis Exam is benign without peritoneal signs or garding Initial: transabdominal ultrasound

DM diagnostic criteria? Besides akanthos niagricans what skin manifestation is related to insulin resistance?

Random fasting PLASMA glucose ≥200 with hyperglycemic symptoms HBA1C≥6.5 Fasting plasma glucose ≥126 2hr plasma glucose with OGT ≥200 Skin tags in image

In a case control study design which uses volunteers with newly diagnosed crones to fill out a questionnaire about past NSAID use can lead to what type of bias and classification

Recall bias and misclassification of exposure! misclassifications of outcome would not be an issue in this instance since the outcome is Chrone's.

When is a txf allowed in those with HB ≥8?

Recommended in pt with stable CAD or in those undergoing major surgery Other indications: Cancer increased bleed risk TCP severe ACS

What are Recurrence risk factors for Peripartum cardiomyopathy?

Recurrence risk LVEF <20% at diagnosis Persistent left ventricular systolic dysfunction

What would be the recurrence risk of turners in subsequent pregnancies if mom has a turners baby girl?

Recurrence risk is close to that of general population. Nondisjunciton leading to partial loss of X by random error = monosomy X sporadic event

Which nerves represent red flags for cauda equina

Red flag symptoms 1. New urinary retention/incontinence, fecal incontinence, 2. saddle anesthesia 3. severe lower back pain with bilateral sciatica, 4. motor deficits 2/2 MULTIPLE nerve root. EG weakness in both hip flexion and ankle dorsiflexion represents a motor deficit in more > 1 nerve root. - Hip flexion ( iliacus and psoaS)--> femoral nerve (nerve roots L2-L3). - Ankle dorsiflexion (T.A) --> peroneal (nerve roots L4-L5).

skin lesion that can be erythematous, papular, or nodular. Fever is present in <50% of cases. ____ follows within 1-2 weeks; it is the hallmark of CSD (cat scratch) and can be the presenting sign if the skin lesion goes unnoticed.

Regional lymphadenopathy

What should be suspected in a pt with prior CLL who presents s/p chemo with F, C, night sweats, 10lbs loss, splenomegally, anterior cervical chain LAD and axillary, inguinal adenopathy >3cm?

Reichter transformation occurs when CLL transforms into DLBCL (diffuse large B cell lymph.) and portends a poor prognosis.

What is 1-RR?

Relative risk reduction which is the proportion of risk redcution attributed to intervention or exposure RR = Risk of disease in exposed group / risk of disease in unexposed group Can also be calculated with risk in unexposed - exposed divided by risk in unexposed.

Describe the miller fischer test and what condition itis used for and why, what the presentation and treatment are.

Removal of 50ml LP tap to see improvement in gait, memory, verbal and visual memory in Normal pressure hydrocephalus. This test is used for PROGNOSTIC indicator of how well pt will respond to VP shunting to treat NPH! Diagnostic: MRI ventriculomegally and triad; improved gait with MF test NPH p/w triad of gait ∆, impaired congnition and urinary incontinence.

--- is typically associated with smoking, obesity, and hypertension. Patients usually have hematuria, abdominal mass, and/or flank pain. Paraneoplastic syndromes (eg, --- secretion) are common. Diagnosis requires a ---

Renal cell carcinoma erythropoietin PARANEOPLASTIC S/O DX: CT scan of the abdomen.

Baby who is not eating, and doesnt move is 3mo old likely has what if these are findings? ↑ WBC, ESR, CRP Blood culture Joint aspiration (synovial WBC count of >50,000/mm3) Effusion on ultrasound/MRI

SEPTIC ARTHITIS W STAPH A OR GAS treat asap Joint drainage & debridement IV antibiotics

What does demeclocycline treat 2nd line

SEVERE SIADH (1st line treatment is vaptans) : v2 receptor blockade which directly block adh action is 1st line for 2nd line: with demeclocycline acts at CT to block ADH signaling. MILD SIADH can be treated with free water restriction or salt admin

How can dka be distinguished from SGLT2 causing metabolic abnormality?

SGLT causes a EUGLYCEMIC KETOACIDOSIS ! <250 GLU

What drug causes ketoacidosis and which causes lactic acidosis

SGLT2 and MET respectively.

Which paraneoplastic syndromes results in salt wasting?

SIADH in small cell carcinoma also

When would CHest CT be helpful to investigate a pneumonia that seems reoccuring 3 weeks ago?

SMoker with pneumonia need to get CT to r/o cancer and to see if there is abscess or empyema

Gold standard diagnose TB quantitative results, takes 3-8 wks

SPUTUM CULTURE - but cannot always RO disease as sensitivity is low!

What should be suspected in someone with food getting stuck in her throat and swollen arthritic hands

SS due to progresive tissue fibrosis and vascular dysfunction CLinicaly features calcinosis cutis and PAHtn in limited crest ! systemic sclerosis has more ILDx all pt with scleroderma (limited or systemic) should be screened with PFTs!

What pain characteristically disappears/decreases upon sitting down, increases with spine extension, and decreases with flexion. Therapy can be conservative or can include ______Surgical decompression through a_____ option when other therapies fail. hint think pmnr

SS treated with: 1. conservative 2. a lumbar epidural block. 3. laminectomy

Phenelzine is a MOI that is contraindicated with whaat drug?

SSRI causes serotonin syndrome need a 2 week wash out or 5 weeks if on fluoxetine!

Why isnt CBT 1st line for depression?

SSRI first line so pt can engage in CBT or interpersonal psychotherapy.

What is initial eval for dysentery? and Mgmt?

STOOL STUDY: culture, Shiga toxin, fecal leukocytes Endoscopy for IBD MGMT oral rehydration empiric abx unless ehec suspected

repetitive collapse of the upper airway during sleep can be diagnosed definitavely via what questions?

STOP-BANG questionnaire: 1. Do you snore loudly? 2. Do you often feel tired, fatigued, or sleepy during the daytime? 3. Has anyone observed you stop breathing during sleep?4. Do you have high blood pressure? 5. BMI > 35 kg/m26. Age > 50 7. Neck circumference > 40cm 8. Gender - male

afib with RVR is treated with what?

SVT RVR goal is to slow rate to allow for proper ventricular filling goal of rate <111 use an AV nodal blocker Beta blocker metoprolol or non-dhp CCB use with caution in those with hypotension and decompensated HF as they are also NEGATIVE ionotropic

young man emigrated from Ghana has dysuria, urinary frequency, and terminal hematuria. Peripheral eosinophilia,, hg 10 (anemia) should be treated and diagnosed by ---?

Schistosomiasis is a parasitic blood fluke infection in sub-Saharan Africa. Dx: identification of eggs using urine sediment microscopy. tx: with praziquantel is usually curative.

When is MRI needed for LBP?

Sciatic lasting over 1 mo. cauda equina infection malignancy

When is an Ace inhibitor not contraindicated in acute renal failure?

Scleroderma renal crisis as long as renal function is monitored (mild bump in CRT is ok)

What is the stop bang questionair and what does it tell you?

Score less than 3 means no OSA as cause of snoring STop screen has HIGH NEGATIVE PPV for OSA Snoring is common and not always due to OSA Consider ETOH before bed consumed or smoking cessation counseling first

If a pt has evidence of barrett esophagus without dysplasia on bx when should next surveillance endoscopy be done? What if bx showed low grade dysplasia vs high grade?

Screening done in 3-5yrs, however evidence of dysplasia warrants endoscopy every 6-12months or endoscopic eradication. High grade dysplasia is indication for endoscopic erradication.

What pathogen causes erythematous plaques with loose, greasy-looking scales on the scalp ("dandruff"), central face, and ears. ITCHY What is the treatment? Hint sketchy spaghetti dandrif shampoooooooo

Seborrheic dermatitis caused by furfur (also found in TPN nutrition - sepsis) 1. topical antifungal agents (eg, ketoconazole, selenium sulfide)- dandrif shampoooooooo 2. keratolytic agents - (eg, salicylic acid), - coal tar, - topical glucocorticoids, - topical calcineurin inhibitors a. pimecrolimus, b. tacrolimus

septic shock CO PCWP and PVR?

Septic shock: Increased CO, Decreased PCWP, Decreased PVR

Postmenopausal osteoporosis Initial evaluation with what 3 components?

Serum chemistry panel Complete blood count Serum 25-hydroxyvitamin D

Fever, skin rash, polyarthralgia following exposure to a substance or virus is called what? What mediates the reaction and is it considered a hypersensitivity reaction? How is it distinguished from septic arthritis? How long after exposure do symptoms manifest and what is the main treatment?

Serum sickness like reaction which is a T3HSR mediated by immune complex deposition, is caused by recent HEP B infection, B lactam and sulfa drugs Septic arthritis is UL while SS like rxn is BL Occurs >1 week after, treat with steroid or plasmapheresis in severe

How to monitor treatment of graves while on antithyroid drugs? what state causes increase in TBG

Serum total T3 FT4. State causing TBG increase: liver dx, pregnancy, aspirin and OCP use

When can you not use colchicine?

Sever renal or liver dx Combo with cp450 inhibitor Give lower doses in elderly

murmur of severe aortic stenosis

Severe AS: - soft, single second heart sound (S2) vs a2 and p2 -delayed and diminished carotid pulse ("parvus et tardus") - Loud and late-peaking systolic murmur

Define severe CDI vs Fulminant CDI?

Severe CDI: white blood cell count ≥15,000 cells/mm3 and/or serum creatinine >1.5 mg/dL (treated same as those with initial CDI) Fulminant: hypotension, shock, ileus, megacolon is present

How does one treat Otitis externa ("swimmer's ear") that is blocked? and malignant OE

Severe cases with edema that blocks antibiotic penetration w/ wick. - Systemic antibiotics are reserved for invasive infection or for immunosuppressed patients.

What is another name for dimercaprol? Why is it used hint eat my d and BAL brit anti lew (severely)

Severe lead toxicity ≥70 μg/dL Dimercaprol (British Anti-Lewisite) plus calcium disodium edetate (EDTA)

What does parkingsons and hiv have in common?

Severe seborrheic dermatitis TREAT WITH TOPICAL ANTIFUNGALS, GLUCOCORT., CALCINEURIN INH.

name the short acting insulin's

Short-acting insulin: Analogs: Fast onset / offset. Peak coincides with food 1. Lispro 2. Aspart 3. Glulisine Regular Slow onset & offset

What should be suspected in prego who p/w retraction of the fetal head into the perineum after the head delivers? aka TURTLE SIGN what is the initial management?

Shoulder dystocia: the failure of the usual OB maneuvers to deliver the fetal anterior shoulder, is an obstetric emergency. Initial management includes the McRoberts maneuver and suprapubic pressure.

Common manifestations of superior pulmonary sulcus tumor

Shoulder pain + Weight loss Horner syndrome (invasion of paravertebral sympathetic chain/stellate ganglion)Ipsilateral ptosis, miosis, enophthalmos & anhidrosis Neurologic symptoms in the arm (invasion of C8-T2 nerves) Weakness/atrophy of intrinsic hand muscles - Pain/paresthesia of 4th/5th digits & medial arm/forearm Supraclavicular lymphadenopathy

What is the treatment of RV MI?

Similar to STEMI management

hepatorenal syndrome (dx of exlusion!) can cause acute renal failure in pt with cirrhosis but how can it be confirmed that it is not just 2/2 intravascular volume depletion?

Since hepatorenal is a diagnosis of exlusionm a fluid bolus is needed to confirm that the renal failure is not 2/2 volume depletion and thus should not improve with bolus

Acute inferior wall MI causes what arrythymia and why?

Sinus bradycardia due to increased vagal tone triggered by SA node (RCA supplies SA node which causes increases vagal tone) and RV wall ischemia.

The presence of what arrhythmias is associated with an increased risk of developing torsades de pointes in patients with drug-induced acquired long QT syndrome (LQTS).

Sinus node dysfunction Atrioventricular block (2nd or 3rd degree) (sinus bradycardia or pauses)

What is the most common ECG abnormality in PE patient?

Sinus tachycardia is the most common ECG abnormality seen in patients with pulmonary embolism

How does one typically differentiate dermatitis herpetiformis vs autoimmune pemphigoid conditions?

Skin bx Bullous- tense itching bull Pemphigus vul: flacid sloughing of skin + oral involvement DH - itchy crusing ciliac dx Linear IGA bullous dermatosis - grouped lesion - typically linear or annular paterns

diagnose lead poisoning is extremely important since kids are at high risk for lead poisoning - smaller bodies - absorbed at a faster rate - crawl and walk = exposure via ingesting / inhaling dust - ____(2 main complications)___= both commonly seen.

Slow development of talking + permanent intellectual disability

§ Treatment for arsenic poisoning?

Sodium 2,3-dimercapto-1-propane sulfate (DMPS)

Sodium bicarbonate ___ serum pH and extracellular sodium. If pt is refractory to sodium bicarbonate may respond to adjuvant ____.

Sodium bicarbonate inc. pH and extracellular sodium thus decreases binding of TCA to rapid NA channel on heart!! if refractory give magnesium or lidocaine.

______ occurs in association with pancreatitis, may be asymptomatic or present with variceal bleeding due to isolated gastric varices, the disease's hallmark.

Splenic vein thrombosis (SVT): Varices develop due to redirection of blood flow to the collateral gastroepiploic system and short gastric veins.

If a pt with suspected TB undergoes sputum acid fast bacilli smear and all 3 are negative can you definitively RO active dx?

Sputum acid-fast bacilli smear is a common initial diagnostic test for active pulmonary tuberculosis. Positive testing allows empiric treatment. - mycobacterial culture or nucleic acid amplification is required for confirmation since low sensitivity and a lack of distinction between tuberculous and nontuberculous mycobacterium, cannot be used to definitively rule in/out the diagnosis.

TB SPUTUM SAMPLING PROTOCOL how many samples are needed for confirmation and what tests need to be run?

Sputum sampling (least invasive and cheapest) for microbial confirmation. 3 single sputum samples (spontaneous or induced) are submitted in 8- to 24-hour intervals with at least 1 early-morning sample. Sputum should be sent for acid-fast bacillus smear, mycobacterial culture, and nucleic acid amplification testing.

What cancer causes hypercalcemia of malignancy and ↓ PTH ↑ PTHrP

Squamous cell Renal & bladder Breast & ovarian

Stage A in the development of heart failure

Stage A High risk for heart failure, but without structural heart disease or symptoms of heart failure patients with diabetes or hypertension, patients exposed to cardiotoxic drugs

After confirming hit 2 with the serotonin release assy what therapy should be done besides stoping heparin?

Start direct thrombin inhibitor ARGATROBAN or fondaparinux

What is Mirrizzi Syndrome?

Stone in the cystic duct resulting inflammation and obstruction of the common duct (Mirizzi syndrome). but usually stone in CBD--> dilated intrahepatic ducts rather than stone in cystic duct.

The most common cause of acute bacterial sinusitis and otitis media in children is ____.

Streptococcus pneumoniae.

In someone with high risk stress testing features what would they benefit from? What if they are not high risk but have persistent symptoms

Stress test (identification) PCA to assess for stent placement or coronary artery bypass graft After months of optimal medical mgmt and anginal symptoms are still present they need to undergo coronary angio

Hydrofluoric acid burn treatment?

Strongly acidic compound tx irrigation with water, calcium gluconate gel may be applied locally to the affected areas. The most definitive treatment does include intra-arterial calcium injection; NOT the most appropriate initial treatment.

Stage C in HF

Structural heart disease with prior or current symptoms of heart failure

Stage B in the development of heart failure

Structural heart disease, but without signs or symptoms of heart failure (patients with prior myocardial infarction or valvular heart disease with left ventricular enlargement or low ejection fraction)

_____ is a common cause of shoulder pain - often poorly localized/ near acromion. - worst with overhead activities PE: benign, + Neer's impingement test or Hawkin's test

Subacromial bursitis Neer's impingement test - patient in the sitting position. - arm is forward-flexed while the scapula is stabilized. - Pain near the acromion = positive test. Hawkin's test is also sensitive 4 subacromial. - elicited by flexing the shoulder and internally rotating the arm. - Pain near the acromion = positive test.

What should be given to women with Eisenmengers after menses?

Subdermal Progestin implant as pregnancy is contraindicated and estrogen OCP increase Thromboembolism risk.

How is Primary adrenal insufficiency treated and what is a possible cause of it in the homeless population?

TB!!!!! treat with: corticosteroid replacement and aggressive fluid replacement with 5% or 10% intravenous dextrose and saline solutions and treatment of hyperkalemia is mandatory in adrenal failure.

homless shelter, fever, weight loss b symptoms with right sided lobar middle lobe calcification, no cough, sob living in Cali if cxr also shows hilar lymphadenopathy?

TB, initial presenting symptom does not have to be cough! ghon complex is the right middle lobe calcification

Initiation of what antidepressants requires a baseline ECG?

TCA intitation and monitoring due to risk of arrhythmia.

how do TCA's cause hypotension?

TCAs can also decrease calcium influx into the myocardium and increase peripheral vasodilation, causing hypotension.

how do TCA cause arrhythmia?

TCAs inhibit fast sodium channels in the His-Purkinje tissue and the myocardium to decrease conduction speed, increase phase 0 depolarization, and prolong the refractory period.

How is tardive treated, akathsesia, vs parkinsons?

TD: Valbenazine and deutertabenzine AKATH: Restlessness give Beta blocker propranolol, Diphenhydramine, benztropine PK: amantadine or benztropine

People with HIT are prone to developing what?

THROMBOSIS and can go on to develop acute heparin-induced thrombocytopenia and thrombosis syndrome (HITTS)

older patients w/ new onset lethargy, confusion, and depression in setting of normal head ct, cbc, cmp AND no thyromegally on exam warrents what test?

THYROID TEST NOT MRI! Apathetic thyrotoxicosis is an atypical presentation of hyperthyroidism in OLD

Which Patients at increased risk for colon cancer?

THose with relative <60 with CC or If ≥2 first degree relatives with ADVANCED adematous polyps colon cancer Screen needs to begin at 40 or 10 years prior (whichever is earlier); repeat q3-5 YRS

Difference between tinea manuum and ICD?

TM presents on both dorsal and vental hands (also feets usually) with itching and is UL

Specificity =

TN/(TN + FP)

What tests must be done prior to starting biologics?

TNF inhibitors (infliximab ertanecept) after pt passes Interferon gamma release assay is negative or tb skin test (Laten tb RO test)

What are the 4 anatomic features in the most common cyanotic congenital heart defect?

TOF - RV outflow obstruction - overriding aorta - RV hypertrophy - VSD

Sensitivity =

TP/(TP + FN)

SENSITIVITY FORMULA

TP/(TP+FN)

Treatment of hepatic encephalopathy

TREAT UNDERLYING CAUSE (EG volume status, electolytes hypokalemia) Lactulose titrate up, in 48hrs if still encephalopathic --> rifamixin

________ cause elevated TSH and thyroid hormone levels. and biologically inactive alpha subunit

TSH-secreting pituitary adenomas often pt has a diffuse goiter but no extrathyroidal manifestation of graves - graves will not have elevated alpha subunit or elevation of pituatary hormones along with sex hormone binding globulin will all be normal in graves (as hepatic tissue is resistant to the increased thyroid hormone levels)

What acquired autoantibody or hereditary disease is unmasked with pregnancy and p/w 1. petechial rash 2. neurological change (coma, AMS) 3. low platelets 4. low Hg 5. elevated wbc Hint total bili 3.8 along with CRT 1.5

TTP Pregnancy associated with lowered ADAM13 thus unmasks decreased adamts13 activity in TTP--> there is disseminated thrombosis in microvasculature (eg renal artery, skin petechia). sx: renal failure, neuro, fever, abdominal pain, petechia Lab: MAHA +schistocytes, thrombocytopenia tx: plasma exchange

TTP Pentad? (= A disorder of ↑ platelet aggregation leading to mucocutaneous bleeding [petechiae or purpura] and hemolysis)

TTP classically presents with the pentad of altered mental status, fever, thrombocytopenia, renal failure, and hemolytic anemia.

Clinical features of someone with labs showing: Hemolytic anemia (↑ LDH, ↓ haptoglobin, schistocytes) Thrombocytopenia

TTP: FATRN Renal failure Neurologic manifestations (coma, confusion) Fever Anemia Petechial rash Laboratory abnormalities Hemolytic anemia (↑ LDH, ↓ haptoglobin, schistocytes) Thrombocytopenia

What are the 5 WHO classifications of PAH

TYPE 1: PAH TYPE 2: due to Left sided heart disease TYPE 3: chronic lung disease COPD, ILD TYPE 4: chronic Thromboembolism TYPE 5: other causes (e.g Sarcoidosis)

Contraindicated abx during pregnancy for UTI treatment

Tetracyclines Fluoroquinolones Trimethoprim-sulfamethoxazole

pediatric water temp?

The American Academy of Pediatrics recommends setting home water heaters to <48 C (118 F) to minimize the risk of scald injuries should the child have unsupervised access to water.

STATS: What is the hawthorn effect?

The Hawthorne effect occurs when participants modify their behavior due to their awareness of the fact that they are being studied.

what testing is highly specific for h pylori?

The antral mucosal biopsy is highly sensitive and specific for h- pylori and no-repeat testing needed to confirm.

What would the serum thyroglobulin TBG (globulin) levels and 24 hour RI uptake be in someone taking exogenous thyroid hormone?

Thyroglobulin binding which is released by the thyroid follicular cells will be LOW as will the RI scan which measures the follicular activity. Exogenous thyrotoxicosis = Exogenous thyroid hormone decreases follicular level functioning - similar to struma overia (ectopic production)

Treatment of choice for myxedema coma is

Thyroxine and emperic adrenal replacement : 5 to 8 μg/kg of thyroxine (T4) and then 50 to 100 μg intravenously (IV) daily. You MUST also give adrenal replacement empirically since peripheral conversion of T4 to (T3) is impaired, (T3 is more arrhythmogenic).

Stats: when would a survival analysis where the even of interest is death be useful?

Time to event data which is collected when the elapsed time before an event occurs is of significant interest and survival analysis is best for assessing mortality benefit in prospective studies and clinical trials.

When would a diagnostic peritoneal lavage be needed?

To detect hemoperitoneum in a pt who has blunt abdominal trauma

Hip effusion, usually in children age 3-8, often following a viral illness?

Transient synovitis can present as a hip effusion, usually in children age 3-8, often following a viral illness;

Treatment TSH-secreting pituitary adenoma present w thyrotoxicosis ?

Treatment Somatostatin analogs Transsphenoidal surgery

Choriocarcinomas and trophoblastic tumors treatment:

Treatment options include chemotherapy (typically methotrexate) and hysterectomy. After treatment, β-hCG levels are used as a marker for disease remission or progression.

What meds are needed for new onset a-flutter

Treatment with warfarin, an oral factor Xa inhibitor (eg, apixaban, rivaroxaban), or an oral direct thrombin inhibitor (eg, dabigatran) is appropriate for at least 3 weeks prior to ablation/cardioversion.

To avoid venous air embolism, place the patient in the_____ position to keep venous pressure _____ . Air entering the venous circulation collects in the right heart which may cause significant strain.

Trendelenburg positioning to keep venous pp higher than atmospheric pressure If the pulmonary artery pressure elevates, cardiovascular collapse may occur from significant lack of venous return to the left heart.

Which triad is suggestive of true hypoglycemia.

Triad of low blood sugar <60 in DM, symptoms of hypoglycemia, symptoms resolve with glucose = Whipple's triad of hypoglycemia

_____ is effective against S aureus. However, it is avoided during breastfeeding as it is excreted in breast milk and is associated with a risk of neonatal kernicterus.

Trimethoprim-sulfamethoxazole

Norcardia p/w what symptoms and requires what treatment?

Trimethoprim-sulfamethoxazole is often used to treat nocardiosis, which typically causes pulmonary or central nervous system (eg, brain abscess) disease.

Clinical features of Mutation (inherited or de novo) in TSC1 or TSC2 gene: Autosomal dominant?

Tuberous sclerosis tsc Dermatologic 1. Ash-leaf spots 2. Angiofibromas of the malar region 3. Shagreen patches Neurologic 1. CNS lesions (eg, subependymal tumors) 2. Epilepsy (eg, infantile spasms) 3. Intellectual disability 4. Autism & behavioral disorders (eg, hyperactivity) Cardiovascular: rhabdomyomas: no screen needed seen on ultrasound Renal: angiomyolipomas

What is etiology of congenital lymph-edema?

Turners along with 1. broad chest wide nipples and 2. high arch palate 3. low hairline these finding at birth warrant urgent diagnostic confirmation with karyotype.

Which RTA is characterized by failure to resorb bicarb?

Type 2

What are the 2 types of nec fasc

Type 2 (most common) - affects healthy people - predisposing surgery, laceration, trauma - group A Streptococcus (Streptococcus pyogenes) Type 1 (less common) - dec perfusion( DM. vs PVD) - polymicrobial infection 1. Staphylococcus aureus, 2. Bacteroides fragilis, (crepitis) 3. Escherichia coli, 4. group A Streptococcus, and Prevotella species.

STATS: If a study is repeated with less patients in the second group and assuming population for patient recruitment is similar what bias or error is the second study at more risk of?

Type 2 error: detecting a difference when there is none AKA accepting null when it is true Coorelated to power of study P= 1- beta and beta increases as power decreases beta is failure to reject the null hypothesis B and n= cohort are inversely proportional!

NAGMA 8-12 HARD -ASS VS USEDCAR - (hint hes a hardass and NORMALLY takes 8-12 hits )

U - URETERAL DIVERSION S - SALINE INFUSION E - EXOGENOUS ACID D - DIARRHEA C - CARBONIC ANHYDRASE INHIBITOR A - ADRENAL INSUF. R - RTA

USED CAR - NORMAL ANION GAP METABOLIC ACIDOSIS MNEUMONIC VS HARDASS (hint hes a hardass and NORMALLY takes 8-12 hits )

U - URETERAL DIVERSION S - SALINE INFUSION E - EXOGENOUS ACID D - DIARRHEA C - CARBONIC ANHYDRASE INHIBITOR A - ADRENAL INSUF. R - RTA

What is the difference btw herpangina and herpes gingivostomatitis?

Ulcers are anterior oral mucosa in gingivostomatitis while herpangina is posterior oropharynx

Emergency contraception should be offered to patients within 5 days after unprotected intercourse. _____ is the most effective oral emergency contraceptive. The copper intrauterine device is more effective than emergency oral contraceptives, but it is contraindicated in women with acute cervicitis.

Ulipristal

STATS: What is the difference btw PPV and NPV that is confusing?

Unlike PPV which is the first column divided by the total positive tests the Neg PV is the SECOND COLUMN over total! thus it is TN over total! Also PV is based on PREVALANCE UNLIKE SENSITIVITY SPECIFICITY! PPV = probabilty of having a dx given a + test result !!!!!!!!!!!!! a/(a+b) NPV = d/(c+d) a= true positive d= true negative b= false pos c= false negative

What is the risk of breast cancer vs endometrial cancer in those taking menopausal HT? combo vs estrogen alone? HINT E---E (2 BOOBS INCREASE RISK WITH 2 DRUG)

Unopposed estrogen is associated with an increased risk of endometrial cancer, (E--> E) risk of breast cancer is increased with combination therapy (although it may be lower with unopposed estrogen). (2 BOOBS INCREASE RISK WITH 2 DRUG)

How is a urethral diverticulum diagnosis confirmed? and treated?

Urethral diverticulum initial evaluation= 1. urinalysis and 2. urine culture in patients with dysuria or hematuria. Diagnosis can be confirmed pelvic MRI or transvaginal ultrasound. Treatment: 1. manual decompression, 2. needle aspiration, or 3. surgical repair.

PE: tender anterior wall vaginal mass that expresses bloody, purulent fluid on manipulation of the urethra and pt c/o urine dribbling after voiding.

Urethral diverticulum presents with postvoid dribbling, dysuria, dyspareunia, and an anterior vaginal wall mass on pelvic examination.

_____ can slow the progression of primary biliary cholangitis (PBC) and improve overall and transplantation-free survival. curative treatment is ____.

Ursodeoxycholic acid liver transplantation. PBC can recur following transplant; however, if it does recur, it tends to have slower progression. AVOID CST: they are not helpful despite the autoimmune nature of PBC.

WHAT should be suspected in postmenopausal patients with new-onset pelvic pressure or pain, uterine mass, and ascites (eg, free fluid in the posterior cul-de-sac)?

Uterine sarcoma The most common site of metastasis is the lungs (eg, pleural effusion). Risk factors for uterine sarcoma include tamoxifen use and pelvic radiation.

What is the major cause of hypoxemia in pt with COPD that is worsened during exacerbation? What should be done to ameliorate it?

V/Q mismatch due to low V/Q lung regions is the major cause of hypoxemia and worsened during exacerbation. Supplemental 02 helps by inc. o2 in low V/Q areas.

What bipolar med causes a 20 fold increase in neural tube defects and should be DX before pregnancy (particularly DC during first trimester?)

VALPROATE the anticonvulsant used for epilepsy and BPD it is a known teratogen and causes NTD women with BPD should take lithium insted as the ebstien anomaly is rare Do not confuse with Valproate during pregnancy in an epileptic this is different managment ?

hypotension, distant heart sounds, and distended neck veins (Beck's triad).

VENTRICULAR FREE WALL RUPTURE

What gm (-) found in marine setting causes 1. hemorrhagic bullae and 2. rapid progressive cellulitis that progresses 3. rapidly to septicemia in those with liver dx?

VIBRIO VULNIFICUS causes leukocytosism left shift and RENAL insufficiency in those with hepatitis or cirrhosis!

What anti-epileptic can cause - coma, - pinpoint pupils, - toxicity --> encephalopathy and cerebral edema

VPA !! Coma with small pupils may be seen, and this can mimic opioid poisoning.

How many HepB vaccine is given if infant exposed? When will we know for sure the child did not have vertical transmission?

Vaccine 3 series HepB at 0, 2,6 months followed by serology check at 9mo if child does not have HBsAg then child is uninfected.

3 indicators of ruptured membranes

Vaginal pooling: A sterile speculum is used to examine the vaginal vault. Pooling appears as a collection of fluid in the vagina, which can be enhanced by having the patient Valsalva to promote fluid escaping from the cervix.Nitrazine test: Vaginal secretions tend to be acidic compared to amniotic fluid, which is alkaline. If amniotic fluid is present, the nitrazine paper will turn blue due to the alkaline environment, indicating a positive test.Fern test: Estrogens in the amniotic fluid cause crystallization of amniotic salts, which appear as blades of a fern on microscopic evaluation, indicating a positive test.

Cerebral edema from what med causes hypernatremia, metabolic acidosis, hypocalcemia, elevated serum ammonia, and mild liver aminotransferase elevation?

Valproic acid intoxication

What med is used for Juvenile myoclonic epilepsy (JME) and what should be monitored while taking this medication?

Valproic acid: treats juvenile myoclonic epilepsy are at risk for recurrent seizures and require lifelong antiepileptic drugs (AEDs). A baseline AED level is obtained 1st - dose gradually titrated until seizures are suppressed or the patient is at risk for toxicity. - monitor thrombocytopenia

What smoking cessation agent should be avoided in those with psych patients? Is this medication superior to Buepenorphine?

Varenecycline should be avoided in patient w/ current UNSTABLE NEUROPsych history or active SI Yes, varenecycline is better than placebo and Beup. (lowers seizure threshold)

Genu varum is expected to be resolved by what age in infant? Genu valgum begins at 4 and resolves at what age in children?

Varum 18mo (rum 18 months rum) Valgum 7yr (gum stuck in gut 7yr)

What is the most dangerous complications of central line placement?

Venous air embolism

The presence of a loud, churning, machine-like murmur, also known as a "mill-wheel" murmur is characteristically auscultated over the precordium.

Venous air embolism symptoms: 1. dyspnea during the procedure. 2. Subsequent tachycardia, 3. tachypnea, 4. cyanosis, and 5. altered levels of consciousness may also ensue.

combo menopausal hormonal therapy causes what detrimental effects at any age vs if they are greater than 60?

Venous thromboembolism Breast cancer Coronary heart disease (age >60) Stroke (irrespective of age and SBP control) gallbladder disease detrimental effects are age specific (thus can be used in 50-59) MHT can be safely used for a short period (3-5 years) in younger, low-risk women (ie, nonsmokers; no history of breast cancer, venous thromboembolism, or CHD).

Methylene blue is instilled into the bladder to diagnose what?

Vesicovaginal fistulas = painless, continuous incontinence of urine from the vagina. + if a tampon placed in the vagina becomes blue after the dye is instilled in the bladder.

What is the legal term for when an attending doc gets the blame for a residents mistake?

Vicarious liability e.g attending physician being held liable for the actions of the resident physician.

Vit D deficit

Vitamin D deficiency is demonstrated by symptoms of rickets in children and osteomalacia in adults.

____ (latin term) refers to the doctrine stating that a person who knowingly places him- or herself at risk for danger cannot sue for any resulting injuries that may occur and does not apply to the above case.

Volenti non fit injuria

Treatment of HE other than using lactulose to dec ammonia?

Volume and electrolyte repletion

___ ratio not exceeding 0.01 has almost 100% negative predictive value for meningitis on lumbar puncutre

WBC/RBC Traumatic lumbar puncture (LP) results from accidental damage of a blood vessel during the procedure. A high RBC count (>6k) ABSENT xanthochromia is characteristic! XANTHOCHROMIA if present points to SAH

What must be done for someone who has acute chest pain and what factors do you need to rule out initially?

Want to rule out ACS acute coronary syndrome which can be ruled out by the following: 1. EKG negative rules out STEMI 2. serial trops/cardiac enzymes rule out NSTEMI 3. absence of exertional chest pain that worsens over time (crescendo angina) and EKG symptoms that sugest ischemia (t wave inVersion) or dyspnea and diaphroesis rule out Unstable angina

The typical presentation of pt with is a necrotizing vasculitis = affects the lungs, upper respiratory system, and kidneys which leads to nephritic syndrome.

Wegener's granulomatosis affecting 1. upper respiratory system 2. kidneys --> nephritic syndrome. Treatment is with cyclophosphamide.

how can you tell if patient has a TB infection that is healed?

When both the Ghon focus and the lymph nodes are calcified, this is known as a Ranke complex and is usually a sign of healed infection. also may not even have a ghon complex early on

STATS: What is lead time bias?

When compared testing for a disease diagnosis the disease earlier but does not actually increase survival But rather diagnosis sooner giving the false interpretation that the candidate lived longer.

when is detorsion of a testicle used in testicular torsion?

When emergent surgical consult is unavailable For pt Sudden onset of symptoms with lack of flow on color Doppler leads to the diagnosis of testicular torsion.

STATS: What is are the selection biases?

When group studied may by inappropriate or poor f/u

Newborn with hydrocephalus and diffuse intracranial calcifications likely has what congenital infection and should be treated with what?

When infection is suspected prenatally, diagnosis is made via maternal serology or amniocentesis. infection in newborn: neonatal serology. Treatment: antiparasitic therapy (ie, pyrimethamine and sulfadiazine) + supplemental folate for a year.

STATS: What is the ecologic fallacy?

When population level imformation is used to make individual level conclusions Ecologic correlational study is based on population not individual they can however speak to the concordance of changes in variables over a population

What PE exam finding (name) may be associated with underlying pancreatic malignancy. p/w A high fever and chills suggest a coexisting cholangitis.

When the abdomen is examined, the gallbladder may be palpable (Courvoisier sign). p/w A high fever and chills suggest coexisting cholangitis.

Which BSA are indicated to treat nec fasc?

When the etiology of the necrotizing fasciitis is unknown, broad-spectrum therapy should be started. This includes: Piperacillin/tazobactam or a carbapenem (eg, impinem, meropenem) will cover Group A Streptococcus and anaerobes Vancomycin will cover Staphylococcus aureus, including methicillin-resistant isolates Clindamycin is added to inhibit toxin formation by streptococci/staphylococci Once culture information is available post-op, antibiotic therapy should be narrowed based on identified pathogens.

SUBCLINICAL HYPERTHYROID with a TSH of 0.5 an indication to treat? Would you need to have the TPO ab test if you already had diffuse uptake on RI?

When to treat? Prolonged TSH suppression or if TSH <0.1 Ft3t4 can be normal but TSH suppressed <(0.5-5) for a long time = treat! Pt may be assymptomatic demonstrate increased uptake on RI scan and will need meds ASAP Low tsh + Risk factors: age >65, heart dx, osteoporosis, nodular thyroid dx NO, diffuse uptake already diagnosed graves.

A Whipple procedure treats what?

Whipple procedure aka pancreaticoduodenectomy treats malignancy of the head of the pancreas.

Pt h/o IV drug use. PE = multiple, discrete ITCHY violaceous papules and plaqueswhitish, lacy pattern, referred to as Wickham striae, on flexor surface (WRIST). Mucosal lesions on the tongue and buccal mucosa HAS WHAT CONDITION?

Wickham striae whitish, lacy pattern often on the lesion surfaces. Lichen planus LP = pruritic, polygonal-shaped violaceous papules/ plaques on flexural surfaces of the extremities (wrists), buccal mucosa, or external genitalia. - confirm with skin bx.

Pt s/p foosh with Xray that are negative. What type of scaphoid fracture should be suspected? And how is it further managed?

X-rays can initially be negative for nondisplaced scaphoid fractures. Nondisplaced fractures should be treated with a short arm thumb spica cast after further imaging with either: - MRI (or CT scan) of the wrist - Repeat radiographs in 7-14 days Radioscintigraphy bone scan in 3-5 days

Hemophilia B has missing factor _____.

XI also PTT prolonged Hemophilia B, aka as Christmas disease is due to factor IX deficiency. Christmas has 9 letters in it.

What are the indications to get imaging in LBP? What are the various modalities for imaging and which is better for LBP?

Xray indication +/- ESR - OP, compression fx - malignancy -ankylosing spondylitis ( insidious onset with night pain, improves with movement) MRI - sensory/motor deficits -cauda equina syndrome (urine retention, saddle anesthesia) -abscess infection Radionuclide scan / ct: MRI contraindication

STATS: If the CI is greater than 1 and so is the SMR is that value significant?

YES! Eg SMR 2.22 WITH 95% CI of (2,3) (higher incidence of death among group than expected is significant!) SMR >1 without CI crossing null but not for SMR 1.22 with a CI of (.99, 1.66) - this CI crosses 1 thus SMR is not significant bc contains null value

can erysipelas be on the face? what causes it?

Yes GAS causes it usually is a sharply demarcated over cheeks with abrupt onset of systemic fever chills etc

Pt with passing out randomly without preceding symptoms need hospital admission if its the first episode?

Yes admit to hospital for INPATIENT MONITORING AND WORKUP: TELE and get ECHO, treatment will be with either amiodarone, cath ablation or ICD placement if cardiac cause DO NOT DC HOME WITH ICD NEEDS HOSPITAL W/U

Do tb drugs cross the placenta?

Yes but do not cause fetal toxicity (except pyr) However close monthly monitoring and evaluation for hepatitis is needed along with pyridoxine during pregnancy

If you mess up and drug the patient by accident but no harm was done should you still tell the patient?

Yes disclosure should be done in timely manner, IN PERSON, and apologize.

Should RADT be used prior to giving abx in someone with fever, pharyngal exudate, cough, tender anterior cervical adenopathy?

Yes if 3 of centor criteria is filled you need to do strep rapid antigen detection test RADT to confirm microbe dx.

Can absence of resp. drive indicate brain death?

Yes if it is for >8 min and Paco2 >60; and arterial ph<7.28

Can smoking cessasion (in someone who smoked >/1PPy) have their CHD risk return back to normal?

Yes in 2-3 years the risk returns back to normal population if pt was a smoker of at least 1ppy

if a day care worker is exposed to meningococcal meningitis and has hx of IBS and on OCP should she get prophylaxis, if so with what?

Yes prophylaxis is typically with rifampin bid x2d BUT IF ON OCP then give cipro 1x dose as rifampin decreases OCP effect

Would transferring a patients care be deemed necessary if physician is involved with the parent? How about transferring care to another doctor in the practice?

Yes transfer of care to another doctor in ANOTHER PRACTICE is deemed necessary by ethical standards as it would have effect on doctor's judgement. And should be done for any key 3rd party involvement including parents, gaurdians, spouses or partners) Aka transfer care if you are with the patients 3rd party who has control over them somehow!

Can relative risk be calculated for follow up studies using only exposed groups to different levels of intervention?

Yes! Relative risk is a measure of stregth of association btw exposure and disease in follow up studies. Typically it is the risk of exposed group divided by the unexposed BUT CAN BE redefined to designate diff levels of intervention or risk factors

Is additional testing needed for CDI that is recurrent?

Yes! cdi should be confirmed with stool study then treated with oral vanco given in a prolonged pulsed and tapered regimen (2-8 weeks) Or give 10day of fidaxomicin if inital treated with vanco.

Can a publication bias occur in meta analysis?

Yes. Publication bias occurs when studies with positive results are published but studies with negative/null results are not. biased sample of all relevant studies in the literature distorts the true mean effect computed by the meta-analysis and leads to publication bias.

What is the criteria for sever AS?

You must have sever AS 1. Aortic velocity ≥4m or 2. Transvalvular pp ≥40 3. valve area ≤1cm (not required)

How is C diff associated diahrrea diagnosed? CDAD?

Characteristic symptoms - Watery diarrhea (≥3 loose stools in 24 hours) with or without lower abdominal pain, low-grade fever, and leukocytosis AND Positive stool testing for toxigenic C difficile - Stool testing confirms the presence of toxigenic strain but cannot distinguish an asymptomatic carrier.

Which virus causes high fever, polyarticular joint pain that is bilateral, symmetric and in hands, wrists and ankle joints?

Chikungunya fever is mosquito borne virus affecting tropical area of south america, ashia, africa (near equator) Vector aedesmosqueto (dengu, zika and ckgna)

§When would you need a skeletal survey?

Child abuse, Multiple myeloma

Diptheria manifestations?

Children <age 15 toxoid vaccination decreases risk Pharyngitis pseudomembrane

What neoplastic disease p/w abdominal pain, pelvic pressure, irregular vaginal bleed after pregnancy and can present and affects women >40 YO and can metastasize to lung and vagina? Initial eval and treatment include what?

Choriocarcinoma, most aggressive gestational trophoblastic neoplasia. § w/ hydatiform mole. initial evaluation 1. quantitative β-hCG; 2. thyroid, 3. renal, and 4. hepatic function panels; 5. pelvic ultrasound; and 6. chest x-ray.

Recurrence Risk of which disorders increase with maternal age?

Chromosomal nondisjunctions! except for turners thus trisomy 21 nondisjunction and klinefelters syndrome xxy increases w/ AMA

CLL prognosis is worse with presence of what? What cell is specific to this type?

Chronic lymphocytic leukemia is a monoclonal B-cell leukemia that can cause lymphadenopathy, organomegaly, and anemia/thrombocytopenia, all of which are associated with a worse prognosis. Smudge cells

What class of hypovolemic shock pw HR >120 and confusion?

Class III is associated with 1,500-2,000 ml of blood loss, HR > 120, and confusion.

Atrial flutter has what characteristic ECG pattern and in what leads?

Classic biphasic "sawtooth" flutter waves in the inferior leads,

Type 2 hit clinical signs and gold standard?

Clinical signs after >5days with any of 4 symptoms 1. pc reduciton >50% from baseline 2. A or V thrombosis 3. Necrotic skin lesion at heparin injection site 4. anaphalactoid rx after heparin Diagnostic eval: SEROTONIN RELEASE ASSAY (serotonin is a marker of plt activation and elisa detects hep-PF4 ab) Treat: d/c hep and give leprudin (DONOT TRANSFUSE PLT OR give warf until plt >150)

What puts you at high risk for CAD events?

Clinically 1. poor exercise capacity 2. exercise induced angina 3. fall in SBP from baseline 4. chronotropic incompetence ECG variable >1mm st depression st depression at low workload st elevation in leads without q waves v- arrythmia HIGH risk pt will need to have percutaneous coronary angio for revascularization due to their atherosclerosis

How do you diagnose a pt c/o unilateral dermatomal vesicular skin eruption? Are contact and airborne precautions needed?

Clinically, VZV testing is not necessary for diagnosis unless it is atypical. Thus start the pt with local zoster lesion <3days old with oral valacyclovir to reduce risk transmission and posterherpetic neuralgia + analgesia for neuritis Contact and airborne precautions used with disseminated disease; otherwise standard precautions suffice

Indications for stress ulcer prophylaxis Any 1 factor

Coagulopathy: platelets <50,000/mm3, INR >1.5, PTT >2x normal control Mechanical ventilation >48 hours GI bleeding or ulceration in last 12 months Head trauma, spinal cord injury, major burn

1st line preventive for variceal bleed = nonselective beta blockers which help reduce pressure in the portal venous system. The combination of ___ and repeat ____ with _____ results in an even lower mortality rate than either alone.

Combo of 1. beta blockers (eg, propranolol, nadolol), 2. repeat surveillance endoscopy with band ligation = lower mortality rate than either treatment alone. - adding oral nitrate may also further reduce rebleeding.

what is the name of the 2 signs seen with pancreatitis and what is the difference between them?

Cullen's and Grey turner's signs Cullen sign: edema and fat in the SQ region at the umbilicus and is a sign of acute pancreatitis. Grey Turner sign: bruising of the flank caused by pancreatic necrosis or retroperitoneal or intra-abdominal bleeding.

Diptheria diagnosis and treatment for severe and normal infection?

Culture from respiratory secretions and toxin assy to prove toxigenic Treat with erythromycin or penG If severe diptheria antitoxin

Biopsy of cutaneous necrotic lesion with umbilication positive on Periodic acid schiff gomori methenamine silver nitrate stain for encapsulated yeast. What is treatment initial and maintenance?

Cutaneous cryptococcosis is usually treated with >2 weeks of IV amphotericin B and oral flucytosine followed by a year of oral fluconazole (higher dose for 8 weeks, then maintenance).

Hepatic encephalopathy Precipitating factors

Drugs (eg, sedatives, narcotics) Hypovolemia (eg, diarrhea) Electrolyte changes (eg, hypokalemia) ↑ Nitrogen load (eg, GI bleeding) Infection (eg, pneumonia, UTI, SBP) Portosystemic shunting (eg, TIPS)

What imaging should be done to RO vascular cause of ED

Duplex US or penile Angiography in a older smoker in arterial insufficiency

Is dyspepsia the same as gerd?

Dyspepsia = Epigastric pain "burning" ± Nausea, vomiting, epigastric fullness, heartburn. Not same as gerd! Causes: Idiopathic ((functional) - MOST COMMON) GERD PUD NSAID use H PYLORI

WHAT IS the test and treat method for someone with epigastric discomfort and fullness. Can they leave office with treatment?

Dyspepsia: epigastric discomfort, fullness and associated with nausea. test-and-treat method for Helicobacter pylori infection is recommended in patients age <60 without alarm symptoms.

In patient with long standing poorly controlled celiacs disease what condition are they at risk for and how would it manifest?

EATL manifests as weight loss, MELENA, diahrrea, bloody stools, B symptoms and abdominal pain Can lead to bowl obstruciton or perferation This is prevented with gluten free diet !

______ are the classic radiologic finding for obstructive jaundice.

EG ascending cholangitis. -Dilated intrahepatic ducts: the obstruction --> ductal dilation on imaging.

Bacterial causes for enterohemorrhagic acute diahrrea?

EHEC: NOT GIVEN ABX! SHIGELLA CAMPY SALMONELLA

What toxicity is a comon culprit of osmolar gap?

ETOH

What should be done if pt who is at 13weeks gestation has a crescentic hypoechoic area btw gestational sac and uterus consistent with a hematoma?

EXPECTANT MGMT ONLY incedental finding subchorionic hematoma can be due to anticoagulant use, infertility tx, uterine malformation but can be incidental too Can present with bleeding

What is the first manifestation of sacroiliitis? What testing is 1st ? Is hla b27 diagnostic?

Earliest change seen on radiograph thus plain x-ray of SI joint is first diagnostic: HLA b27 is not specific but if negative will RO sacroiliitis. (MRI for progression not hlab27 that's only for diagnosis when x ray is not diagnostic ) Clinical criteria = insidious onset 1. 3 months; LBP morning stiffness improves with exercise 2. reduced forward flexion of L spine on schober testing and 3. reduced chest expansion

What reduces the chances of fat embolism s/p mva? Supportive care is the mainstay of therapy for clinically apparent fat embolism.

Early immobilization and operative fixation of fractures There is no evidence that prophylactic heparin reduces the risk of fat embolism.

What symptoms present in phenytoin toxicity and what levels are normal? Does OCP affect absorption of Phenytoin?

Early sign is presence of nystagmus on far lateral gaze (neurotoxicity) Dose should be less that 20 mcg, thus drug should be decreased if it is near 20. OCP pills do not affect drug level of phenytoin, conversely chronic phenytoin use causes failure of OCP to work bc of induction of hepatic enzymes.

Dx and Tx of: Hypertension in Pregnancy with new-onset tonic-clonic, focal, or multifocal seizures

Eclampsia Supportive measures- Initiation of magnesium sulfate for seizure prophylaxis- Delivery once maternal stability has been achieved

What is the management in pt with adnexal mass, +bhcg >1500 (aka high enough to be discernable on pelvic US), an empty uterus on US?

Ectopic pregnancy Management Stable: methotrexate Unstable: surgery

When would an adjustment to anti-d immune globuline be needed?

Eg. abruptio placentae results in maternal alloimmunization. Feto-Maternal hemmorage Kleihauer betke test should be done to determine amount

What microbes should be targeted in a human bite?

Eikenella corrodens strep viridans (alpha hemolytic strep) staph aureeus anerobes MGMT: local wound care, no primary closure(except face), abx and tetanus booster. Treatment with AUGMENTIN = amoxicillin/clavulanate (amp/sulbactam if parenteral tx needed) is a good treatment for mammalian bites (inc. humans) provides aerobic and anaerobic coverage

When is it necessary to order an FSH level or pelvic ultrasound in someone with delayed onset of puberty in female?

Either if they have lack of menses >15yo or lack of breast development with sort stature in female >12yo. Otherwise menses usually follows after 2-2.5 years after breast develop.

metabolic causes of Long qt syndrome and TDP

Electrolyte imbalances (↓ potassium, ↓ magnesium, ↓ calcium) Starvation Hypothyroidism

What is the most common transfusion reaction? What are symptoms and onset time?

Febrile NON - hemolytic transfusion reaction: - time: 1-6 hrs s/p transfusion - cause: 2/2 cytokine accumulation during storage - symptoms: fever and chills - prevention: leukoreduciton.

Knee jerk is lost and sensation on anterior thigh and medial aspect of leg/ foot is decreased what nerve is damaged? How about it peroneal nerve injury?

Femoral n --> saphenous n Peroneal nerve which runs over lateral fibular head --> superficial peroneal nerve foot drop bc the foot cannot dorsiflex and evert

What type of hip fracture typically present with pain without significant ecchymoses and have a higher risk of avascular necrosis?

Femoral neck fractures

Which molar pregnancy (complete vs partial hydatidiform) can p/w a fetus, amniotic fluid and gestational sac can be seen on ultrasound?

Fetus visualized with amniotic and gestaional sac is seen with partial molar pregnancies (sometimes even with fetal heartbeat), whereas no fetus is seen in complete molar pregnancy.

Centor criteria

Fever, tonsilar exudate, no cough = strep pharyngitis

Treatment of Clostridioides difficile infection recurrence?

First recurrence: - Vancomycin PO in a prolonged pulse/taper course OR - Fidaxomicin ( IF vanco used in initialLY) Multiple recurrences: - Vancomycin PO followed by rifaximin -Fecal microbiota transplant + rifaximin (or above regimens)

Treatment of asymptomatic bacteriuria during pregnancy and is retest needed?

First-line antibiotics during pregnancy include cephalexin, amoxicillin-clavulanate, or fosfomycin treatment failure is common, a repeat urine culture is performed a week after completion of the antibiotic regimen to test for cure.

What is the management of someone with PU perforation on xray?

Fluid resuscitation and BSA IV with Gram negative coverage and IV PPI prior to having EMERGENT SURGERY

___ is the antidote of choice in cases of ethylene glycol and methanol intoxication. Simultaneous use with ____ is not recommended.

Fomepizole Fomepizole prolongs t1/2 ethanol and binds ADH better than etoh and prevents formaiton of toxic metabolites --> improved acidemia and prevents renal failure

When would one need a Repeat DXA scan in 6 months?

Frequent (eg, at 6-month intervals) bone density testing is recommended for patients with unusually rapid bone loss (eg, systemic glucocorticoid therapy).

STATS: What does this specific image of a funnel plot describe? What does the absence of symmetry reflect?

Funnel plots should be symmetric in absence of bias if it is not then could be due to heterogeneity, method anaomalies, artifact or chance Most common reson for absence is no heterogeneity and publication bias otherwise the plot would be symmetric and funneled to top rather than at base

HIV pt with CD4 49 needs what medication if presenting with odonophagia and dysphagia?

GANCYCLOVIR

Anterior cervical LAD and tonsillar exudates, FEVER, is likely due to what infection?

GAS infection pharyngitis

First line treatment for toxic megacolon is? What drugs should be avoided in toxic megacolon?

GC to lessen colitis and abx for infectious colitis 5ASA and opoids should be AVOIDED in pt with toxic megacolon

What is the relative reduction in mortality risk Lung cancer screening with low-dose chest CT? Does the false positive rate tell you about chance of getting cancer if a 4mm lung nodule found ?

age 55-80 with a >30/ or quit within the last 15 years. Screening is associated with a 20% relative reduction in mortality risk but a false positive rate of nearly 96%. >/ 4mm lung nodule found 10% rate of having lung cancer patients should be advised that any positive findings will require subsequent confirmation.

What is the most important risk for osteoporosis and bone fracture?

age is the single most important factor!

Statins are indicated for all patients age ≥_______ with ______ .

age ≥40 diabetes mellitus.

Failure to adjust the dose of anti-D immune globulin after massive fetomaternal hemorrhage (eg, abruptio placentae) may result in maternal ______.

alloimmunization

visual impairment and hematuria

also hearing loss = alport syndrome! x linked (commonly)

bismuth subsalicylate, metronidazole, and tetracycline (BMT) for 14 days; and lansoprazole, amoxicillin, and clarithromycin (LAC), which has been approved for either 10 days or 14 days of treatment.

alternative tx pud but 10 day tx for OAC

LCIS found on biopsy needs what follow up management and why?

although benign can progress the DCIS or become invasive thus must get excisional bx

What is the most common nosocomial diarrhea and what are the antibiotic restricitons that can be placed?

aminoglycosides or TMP/SMX used in place of FQN Also infection control with contact precautions, washing hands SOAP AND WATER. Hospital control with cleaning hypoclorite sln.

AFIB treatment (hint: abcd)

amiodarone bb ccb- non dihydrop. digoxin

§UTIs are common in pregnant women and should be treated with safe and appropriate empiric antibiotics until culture results return. Options for empiric treatment include

amoxicillin-clavulanate, fosfomycin, and cefpodoxime. bactrim (ok if 2nd trimester) avoid at term and 1st trimester avoid Macrobid at term

obstructing endobronchial malignancy should be suspected in patients with what risk factors?

an extensive smoking history who have nonresolution of a pneumonia. 1. Initial pneumonia = xray 2. nonresolution / reoccured in 3 weeks = chest CT 3. bronchoscopy 4. Open lung biopsy would only be considered if both chest CT and bronchoscopy were nondiagnostic

persistent lymphadenopathy is sometimes the first clinical manifestation of

an underlying lymphoma. eg pt with LAD >1month after infection should be evaluated with bx

NEc fasc with crepitus is more common among what organisims?

anaerobics -Clostridium perfringens or - B fragilis,

PT WITH OCCULT GI BLEED and colonoscopy with small flat cherry red lesions in right colon in a pt who has low H/h and 400 plt is likely due to what?

angiodysplasia 2/2 ESRD! End stage kidney disease. - age >60 due to chronic occlusion of submucosal veins forming av collaterals - common cause of hematochezia if predisposing factors of esrd, vwd, and AS present

centor

anterior cervical lymphadenopathy and tonsillar exudates + fever = group A streptococcal pharyngitis

ECG shows >2 mm ST elevation in V2 and V3, and >1 mm ST elevation in aVL, V1, and V4 (each box is 5 mm in height), consistent with ---?

anterolateral STEMI. Reciprocal ST depression in the inferior ECG leads (II, III, and aVF) is also present.

PE treatment

anticoagulation UFH or anti factor Xa bolus 80u/kg inital infusion - max 10k units 18u /kg/hr with max 2k units/hr

Common medications implicated in LQTS include

antipsychotics, antidepressants, macrolides, fluoroquinolones, and antifungals.

what murmur is associated with a wide pulse pressure?

aortic regurg.

Reversible causes of falls in the elderly due to balance and gait issues should be screened with what examination? & potential interventions ?

GET UP AND GO eval for postural instability - NOT CT HEAD! IN absence of trauma, elderly, multiple falls or have gait abnormalities should be evaluated with a comprehensive and multifactorial approach = medication use, alcohol use, chronic diseases, environmental factors, blood pressure, and postural stability. intervention includes: - Environmental modifications - PT for gait & balance training - Assessment for assistive device - Exercise program

Diagnosis of primary psychotic disorders EG PT W/ New-onset psychosis in the ED the physician must first rule out

GET Urine toxicology FIRST New-onset psychosis : ED the physician must first RO substance-induced and medical causes. Intoxication substances: cocaine, amphetamines, and phencyclidine may mimic the symptoms of schizophrenia

Explosive onset of multiple itchy seborrheic keratoses?

GI MALIGNANCY

Anion gap metabolic acidosis GOLDMARK VS MUDPILES:

GOLDMARK - GLYCOLS ethylene and propylene - Oxyproline - L lactate - D lactate - Methanol - Aspirin (salicylates ) - Renal failure (uremia), Rhabdomyolysis - Ketoacidosis (etoh, diabetic) Methanol, Uremia, Diabetic ketoacidosis (or alcoholic ketoacidosis,) Paraldehyde, Iron (or Isoniazid,) Lactic acidosis, Ethylene glycol, and Salicylates.

Side effects of SGLT2 inhibitors include what GU infections and what type of fluid loss?

Genitourinary infections Vulvovaginal candidiasis Urinary tract infection Fluid loss Symptomatic hypotension Acute kidney injury

_____ is avoided during pregnancy as it crosses the placenta and is associated with irreversible congenital deafness.

Gentamicin, an aminoglycoside,

What is the treatment of acute inferior wall MI?

Give them IV atropine in those with hemodynamically significant bradycardia (eg pulmonary edema, hypotension) due to inferior wall MI. Even if nitro was given do not fluid resuscitate otherwise you will risk pulmonary edema worsening.

When is prophylactic anti d IGG for Rh D negative patients needed (hint your mom was RHD negative and so am I) ?

Given at 28-32 weeks if negative anti d ab screening 1. < 72 hrs after rhd + child delivery 2. <72 hrs after 3. spontaneous abortion 4. threatened abortion 5. hydatidiform mole 6. CVS sampling and amniocentesis 7. abdominal trauma 8. 2nd 3rd trimester bleed 9. external cephalic version

Long-acting insulin that has a peakless effect - higher concentrations of (300 vs. 100 units/mL) have a longer duration of action

Glargine. 20-24H. Peakless effect less likely to cause hypoglycemia

Indications for stress ulcer prophylaxis >2 factors

Glucocorticoid therapy >1 week ICU stay Occult GI bleeding >6 days Sepsis GI = gastrointestinal; ICU = intensive care unit; INR = International Normalized Ratio; PTT = partial thromboplastin time.

STATS: What is the primary purpose of a study with an intention to treat approach?

Goal: preserving randomization - ITT analysis tries to avoid the effects of crossover and dropout which break randomization - trials are analyzed in groups - groups are randomized regardless if they got treatment, dropped out, or adhered to the intervention!

Osteomyelitis antibiotic choice? What adjunctive treatment also done for OM in DM2?

Gold standard is to get a BONE Biopsy w CULTURES, DM foot infection treated with glucose control, surgical debridement, revascularization, weight off-loading, and ABX> 6 WEEKS

ANTI GBM is used to diagnose which renal pathology? Serum cryoglobulins are associated with what viral infection and renal syndrome?

Goodpastures: hemoptosis Hep C infection and is called mixed cryoglobulinemia HEP B,a ,d hiv = FSGS

What rash presents in Dermato-M and what is the treatment? What screening should be done?

Gottron's papules and heliotrope rash manage with high dose glucocorticoids PLUS gc-sparing agents Screen for malignancy they are at increased risk of adenocarcinoma (most common cancer) DM can resolve if cancer is treated (LOC dem lung ovarian colon cancer)

persistant URI, eye and ear disease along with RPGN, skin and arthritic symptoms and + c-ANCA should have what treatment (hint glasses and nerds )

Granulomatosis with polyangiitis (GPA dude got a C on test) corticosteroids and cylophosphamide CCCCCC

What pathogen does grisleofluvin treat and which pathogen does it worsen?

Griseofulvin is used in 1. tinea capitis, a dermatophyte infection most commonly seen in children, and 2. tinea corporis, wassim causes annular plaques + peripheral scaling and central clearing. Griseofulvin is NOT effective against Malassezia species and can worsen SD Subhor. dermatitis.

treatment w/ IVIG or plasma exchange results in what

Guillain - Barré recover without intervention, treatment with plasma exchange or intravenous immunoglobulin shortens the time to recovery by approximately 50%.

If no dangerous signs of HA what should be initial step in evaluation?

H&P Headache diaries - frequency, duration, intensity, associated symptoms, and medications used for treatment. >10 days/month for >3 months = secondary headache disorder w/ analgesics (eg, acetaminophen) overuse

Iron deficiency anemia is common in infancy and is characterized by hemoglobin <____, low mean corpuscular volume, and Mentzer index >____ and commonly due to what ? Prevention of FE deficit.: introducing iron-rich foods (eg, pureed meats) @ 6mo

HB <11 g/dL, LOW MCV and Mentzer index >13. DUE TO: breast milk diet thus, introducing iron-rich foods (eg, pureed meats) around age 6 months is necessary to prevent iron deficiency.

What is a common cause of hepatic encephalopathy?

HE is triggered by elevated ammonia due to underlying event = commonly excessive diuresis which reduces IVascular volume --> HYPOKALEMIA and metabolic alkalosis.

HELLP syndrome most commonly presents with what?

HELLP s/o p/w 1. abdominal pain--> followed by nausea, vomiting, and malaise. HTN, proteinurea are NOT necessary as HELLP may not p/w preeclampsia 15% of the time!

Hypertension in Pregnancy

HELLP syndrome is thought to be a subset of preeclampsia with severe features - TCP: platelets <100,K - Renal CRT >1.1 mg/dL or a double of the baseline - Elevated liver transaminases to twice normal concentration

In a pt with ast 1000's, INR 1, Hep BSaG and IGM positive, who is icteric and jaundice what is her diagnosis? Is hospitalization necessary and what is the treatment? What happens if after 8 weeks AST drops but titters are high? When would HepB vaccine and Immuneglobulin be used?

HEP B treatment is close OUTPATIENT supportive care and lab f/u HEP B almost never progresses to Acute Liver failure which would warent hospitalization however have: 1. INR >1.5 2. Liver injury (eg ast) 3. Encephalopathy (LIE PNEUMONIC HEP B LIVER FAILURE) Otherwise Hep B resolves spontaneously and needs only outpatient care and close f/u to monitor for drop-in ast and HBV DNA levels HBV SURFACE AG usually clears after 6mo. If it does not clear by 6mo (5% adults) go on to develop Chronic Hep B. Then pt can be tested for Viral genotype for Interferon therapy for Genotype A Vaccine and IG used for Post exposure prophylaxis and for Health care worker exposure (give within 24 hours)

Does RA have a high or low ferritin?

HIGH ferritin is an acute phase reactant

What does someone with sudden onset psoriasis, HSV recurrence, and disseminated mulluscum contagiosum have?

HIV

Who is at risk for cutaneous cryptococcosis; how does it manifest? What is the treatment What is a HY association with relapsing HA in setting of Crypto?

HIV + individuals with CD4 count <100; P/W: papular lesions with central umbilication and necrosis (center is red); D/w: lesion bx with histo T/w: >2weeks of IV AMPHOTERICIN B + oral flucytosine f/b: 1 year of oral fluconazole Fungal burden so high in HIV there is arachnoid matter blockage and Increased ICP --> TREAT WITH serial LP!

A young male with submandibular LAD (single non-tender nodule in neck) along with a plaque that can be scraped off in mouth most likely needs what testing and why?

HIV1/2 antigen + p24 antigen (4th gen) testing as oropharyngeal thrush in a young male without immunosuppression is suggestive of HIV and requires 4th gen testing

What are PAD risk factors?

HLD, DM, smoking Note can cause butt and thigh pain due to intermittent claudication from aortoiliac PAD and can caus eweakness of hip and thigh during walking Bilateral aortoiliac dx can cause ED like in Leriche syndrome

Why did the radiation onc. doc at ARMC see so many head and neck cancer pt who were bald from chemo?

HNC are majorly inoperable as 60% are stage 3/4 at time of dx! Thus a combinaiton of CHEMO AND RADIATION CRT promises superior 5 year survival rate!

ACS assessment

HP 12 lead EKG every 10 minutes Troponin level Electrolytes CBC Cardiology consult if shock, new LHF or sustained VT

STATS: What is the use of hazard ratio and how does it differ from RR

HR: chance of event occurring in treatment vs control. HR>1 inc risk HR = 1 no change in risk HR: can be calculated at MULTIPLE time intervals in study period (while RR does not) ken state stats: https://libguides.library.kent.edu/SPSS/PearsonCorr

what puts you at increased risk for Aortic dissection?

HTN (elevation of SBP)

Clinical features of aortic dissection?

HTN hx, genetic marfans, sharp tearing chest back pain, >20mmhg difference in SBP btw arms

Wha is goal bp for HTN with DM or with CKD?

HTN with DM goal bp should be less than 140/90 in CKD bp should be <130/80 in nephropathy dm pt and proteinuria ≥500 mg per day

What distinguishes thyroid lymphoma from other thyroid cancers?

Hashimotos thyroidits contributes to cause Rapidly enlarging neck mass

When is neuroimaging needed to assess macrocephally in a kid?

Head circumference measured at birth- age 2-3 would be macrocephalic if >97th percentile for age Neuroimaging needed with rapid expansion, seizures or develop delay (further eval for chromosomes may be warrented AFTER ULTRASOUND! eg sotos s/o or organic aciduria).

What are the symptoms of pheochromocytoma and what is diagnosis?

Headache, Diaphoresis, tachycardia, paroxysmal HTN, suddent onset refractor HTN Diagnosis is with plasma free metanephrine or 24 hr urine catecholamine + metanephrines (not vanyl acid low s$s) In setting of normal TSH and anxiety RO

Stage D HF definition

Heart failure symptoms at rest or refractory end-stage heart failure

What is the difference btw hellp and TTP? When does each occur?

Hellp in 3rd trimester has maha, tcp, and ELEVATED TRANSAMINASES TTP can occur during pregnancy or postpartum period and Ast/alt are normal

When do you not get a colonoscopy in someone with BRBPR

Hemodynamically unstable after initial resuscitation attempt --> EGD THEN CSP Contraindicated in DiverticuLITIS for risk of perforation (not to be confused with diverticulOSIS which does need an CSP)

Descending dissection complications?

Hemothroax or peritoneum renal injury (renal a. occlusion) SMA occlusion LE ischemia Paraplegia - sCa ischemia

HBsAG means what?

HepB chronic infection

_______ is an oropharyngeal infection in child who p/w fever, drooling, sore throat, loss of appetite, headache, and painful posterior pharyngeal vesicles.

Herpangina caused by Coxsackie group A virus usually seen during the summer in children age 3-10 self limiting, handwashing prevention

Kawasaki disease treatment

High-dose ASA (for inflammation and fever) and IVIG (to prevent aneurysms). Low-dose ASA is then continued, usually for 6 weeks. Children who develop coronary aneurysms may require chronic anticoagulation with ASA or other anti-platelet medications. Contortionists may be used in IVIG-refractory cases, but routine use is not recommended.

What is SNout and SPin mean?

Highly sensitive test for screening thus high sensitivity and negative result rules out disease Specific test, positive test rules in disease thus confirmatory

_______ p/w abdominal distension delayed passage of meconium, expulsion of stool on rectal examination in Neonatal period. _________ gold standard for diagnosis demonstrates the absence of ganglion cells from the affected segment and nerve fiber hypertrophy.

Hirschsprung's disease Rectal suction biopsy, Patho: failure of neural crest cells to migrate to the distal colon during development,

________ exposure accounts for the majority of C difficile transmissions.

Hospitalization - Nosocomial

What is the most common culprit of asthma?

House dust mites (not air pollution)

AUDIT-C vs AUDIT

How often do you drink alcohol? How many drinks do you have on a typical day when you are drinking? How often do you have 6 (4 for women) or more drinks on 1 occasion? 10-item screen assessing frequency, number of drinks & psychosocial consequences

What testing should be done in someone with SAH but negative non- contrast CT

If early presentation may be missed on imaging in first 6 hrs. Must diffinitively RO SAH with LP if no xanthochromia MRI with contrast is § similar to head ct aka wrong answer

When can you treat with statins without calculating an ascvd score?

If pt has LDL >190, and is >45yo with DM2 Otherwise ascvd score of 7.5-10 needed to treat with moderate or high intensity statins.

In the event that a HCP gets stuck by a needle and the pt refuses HIV testing, what should be done?

If pt refuses first administer DO NOT TEST PT OR waste time talking to the pt, give HIV PEP to HCP immediately! Time is of the essence in this senario and PEP can be discontinued later but not started later.

When can DM2 not be diagnosed if pt has elevated hbA1c

If they are asymptomatic then screen repeated on another day. Formally need at least 2 criteria to meet DM2 diagnosis that day (eg Hba1c ≥6.8 + Random fasting >200) or 1 lab value + symptoms can be diagnosed with DM2 without confirmatory testing!

When are glucocorticoids avoided in toxic megacolon?

If toxic megacolon 2/2 cdiff infection

What kidney disease occurs only after strep throat but not strep impetigo and presents with gross hematuria?

IgA nephropathy (bergers) may treat with CST or ACE but self limiting

When do you need to stop a blood transfusion and give intravenous glucocorticoids?

Iga deficint people who have anaphylactic reaction seconds after transfusion.

When is vanco administered rectally?

In fulminant cdiff c diff infection WITH ILEUS Fulminant: hypotension/shock,ileus, megacolon - IV metro with high-dose vancomycin PO - IV METRO + VANCO PR rectallY (if ileus is present) - Surgical evaluation

can c-section be done on HELLP?

In patients with HELLP syndrome, vaginal delivery should be performed after 34 weeks of gestation. c- section with REGIONAL anesthesia cant be done if plt <100k! thus they need GENERAL ANESTHESIA to proceed which has higher M&M to both mom and baby

When would sputum or gastric lavage acid fast bacilli smear and culture be needed?

In someone who has active pulmonary TB NOT latent tb

Most common thyroid cancer morphology?

are solid and cold ("Cold as ice!"). Remember the "P's" of papillary thyroid carcinoma: most Popular, Papillae, "Pupil" nuclei (Orphan Annie nuclei), Psammoma bodies, and Positive Prognosis.

When venous air embolism is suspected, immediately _____.

aspirate the catheter, administer 100% oxygen, and place the patient in the lateral position with left side down.

pediatric condition treated with nebs (beta agonist) vs when would you need to get CXR and neck x-ray?

asthma which presents with wheezing (rather than stridor in croup and epiglot) treated with nebs beta agonist.

when is rho gam administered to women who are Rh negative?

at 28w and <72 hrs post-partum

Phenytoin intoxication can occur with increased doses. The most common manifestations are

ataxia, nystagmus, and drowsiness. The overdose syndrome is usually mild.

Pt 2-5 days post-op; c/b increased work of breathing or hypoxemia. ABG: increased alveolar-arterial gradient due to intrapulmonary shunting. CXR: linear opacifications in the bilateral lung bases, sometimes with an accompanying shift of structures toward the opacification (if large). What is management?

atelectasis managed with - mild: CPAP - severe: aggressive pulmonary hygiene, including chest physiotherapy and suctioning, rather than CPAP.

Patients with unexplained right atrial or ventricular dilation should be evaluated for possible _______ Cardiac auscultation wide and fixed splitting of the second heart sound.

atrial septal defect (ASD): with large left-to-right shunt and normal pulmonary artery pressure typically reveals a characteristic 2nd heart sound Echo shows dilation of Right atria and ventricle

OP poisoning treatment

atropine --> praladoxamine = atrrrrr perfume pray pralidoxamine is an acytylcholeneesterase activator

bradycardia treatment

atropine pines

If someone has T1DM what else could they have

autoimmune conditions make you more prone to development of other autoimmune diseases!

In addition to minimizing antibiotic use, which of the following prevents recurrent CDAD episodes of the disease ?

avoid PPI OR gastric acid suppression

what is commonly damaged with anterior shoulder dislocaiton

axillary nerve responsible Specifically, the deltoid is the main muscle responsible for abduction after the first 15 degrees.

pediatric condition treated with IV abx?

bacterial pneumonia which p/w rales fever cough and tachypnea Toxic appearance: high fever, drooling, leaning, respiratory distress!) Intubation and intravenous antibiotic therapy are required for patients suffering from airway compromise secondary to epiglottitis.COMBANK Insight : Another clinical scenario of epiglottitis may present you with the following picture with the classic "thumb-print" sign.

Seborrheic dermatitis is a chronic, relapsing condition. Initial treatment can provide significant improvement in symptoms, but patients usually benefit from intermittent re-treatment WITH WHAT?

benefit from intermittent re-treatment. Typical regimens include topical ketoconazole or ciclopirox every 1-2 weeks. DANDRIF SHAMPOO RINSE AND REPEAT

Clostridium difficile Nontoxigenic strains are typically

benign, but toxigenic strains can cause colitis in susceptible patients (eg, hospitalized, elderly, recent antibiotic users).

Intravenous ____ can be used if there is evidence of atrial fibrillation leading to an embolic stroke.

beta blockers

For relief of stable angina symptoms with what meds?

beta blockers (BBs), calcium channel blockers (CACBs), and nitrates?

Due to birth mechanics, ____ is a common dysfunction in the postpartum patient and contributes to low back pain.

bilateral sacral flexion bilateral deep sulci, bilateral shallow ILAs, and increased lumbar curvature. SFT: false-negative SPRING: springing at sulci, and restricted ILAs

Diagnosis of Vibrio vulnificus?

blood and wound cultures; treatment with intravenous antibiotics should not be delayed.

What should be done for a HCP who is stuck by needle of pt who is positive for HBsAG and HBcore Antibody but negative HBeAG? If the HCP worker had vaccination but titer in <10 1-2 months after series?

In worker who had a vaccination series but failed to have addequate immune response (hep surface ab ≥10) should be treated as if they never had a vaccination and given IG THus Hep B vaccine given in one arm and HepB immune globulin given at another site, followed by the rest of series later. This senario is the only time you give HBIG otherwise the PEP is just vaccination!

Pathogenesis of refeeding s/o? Would a normal heart rate be good in someone who was given TPN if they were initially bradycardic

Inadequate ATP and thiamine = tissue hypoxia --> myocardial dysfunction. Low K and MG --> arrhythmias. CHF develops from a weak, atrophic heart that cannot handle fluid and electrolyte shifts. Thus a "normal" heart rate (60-100/min) in formerly bradycardic patients can be an ominous sign of cardiac compromise.

Mother was likely infected by ______ to have a baby w/ torch infection that may manifest as eye abnormalities, neurologic findings, and hearing impairment.

Inadvertent consumption by pregnant women of the toxoplasmosis parasite in cat feces, undercooked meat, or contaminated soil may be associated with congenital disease,

Do incarcerated individuals have the right to consent in research trials? And is acceptance of compensation considered coercive?

Incarcerated people have right to consent but need further IRB review to ensure there is no exploitation They can accept compensation if it is not considered coercive by IRB oversight

when is levothyroxine increased in prego?

Increase levothyroxine dosage after 5 weeks into the first trimester.

Papilledema occurs when elevated ICP is transmitted to the optic nerve. Common causes include the following: Mass lesions

Increased cerebrospinal fluid production Decreased cerebrospinal fluid outflow (venous sinus thrombosis) Idiopathic intracranial hypertension (pseudotumor cerebri) Patients such as this one with a chronic daily headache and evidence of raised ICP should undergo neuroimaging, preferably an MRI with contrast. If there is a concern for acute subarachnoid hemorrhage (eg, acute onset, "worst headache of life"), noncontrast CT scan is preferred.

What is diagnostic of vitamin B12 deficiency (cobalamin deficiency)?

Increased methylmalonic acid and homocysteine levels

When would colonoscopy be indicated in someone who did prior screening w/ sigmoidoscopy

Increased risk for advanced neoplasia in the proximal colon is seen with: 1. Large (>1.0 cm) adenomatous polyps 2. Multiple adenomatous polyps 3. Polyps with villous or tubulovillous morphology Since sigmoidoscopy unable to see proximal colon and is less sensitive any such polyps detected on sigmoidoscopy should have visualization of the entire colon with colonoscopy ASAP!

When is fibrate FIBR888888 therapy needed?

Indicated to reduce TG levels in pt with severe hypertryglyceridemia (>880)

What are the indications to treat subclinical hyperthyroidism?

Indications for treatment TSH persistently <0.1 µU/mL TSH 0.1-0.5 µU/mL plus additional risk factors: 1. Age >65 2. Heart disease 3. Osteoporosis 4. Nodular thyroid disease

Septic arthritis of the hip in peds common at what age - clinical sx - red flags - labs - treatment

Infants with septic arthritis have subtle sx: irritability, poor feeding, aversion to being held, and pseudoparalysis. red flags are for infection. +/- fever may be absent, elevated inflammatory markers treat: Joint drainage & debridement IV antibiotics - <3mo treat for gram neg bacilli (ecoli?), GBS staph A, if ≥3mo treat with GAS and staph A

MAT initial treatment and subsequent treatment is focused around what? When are av blocking agents used? Who gets MAT and what is the diagnostic criteria ?

Initial = non-invasive ventilation followed by correction of underlying changes - pulmonary disease - electolyte disturbances such as hypokalemai Lastly ndCCB like verapamil (also BB esmolol) is used in continued MAT to low AV conduction and RVR of MAT. Diagnosis via rate >100 and at least 3 different p morphologies on EKG

EKG with initial evidence of anteriorlateral STEMI requires what further management?

Initial EKG can be diagnostic of ACS due to stemi and does not need further tests! percutaneous coronary intervention (PCI), 90 minutes of first medical contact or within 120 minutes for patients who require transfer to a PCI-capable facility.

What is the initial treatment goal for RA, what class of drugs should be considered after failure of initial therapy?

Initial treatment is to control synovitis and slow joint destruction. DMARDS slow progression of disease 1st: MTX chosen as initial and combined with oral glucocorticoid (prednisone) for symptom relief. 2nd: if symptoms persist then biologics

What testing should be done if someone has a triad of fever, severe spinal tenderness, neuro deficits?

Initial: CBC, ESR, CRP and URGENT MRI w/ contrast to evaluate for (SEA) and cord compression Progress to develop radiculopathy, motor/ sensory deficits, incontinence --> eventual paralysis.

What part of the colon is bx to diagnose Celiacs? What other tests should be ordered?

Initial: evaluate for increased osmotic gap Labs: IGA anti-tissue transglutaminase Small bowel bx: distal duodenum shows villous atrophy* lymphocyte crypt hyperplasia...

3 Phases of CHAGAS?

Initial: fever myalgia Intermediate: serologic phase without symptoms FInal: dilated CMP, R heart failure, GI disease

What labs should be ordered to diagnose someone with proximal muscle weakness, interstitial Lung disease, dysphasia, and myocarditis?

Initially ANA screening is test of choice (80% +) Diagnosis with increased: CPK, Aldolase, LDH, anti RNP, anti JO 1, anti MI 2 diagnostic uncertainty warrants EMG and bx of skin/mm

Child with marasmus and kwashiorkor what is the initial and subsequent managment?

Initially oral rehydration (IV if shock) Abx cautiously refeed If done quickly can risk heart failure or refeeding s/o.

What is the preferred therapy for Guillain-Barré syndrome.

Initially: assess vital capacity! Second: give Pooled human immune globulin or Plasma exchange

Internal validity answers what question? What is major threat to internal validity and how can we mitigate it?

Internal validity = cause and effect in a study and answers the question, "Are we observing/measuring what we think we are observing/measuring?" The major threat to internal validity is confounding, Randomization improves internal validity by balancing the distribution of confounding variables among the groups.

Who is at high risk of having lead toxicity and what should be done if it is severe?

International adoptees / immigrants Severe tox with acute encephalopathy needs hospitalization with IM dimercaprol that crosses BBB --> and EDTA to lower risk of severe encephalopathy

After preforming air enema for Intussuception what could be an adverse outcome and how is it recognized/ treated?

Intestinal perforation s/p air or saline enema from corrected telescoped cecum into the ilium Xray diagnosis and intestinal perforation is taken to surgery asap.

How does carbamezepine intoxication manifest?

Intoxication causes drowsiness, stupor, coma, or seizures. However, dilated pupils and tachycardia are more common.

What is the prefered way of treating keloid scar?

Intralesional glucocorticoids (like alopecia)

______ is used for pharmacological cardioversion in patients with atrial fibrillation.

Intravenous amiodarone

How is airway compromise treated in epiglottitis?

Intubation and IV antibiotic therapy are

In a child who has severe abdominal pain with current jelly stools should have what for diagnosis and treatment?

Intussiception can be diagnosed via target sign on US However air or saline contrast enema is both the gold standard for diagnosis and is also theraputic!

What is this difference between someone with mekels or intussuception?

Intussuception has crampy abdominal pain with current jelly stools and NONBILIOUS VOMITIING MEKEL DIAGNOSED WITH TECHNICIUM 99 scan and is painless bleeding! Volvulus has bilious emesis and diagnosed via Upper GI SERIES

How do you determine brain death?

Irreversible absence of cerebral and brain stem reflexes ( peripheral reflexes remain present e.g: BABINSKY, limb twitches, and muscle movement)

STATS: If the CI does not include the null value (1.0 for SMR) what does that indicate?

It indicates statistically significant difference between observed and expected # death. statistical significance of smr is established by the confidence interval (if it crosses 1 - null value its Not Significant)

Shin splin and eventual tibial stress fracture is best seen with what imaging modality?

It is a clinical dx but evident on MRI after 3 weeks, x-ray usually will not show changes until much later.

STATS: What the hell is SMR? WTf does it mean is it freaking adjusted or not? SMR SOAD

It is effing adjusted to measure the overall mortality and calculated by dividing observed death by the expected number ( using a standard effing population) SOAD -SMR SYSTEM OF A DOWN? NO Smr= Observed/ (expected anticipated) death

STATS: What are the 3 events followed?

It is used when subjects in a clinical trial are followed over time and have the following events: 1. death, hospitalization, (event eg cancer) 2. completion of study 3. loss to follow up

STATS: factorial study design is considered fully crossed design because of what?

It utilizes ≥2 interventions and all combos of the intervention eg getting 1, both, none, or the other

When would daily rifampin be used in someone exposed to TB and who has a +IGA and is asymptomatic without TB findings on CXR?

It would be used on someone who likely has latent tb that has been found to be resistant to Isoniazide 9 month treatement. EG in pt whos brother was tested by gastric lavage and found to have resistant tb Thus treat with rifampin for 4-6month instead. Children with latent tb need treatment as they are at risk of conversion to active tb

Which nerve is responsible for innervating the flexor carpi radialis and thus allows for flexion and abduction of radiocarpal joint?

KATHRYN ITS THE MEDIAN NERVE NOT THE AIN

What are some contraindications to NSAID use?

Kidney disease CHF PUD NSAID ALLERGY Taking anticoagulation meds

MVA in a RH negative pregnant pt with normal BPF biophysical feedback test and HR should have what test done?

Klein beke test to detect amount of alloimmunization and thus amount of immune globuline to administer. Unless RH+

After first giving thiamine to WE you find they have long-standing brain damage to what part of the brain? and resulting in what syndrome?

Korsakoff syndrome (KS) permanent damage of the mammillary body from long standing WE-- confabulation

What is associated with embolic events in artery in a young pt without atherosclerosis ?

LAM left atrial myxoma - most common cardiac tumor presents with diastolic murmur, rapid onset heart failure, new AFIB, and embolize systemically leading to acute arterial occlusion in otherwise healthy young person!

In an asymptomatic person when is DMSA/succimer used? When would dimercaprol (british anti-lewisite + CA disodium edetate EDTA) ?

LEAD 45 TO <70 SEVERE LEAD ≥70

What is pathomneumonic for chagas disease on ECHO?

LEFT VENTRICULAR APICAL ANEURYSM

which carcinoma causes antibodys to attack voltage gated calcium channels and proximal mm weakness (think of fat dude sitting on ass with prison keys) with calciyum ice cream spilled

LEMS causes decreased ach due to blocked presynaptic calcium channels --> proximal mm weakness in small cell!

Primary osteolytic lesions are due to what 3 cancers? and what is the imaging modality needed?

LMNNOP L(lytic)MNN -Osteo- P(pet) myeloma non small cell non hodgkin lymphoma xray or pet or pet/ct

Euthyroid sick syndrome is characterize by what levels of TSH and t3t34?

LOW T3 normal tsh and t4 in pt with acute illness due to decreased peripheral conversion of t4 --> t3 no treatment needed unless thyroid function persist after 6 weeks / return to basline

STATS: What is the LR formula? Does it take into account prevalence? What does it tell you?

LR + sensitivity/ (1-sp) LR- (1-sensitivity)/ specificity LR tells you the likelihood of a pt with disease testing negative or positive Not based upon prevalence! PV tells you about the value of a diagnostic test (eg >10 ratio means strong evidence to rule in disease, (.5- 2) meaning no evidence to rule in or out the disease .1-.2 moderate evidence to RO disease <0.1 strong evidence to rule out dx)

borborygmi

Lactose intolerance is also frequently associated with borborygmi and abnormal bulky, watery stools.

PAH symptoms?

Left parasternal lift, right ventricular heave Loud P2, right-sided S3- diastole (s3 s4 diastole) Pansystolic murmur of tricuspid regurgitation JVD, ascites, peripheral edema, hepatomegaly cor pulm type3 pah; bisantine, pde inhibitor, pg

What idiopathic hip condition occurs in 3-10 year olds and evident on xray and p/w insidious-onset hip pain and limp. -Deformity of the femoral head is seen on radiograph and MRI.

Legg-Calvé-Perthes disease 1st do PT

Atypical presentations of what disease presents with Levine sign? What are the atypical symptoms of this disease and who should we suspect it in?

Levine sign: when pt place fist clenched in the center of their chest but cannot describe the pain! MI p/w abdominal pain in elderly, women, diabetics Treat: nba stays cut NitroBB , Asprin Statin Coronary reperfusion / anticoag /clopidogrel? (prevent serious morbidity and mortality)

If lights criteria is met what is the pathophys. and common cause?

Lights: P/S PROT >.5 P/S LDH >.6 P LDH>60 PATH: INFLAMMATION cased by: infection (tb pna) cancer connective tissue dx PEmbolism S/P CABG pancreatitis

Crepitus and swelling over med clavicle in a newborn is indicative of what diagnosis? What should be done?

Likely shoulder dystocia can be diagnosed via xray, no management needed only immobilization and monitor.

Which drug warrants prior screening for metabolic diseases such as thyroid function and DM

Lithium

What drug causes nephrogenic DI?

Lithium - treat with amilioride

What syndrome p/w erythema nodosum, lung adenopathy, migratory joint pain?

Lofgren's syndrome: erythema nodosum, bilateral hilar adenopathy, migratory polyarthralgias, and fever. 95% diagnostic specificity and allows diagnosis of sarcoidosis without a biopsy.

What is a good prognosis of symptoms in a european with sarcoid? How is it treated?

Lofgren's syndrome: portends to a good prognosis. Symptomatic treatment of mild disease with -NSAID agents or - low dose prednisone may be added.

Lumbar spinal stenosis is frequently seen in elderly patients. It usually appears during the _____ of life, and is very unusual before that age.

Lumbar spinal stenosis appears during the sixth decade of life, and is very unusual before that age. The associated pain characteristically disappears/decreases upon sitting down, increases with spine extension, and decreases with flexion. Therapy can be conservative or can include a lumbar epidural block. Surgical decompression through a laminectomy is an option when other therapies fail.

Most cases of Non ghon Urethritis that are not effectively treated with azithromycin are due to what organism and what is the tx?

M genitalium infection, which usually requires treatment with moxifloxacin.

What is the gold standard for TB infection or exclusion?

MAC culture is the GOLD standard for microbial confirmation and drug susceptibility but takes 2-6weeks NAA testing- is sensitive and specifiic and can differentiate Mtb from other microbes.

Pt presenting 1 week after post partum with RUQ epigastric pain, HTN, HA, proteinurea PE 2+ pitting edema to the knees. Lab: Hematocrit 26% Platelets 60,000/mm3 Total bilirubin 1.6 mg/dL Direct bilirubin 0.5 mg/dL ALP 220 U/L AST/ ALT: 137/149 U/L LDH, serum 436 U/L WHAT IS best next step in management of this patient?

MAG SULFATE 1ST LINE FOR SEIZURE PROPHY MAGNESIUM SULFATE

Laboratory abnormalities such as anemia and hyperbilirubinemia are common due to red blood cell lysis CAUSE by what mosquito borne protozoa?

MALARIA

Mentzer index is a value that can be used to help differentiate beta-thalassemia from other microcytic diseases and is calculated by dividing the ___ Values < ___ are suggestive of thalassemia

MCV by the erythrocyte count. Values < 13

MEN 1 VS 2

MEN 1 start with P - parathyroid, pituitary, prolactinoma (liPPPOma) MEN 2 - medullary (2p for b) Men 2b - only 1 p= pheochomo.

Cerebral venous sinus thrombosis (CVST) is a rare, potentially life-threatening condition characterized by the formation of _______ and occurs in what states?

blood clot within the dural sinuses. Most cases are associated with pregnancy, combination estrogen-progestin contraception, malignancy, infection, or head trauma. dural sinuses drain CSF and venous blood from the brain & obstruction = 1. Increased ICP leading to a gradually worsening headache MORNING and with Valsalva 2. Venous congestion of the brain, leading to focal deficits (eg, hemiparesis), seizures, and/or confusion

A third heart sound is typically caused by (hint 3ddddddd)

blood filling a dilated ventricular cavity in a patient with heart failure.

Bone turnover markers (eg, ________) correlate with the rate of bone loss and risk of fracture BUT ARE NOT USED TO DETERMINE TREATMENT FOR OSTEOPEROSIS

bone-specific alkaline phosphatase, propeptide of type I procollagen

Recurrent pneumonia in an elderly smoker may be the first manifestation of what?

bronchogenic carcinoma.

SSRI- sexual dysfunction A & M: 1. RO other cause: 2/2 depression, primary sexual d/o, stress/relationship issues, drugs. 2. Switch to non-SSRI antidepressant: __(NAME 2 DRUGS) __ Adjunctive therapy with __(NAME 2 DRUGS) __

bupropion or mirtazapine or Adjunctive therapy with sildenafil or bupropion

Patients present with saddle nose, rhinitis, hemoptysis and other upper and lower respiratory symptoms. A positive _____ is also another typical finding. Chest X-ray may show_____ nodules. Treatment is with ______. .

c-ANCA cavitary nodules cyclophosphamide WeCgener's granulomatosis necrotizing vasculitis t

The best response to the patient's seductive behavior, inappropriate familiarity, and request for special treatment is to

calmly set limits and establish clear professional boundaries. EG state that this is non-urgent and can call office and schedule apmts. Do NOT SAY GO TO ER if it is not urgent and you are off shift.

trigeminal neuralgia treatment?

carbamazepine

Hyponatremia due to inappropriate antidiuretic hormone release is a side effect of which AED?

carbamezapine

tricuspid valve regurgitation is associated with what syndrome?

carcinoid syndrome causes flushing and diarrhea

Flushing, diarrhea, and bronchospasm are the classic signs of

carcinoid syndrome, caused by metastasis of a carcinoid tumor. Carcinoid tumors usually occur in the Malignancies typically are related to the appendix, - most common site of carcinoid tumors is the terminal ileum.

Major depression is an independent risk factor for increased morbidity and mortality in what?!

cardiovascular dx (ssri)

CRAO (central retinal artery occlusion) is caused by what?

carotid artery atherosclerosis, afferent pupillary defect and cherry red spot - reduce IOP with massage and acetazolamide and urgent optho consult

if pt has inflammatory acne what medication can be used that is safe in pregnancy? What meds are category x vs cat b in pregos?

category b = likely safe c= use if benefit outweigh risk x= no (isotretinoin) prego = category B. topical erythromycin, clindamycin (inflammatory acne), or azelaic acid (comedonal acne); these are designated FDA pregnancy risk

What bacteria causes non bloody diahrrea

cdiff

invasive candidiasis causes and associations?

central venous catheter endophlabitis thus need eye exam

afib should be rate or rhythmn controlled? (hint ∆ lying with abc)

change the rate of lies! afib rate control abc = afib bb ccb

New-onset ReA in patients with a history of chlamydia infection should prompt repeat testing for

chlamydia, even in those who are otherwise asymptomatic. get urine chlamydia NOT of ulcers as ulcer on penis circinate balanitis is a manifestation of ReA

When is prostatic massage used to diagnose infection?

chronic BACTERIAL prostatits only

16Yo female with weight loss, abdominal pain, asthenia, amenorrhea, fatigue, weakness, and poor appetite. Physical examination shows muscle tenderness, hyperpigmentation on lips, decreased axillary and pubic hair has what condition?

chronic adrenal insufficiency (Addison's disease) axillary and pubic hair due to (due to decreased adrenal androgen production), increased pigmentation (due to co-secretion of adrenocorticotropic hormone [ACTH] and melanocyte-stimulating hormone).

§ . Treatment of inhalational anthrax without meningitis includes ___ plus ___.

ciprofloxacin plus clindamycin (≈ 60 days) and anthrax immunoglobulin or raxibacumab

What causes pancytopenia or transaminitis along with conjunctivitis, rash, HA, and polyarthralgia

ckngna always has polyarthralgia and treated supportively may need mtx for arthritis in 50% if persists

Class ___hypovolemic shock is defined by < 15% blood loss with < 750 ml loss of blood volume and ____ vitals/mentation.

class 1 normal vitals

antistaphylococcal and streptococcal coverage with what _____ may be added to empirically treat the most common organisms causing lymphadenitis.

clindamycin may be added to empirically treat the most common organisms causing lymphadenitis.

Pregnancy and hyperestrogenic states raise TBG but remain ----

clinically EUTHYROID! TSH is low normal and total t3 t4 is elevated from increase levels of thyroxine binding globulin

What type of acne cannot be treated with antibiotics? a. comedomal b. inflammatory c. nodular (cystic) acne

comedomal is not treated with abx but with topical retinoids (category x in preg) and salicilic, glycolic acid inflammatory: topical and oral abx for severe nodular: topical then oral abx and isotretinoin if severe

fourth heart sound is due to (4th conscious )

concentric left ventricular hypertrophy and decreased wall compliance) is more likely in someone with aortic stenosis

baby with clinical features of enlarged head, CT with hydrocephalus and diffuse calcifications, chorio, seizures, deafness and nonspecific signs of congenital infection ( eg, jaundice, growth restriction, hepatosplenomegaly, blueberry muffin spots) has what?

congenital toxo - due to mom consuming raw undercooked meat (cysts in raw meat bowl) unwashed produce etc

Diverticulitis without peritonitis or hemodynamic instability can be managed ___ Abscesses less than 4 cm can generally be treated ___

conservatively with intravenous antibiotics and NPO status. with antibiotics alone. unless they dont improve and need a percutaneous drainage, >4cm with hemodynamic instab needs surgical manage,menty

If there is disagreement about whether further care is futile, providers should involve _______ (eg in baby with pulmonary hypoplasia who has no further medical management left but parents still want trach)

consultants and the hospital ethics committee. End-of-life care is focused on minimizing discomfort, anxiety, and distress for the patient and family once efforts to cure or modify disease become futile.

awareness of negative consequences is characteristic of the what stage in which the patient thinks about changing but has not taken any steps to do so?

contemplation

What causes a Reduced FEV1/FVC and reduced DLCO

copd

internal capsule is composed largely of the what tract? which brings information from the primary motor cortex to the lower motor neurons in the spinal cord? Damage to this area typically results in ____.

corticospinal contralateral hemiparesis or hemiplegia.

What is the main difference btw croup and epiglotitis

cough

Rhythmic impulses that can be palpated on the human skull exist, and are referred to as the _____normally ____.

cranial rhythmic impulse (CRI). The CRI is 8 to 12 cycles per minute. There are factors that increase and decrease the rate and quality of the CRI.

What cancer is associated with mycotic fungoides

cutaneous t cell lympoma

What is AHA and what type of HS reaction is it classified as? Which HSR is delayed vs immediate?

cytotoxic antibody in circulation is directed at host RBC's (T2HSR) T4 delayed is contact dermatitis and is T cell mediated skin rash (eg sofia rash) T1 anaphylaxis IGE mediated with urticarial rash

What does the JVP normally do on inhale

decreases if not then it is termed kussmal sign

Hypoglycemic episodes during exercise in type 1 diabetics can be managed by___.

decreasing the insulin dose, eating before exercising, and avoiding injections of insulin in the exercising limbs. Type 1 diabetics do not have endogenous insulin production and require basal insulin by injection of twice-daily NPH or one injection of glargine insulin at bedtime.

inspiration pulls blood into the right side of the heart and results in -- closure of the pulmonic valve and -- closure of the aortic valve = of (A2) and pulmonic (P2) components SPLIT of S2

delayed closure of the pulmonic valve and earlier aortic valve; UNLESS AS, closure of the aortic valve is delayed, --> simultaneous closure of the A2 and P2 during inspiration, and is appreciated on examination as a single S2. ( eg sketchy sisters crossed out to become single)

what is associated with polyhydramnios?

diabetes duodenal atresia and TEF - unable to swallow amniotic fluid

larger sample sizes are needed when a higher level of power is required or when

differences between groups are small.

For documented clinical thrombosis associated with HIT, patients should be treated with what?

direct thrombin inhibitor at therapeutic activated partial thromboplastin time for 7 to 10 days.

Which DM adjunctive treatment is used to control hyperglycemia and rarely causes hypoglycemic episodes?

exenatide (glp 1 analog)

Pemberton's sign is the presence of _____. Caused by what?

facial plethora or neck vein distention when the arms are raised and confirms an enlarged thyroid gland as the cause of esophageal obstructive symptoms. Thyroid lymphoma usually presents as rapid enlargement of the thyroid gland in patients with Hashimoto's thyroiditis.

type 2 error (probability of ______ OR ___)

failing to reject a false null hypothesis (FALSE NEGATIVE) TYPE 2 error as probability of type 2 error increase this leads to decrease in power POWER = 1-B

Symptoms of severe anemia

fainting, chest pain, angina and may warrent PRBC otherwise pt with anemia fatigue and exertional symptoms (dyspnea) are only monitored! Those with massive bleed and normal HB may require txf as initial levels

The probability of a type I error (rejecting a true null hypothesis __]) usually increases as _____ increases.

false positive increases as sample size increases or decreases with more non-respondants

Allergic interstitial nephritis is also known as tubulointerstitial nephritis. Patients will typically have 1. PE finding 2. lab 3. UA 4. serum

fever, rash, white blood cell casts, hematuria, eosinophilia, and/or eosinophiluria. It is typically caused by infection, drugs including NSAIDs, penicillin, cephalosporins, and phenytoin.

During craniosacral ___ at the sphenobasilar synchondrosis (SBS), there is flexion of the midline bones, external rotation of paired bones, and sacral extension. During craniosacral extension at the SBS, there is ____ of the midline bones, internal rotation of the paired bones, and sacral flexion

flexion there is flexion of the midline bones, external rotation of paired bones, and sacral extension. extension-- OPPOSITE

FNA is reliable for all thyroid cancers EXCEPT? (don't fall for it!)

follicular

colles fracture occurs when? What is it associated with?

foosh dorsally angulated displacement of distal radius (dinner fork deformity) confirmed by lateral radiographs. CPS: early complication of displaced or comminuted distal radius fracture treated with too many reductions or splinted in extreme wrist flxn.

Injury/compression of the common peroneal (fibular) nerve results in decreased sensation of the anterior lateral leg and____

foot drop. This is commonly seen in cases where there is injury to the fibular head or fibular head posterior somatic dysfunction.

what level of screening are mammograms, pap smears and CSP

form of secondary prevention as pt may have precursors from disease is asymptomatic

When would you start a pt on bisphosphonates regardless of the dexa score?

fragility fracture, FRAX is >20% for major osteoporotic fractures or >3% for a hip fracture (10-year calculated probability)

Enteral feeding introduces higher pH tube feeds into the the stomach may protect

from ulcer formation and bleeding. In critically ill patients, early initiation (eg, within 72 hours of ICU admission) is also associated with lower risk of infection.

What cause of acute painless vision loss has blood and thunder on exam?

fundus with retinal hemorrhage and optic disk with edema (blood and thunder) = central reinal vein occlusion

What does cocaine do to circulation? What is the initial treatment aimed at and what is the subsequent treatment? What medication is used if initial treatment does not work? And what Med if persistent CP? What drug is contraindicated in cocaine tox. and why?

MI, AD, increases platelet activity and promotes thrombus formation -> stroke Initial treatment is via sympathetic targeting with Benzo to decrease symp. discharge and thus calm psychomotor agitation and dec cardiac work Followed by Nitro --> dec preload If HTN in still present despite Benzo may give Alpha receptor antagonist with IV phentolamine. CCB given if chest pain persists

Which benzo does not affect the respiratory drive?

MIDAZOLAM

When is the MMR vaccine administered? Is it live or killed?

MMR admin at age 1 and 4. - live attenuated vaccine and should be given even if the last vaccine had common adverse reaction ( fever, rash, LAD) - contraindicated if anaphylaxis, pregnant or attempting, or CD4 <200

Hypertrophic cardiomyopathy is the most ---- cause of SCD in young athletes in the United States; however, evidence of ---- on ECG and a small ---- cavity on echocardiogram would be expected.

MOST COMMON cause of SCD in USA - left ventricular hypertrophy on EKG - left ventricular cavity on echo

what imaging should you do for a man who is a drunken sailor with spagettie legs

MRI for spinal stenosis In some patients, gait disturbance is so prominent that they complain of having "spaghetti legs" or walking "like a drunken sailor."

What is the investigative procedure of choice for suspected lumbar stenosis?

MRI in order to see A. encroaching osteophytes at facet B. hypertrophic ligamentum flavum C. disk protrusion D. narrowing of spinal canal

After biochemical test for pheochromocytoma what should be done? Alpha before Beta?

MRI to localize if neg. will need Metiadobenzylguanadine scan to localize Prior to removal surgery give ALPHA 1ST befor beta blockade 14 days of alpha befor beta to ensure adequate complete alpha block,

Treatment for transverse Myelitis?

MRI with contrast and high dose steroids

What is the definitive gold standerard for Osteomyelitis that has the best NPV? (sensitive test with ok specificity)

MRI with contrast! changes usually evident after 5 days thus MRI is how you rule out Osteomyelitis

What drugs cause neural tube defects? What disease causes it?

MTX ANTIEPILEPTIC DM

Progressive distortion and loss of vision at center of visual field, central scotomas. If dry --> drusen

Macular degeneration

Neonatal hyperbilirubinemia management Mild? Moderate: Phototherapy Consider formula supplementation, intravenous hydration Severe Exchange transfusion

Mild (physiologic) Maximize breastfeeding (every 2-3 hours) severe is treated with Exchange transfusion likely 2/2 ABO hemolytic dx severe hyperbilirubinemia (bilirubin >20-25 mg/dL) in which bilirubin and circulating antibodies are removed and most of the infant's red blood cells are replaced with donor red blood cells.

Treatment of asymptomatic lead toxicity mild vs moderate? hint suck my d moderately (MESOhny)

Mild 5-44 μg/dL No medication Moderate 45-69 μg/dL Meso-2,3-dimercaptosuccinic acid (DMSA, succimer)

What is moderate vs mild persistent asthma? How to treat?

Mild: >2day/week, 4x/mo night awakening treat with (step 2) MODERATE: daily symptoms with saba use, weekly night awakening (step 3)

multiple symmetric and bilateral pseudofractures is known as ____ syndrome.

Milkman

Pt starts with magnetic gait that later progressed to cognition changes and psychomotor slowing; now he has a later manifestation of urinary incontinece this timeline indicates the need for what testing

Miller fischer removal of 50ml csf tap to see if this pt who has NPH is amenable to have VP shunt for treatment

WHAT syndrome, could be the reason a gallstone impacted in the cystic duct compresses the adjacent common bile duct, resulting in obstructive symptoms.

Mirizzi syndrome, a gallstone impacted in the cystic duct compresses the adjacent common bile duct, resulting in obstructive symptoms.

Which Anti-depresent stimulates appetite?

Mirtazapine along with weight gain and sedation

What type of inheritance pattern is only transmitted via females?

Mitochondrial diseases only transmitted via mother but can affect male offspring.

herbal remedies associated with increased bleeding risk include ____? hint start with g

ginkgo biloba, ginseng and garlic.

The sulfonylurea agents such as ____ have been implicated in severe, refractory hypoglycemia (especially in the setting of concomitant renal failure) and should be suspected in patients presenting with signs of hypoglycemia.

glipizide

DMD treatment

glucocorticoids

Once the diagnosis of ABPA is confirmed, WHAT preventative meds are needed and what treats it?

glucocorticoids and itraconazole are used to control inflammation and prevent irreversible damage.

ransons criteria

glucose > 220 age > 70 LDH> 400 AST >250 WBC > 18K Within 48 hours 1. calcium < 8mg% serum 2. HCT: > 10 % 3. Base deficit > 4mEq/L 4. BUN > 2mg% 5. Sequestrated fluid >6L calcium < 2

Patients present with hemoptysis. Immunofluorescence reveals an IgG and C3 in a linear pattern on glomerular basement membrane. Serum anti-glomerular basement membrane antibodies are present. It is also classified as a type II hypersensitivity reaction.

goodpastures

most common mitochondrial myopathies, age <40 p/w stroke-like episodes -(eg, hemiparesis, vision ∆), seizures, muscle weakness, hearing loss, lactic acidosis

Mitochondrial encephalomyopathy with lactic acidosis and stroke-like episodes (MELAS),

Pt admited for SBP 2/2 cirrhosis ask about 90 day mortality what measures should be used? " HINT SICBBBB WITH SBP"

Model for End-Stage Liver Disease (MELD) score determine 90-day mortality in advanced liver dx. 1. serum bilirubin, 2. INR, 3. serum creatinine, 4. nd serum sodium SICB w/ SBP (Sick b*7*) MELD calculation is used in assessing candidates for transplant livers also

Do pt with gestational diabetes mellitus (GDM) need tx after delivery?

NO D/C anti-hyperglycemic therapy but SCREEN 6-12 weeks postpartum with a 2-hour OGT due to the association between GDM and type 2 diabetes mellitus.

Can Fresh frozen plasma (FFP) indicated in someone with a UGI vericeal bleed 2/2 cirrhosis ?

NO FFP § risk of volume overload in patients with cirrhosis bc variceal bleeding due to inc. portal venous pressure and vascular ∆ not coagulopathy. FFP indicated if INR is >/2 in coagulopathy (eg vK deficiency)

Can you give heparin in HIT2 in the future once antibodies are undetectable in 3 mo?

NO hit2 is contraindication for any heparin product for life including LMWH

Is ASCUS considered evidence of metaplasia at the transformational zone?

NO metaplasia at transformational zone is normal and ASCUS is evidence of a squamous cell with atypical features (IE precancerous change).

Does a low probability V/Q scan rule out acute PE if the person had surgery recently, is now hypoxemic, tachycardic, lack of more probable dx and xray with small pleural effusion,

NO wells score >4 needs a NORMAL VQ scan can rule out PE. CXR with small pleural effusion is commonly due to inflammation from PULMONARY infarct.

If a patient has ASCUS or Low grade squamous intraepithelial lesions should a colpo be done?

NO, instead use HPV co testing in order to determine if pt is at high risk and if so then that warrents a colpo.

NPH is _____-acting insulin with peak actions. hint nph is like addy

NPH is intermediate acting with peak after 4-6hours of injection, effects last 16 to 18 hours. Reducing the morning NPH insulin dose is likely to reduce this patient's hypoglycemic symptoms during the evening hours.

why is Sodium bicarbonate given for TCA overdose? Is it to alkalinize the urine?

No that is for salicylate tox. TCAs have a large volume of distribution and are not effectively eliminated through alkalinization of the urine na bicarb is given to treat arrythmia !

Would metoclopramide (reglan_) be useful in someone with dumping syndrome with neurovegatative symptoms of dizziness and dyspnea, diaphoresis, N, V.

No treatment is aimed at decreasing speed of food passage into duodenum thus need HIGH PROTEIN low carb diet. Reglan/MCP causes lower esophageal sphincter contraction.

Is Dexa sufficient to treat secondary hypoadrenalism?

No, dexamethasone has little mineralocorticoid activity, fluid and electrolyte replacement is also needed

What is the treatment in someone with ammenorrhea mild inc prolactin (50) and increase A subunit (LH FSH IGF1 low normal and normal ) Would octreotide work? Explain the difference in treatment for prolactinoma?

Non functioning pituitary adenoma is treated with trans-sphenoidal surgery 1st line Octreotide is a somatostatin analog works on pituitary to block GH in acromegally (considered functioning adenoma) prlctma treatment is for dopaminergic receptor agonists like CABERGOLINE

_____ confidence intervals always imply a statistically significant difference between groups. while, the opposite is not necessarily true.

Nonoverlapping always imply a statistically significant difference between groups. while, the opposite is not necessarily true.; overlap - may or may not be sig.

What lab findings are found in fibromyalgia? What test should be given to RO common differentials? How is it treated?

Normal acute phase reactants (eg ESR, CRP, TSH) CBC, ESR, TSH should be ordered Treatment: progressive exercise, meds: TCA, cyclic skeletal mm relaxants, SNRI

Is the presence of columnar cells and squamous cells in the transformational zone considered metaplasia or dysplasia? And is this normal?

Normal metaplasia; - columnar cells become squamous cells in the transformational zone which is a NORMAL CHANGE and required to have both cell types to be deemed a sufficient sample.

What does serum uric acid level correlated with?

Nothing it can be normal in acute attacks and shouldn't guide MGMT

What TB test remains positive after treatment?

Nucleic acid amplification can stay positive after tx. NAA: High specificity, higher sensitivity than smear microscopy. DX in 1-2 days & differentiate from non-TB mycobacteria

____ can result in poor suckling of the newborn.

OA compression (or condylar compression) and manifests and decreased cranial rhymic impulse (CRI) and can be nearly absent in severe OA compression.

vaginal bleeding at >20 weeks gestation and no previous anatomy ultrasound require _____ to exclude placenta previa. Can a digital or speculum exam be done as well?

ONly a trans-vaginal ultrasound and speculum can be done digital is contraindicated due to endo-cervical canal disturbing the placenta and results in hemorrhage.

STATS: What does an odds ratio explain and how is it calculated?

OR tells you the measure of association btw an exposure and outcome. OR= AD/BC based upon the 2/2 contingency table

Name Group of Pulm hypertension that is treated with Sildenafil and why?

group 1 pulmonary hypertension (PH): idiopathic pulmonary arterial hypertension. give viagra to the idiotttt Sildenafil is a phosphodiesterase type 5 inhibitor that prolongs the vasodilatory effects of nitric oxide. It has been shown to improve pulmonary hemodynamics and exercise capacity in patients with

what should be done in CDAD symptom reoccurance s/p vanco and retesting is positive?

Observe with no further interventions for C difficile at present Stool testing remains positive for days to weeks after treatment (particularly PCR, which tests for the presence of the toxin-producing gene, not the toxin itself). - positive PCR alone is insufficient to diagnose CDAD recurrence in those with only 1 stool (minimum 3 /d)--> just observed for now.

What sign involves laying pt on the back with hip and knee flexed fully and is positive if pain elicited with examiner rotates the hip by moving the patient's ankle away from the patient's body and allowing the knee to move inward?

Obturator sign, also known as the Cope sign is an indicator of irritation of the obturator internus muscle and is used to test for acute appendicitis.

What is the use for Oral D-penicillamine?

Oral D penicillamine is the primary chelating treatment for copper removal in Wilson disease. NOT used in lead toxicity due to serious adverse effects (eg, leukopenia, thrombocytopenia, hepatotoxicity).

What is the most sensitive test for DM2?

Oral glucose tolerance test.

How is cdiff initially treated? When is CDI considered severe?

Oral vanco or oral fidaxomicin Severe CDI is with WBC ≥15k and/or serum crt. >1.5 or fulminant colitis (hypotension, ileus, toxic megacolon)

The --- the most common site of orbital fractures secondary to blunt trauma and can result in diplopia due to a trapped--- muscle.

Orbital floor is fractured most commonly. - inferior rectus muscle is supplied by CN III (oculomotor) - diplopia and - sensory loss from V2 branch of CN V (courses orbital floor)

______ is the most common primary malignant bone tumor, which will present with deep bone pain and radiographic images demonstrating a sunburst pattern, codman triangle, or cortical destruction.

Osteosarcoma

What does an overlap in 2 different confidence interval signify if neither crosses the null but include an overlapping range?

Overlapping CI means that there may or may not be a difference btw the two groups on the outcome measured If they do not overlap that means there is a significant difference btw both CI

extra articular manifestations of ReA reactive arthritis include what symptoms and how soon after infection?

P/w in 1-4w after initial infection - uveitis - Circinate balanitis of penis (painless lesion on penis that persists for months no LAD as in chlamydia) - asymetric oligoarthritis - dactylitis - urethritis - oral ulcers

In someone with ED and history of dm/ smoking and HLD what diagnosis does he have and what should be RO prior to treating his sexual disorder?

PAD (risk factor smoke artheroscleroisis) ABI < .9 highly s/s for PAD ABI= SBP tibial pulses (or DP) divided by SBP brachial PAD IS HIGHLY RELATED TO HEART CAD RISK THUS MUST GET A STRESS TEST PRIOR TO GIVING PDE5 INHIBITOR TREATMENT FOR ED!

Normal testing but decreased diffusing capacity of the lungs for carbon monoxide (DLCO)

PAH

Thrombolytic therapy drugs for stemi in rural area?

PCI within 120 minutes is not possible, and fibrinolytic therapy (eg, tenecteplase, alteplase, reteplase) is indicated within 30 minutes of hospital arrival.

What is the management of PEA vs Vtach or Vfib

PEA is neither defibrillated nor cardioverted but instead should give chest compression (q 2m), ventilate, epi q 3 min until ROSC achieved

The postierior interosseous nerve branches off what? and what motor function does it serve?

PIN bbr of radial nerve supplies motor for extenors of wrist - entraped at arcade of frohse (part of supinators) --> FINGER DROP (unable to extend thumb and IP joint) radial wrist deviation on extension

Common causes of postoperative hypoxemia that occur after 3 days post op?

PNA: fever and elevated white count Atelectasis: linear opacity and treat w/ chest physiotherapy and frequent oral suctioning (occurs commonly s/p intra-abdominal surgery) PE: uncommon before 3 days and is associated with hypoxemia despite o2 (+ chest pain)

What is the most common cause of post-partum maternal hemorrhage and if there are fist-sized clots in vagina

PPH is ≥500 ml of blood loss and commonly due to: Uterine atony: should be treated with bimanual uterine massage + oxytocin

What is the formula for PPV and how can it be calculated if only sensitivity and specificity is known?

PPV of a diagnostic test is the proportion of patients with a positive test result who actually have the disease. PPV = TP / (TP + FP), where TP denotes true positives and FP denotes false positives. PPV can be calculated from a contingency table.

If pt with venous air embolism is still coding despite aspirating air thru cath. laying pt on left (trandelenberg THEM AKA lay down) and intubating what should be done?

PRAY AS YOU Insertion of a needle into the right 4TH intercostal space with aspiration of blood through the right ventricle should only be performed as a last-resort therapy for venous air embolism

What should birth control should be given to someone with multiple VENOUS AND ARTERIAL THROMBOSIS and + anticardiolipin ab?

PROGESTERONE IUD- better because dec bleed risk and no estrogen to promote thrombus forming not copper iud because antiphospholipid syndrome can predispose to bleed

Drugs OTC supplements

important

When would osteopenia need bisphosphonate treatment

in high-risk instances with a high FRAX score hip fracture risk ≥3 and combined fracture risk ≥20 percent warrants bisphosphonate treatment

STATS: when would a length time bias be a concern?

in slow-progressing benign disease eg prostate cancer These slowly progressive cases also have lower mortality, which may give the impression that screening improves disease survival (but in actuality, it only identifies more benign cases). length time = more benign

Anal abscesses should be treated with prompt

incision and drainage! (later still at risk for fistulae) Antibiotics in those w/ diabetes mellitus, immunosuppression, extensive cellulitis, or valvular heart disease.

cliliacs disease manifest as what extra-intestinal ways

including anemia, osteoporosis, and dermatitis herpetiformis. Depression is more common among nearly all persons with chronic disease.

COMLEX Q what 2 types of abortion have dilated cervix (hint = I)

incomplete and inevitable (I - dilated) 1. Incomplete abortion: some products of conception are passed in the setting of an ultrasound nonviable preg. An open cervical os with products of conception passing through the os is often found on exam. Treatment is expectant management, medical management with misoprostol/mifepristone, or surgical management with a dilation and curettage. 2. inevitable abortion: cervix has dilated during the first 20 weeks of pregnancy in the setting of vaginal bleeding and/or uterine cramping. A viable pregnancy is seen on ultrasound.

Iron deficiency anemia in young children when treated empirically will have increase in what lab value 1-2 weeks with supplementation?

increased reticulocyte followed by H/h rise and lastly ferritin which is FE stores

§ injury to the orbital floor can result in diplopia due to a trapped .

inferior rectus muscle CNIII

How is cdiff treated initially? If recurrent or fulminant (eg with hypotension/shock, ileus, megacolon?)

inital VANCO 125 PO QID or Fidaxomicin 200mg PO BID for 10 days recurrence: vanco po in pulse/taper vs fidaxomicin Multiple episodes: vanco PO followed by rifazimin Fulminant: metronidazole IV pulse high dose vanco PO or pr if ileus is present - surgical evaluation

Pneumothorax is also a potential complication of_____ (2 types) ____venous cannulation and p/w sudden-onset chest pain (90%) and SOB (80%). What should always be order to evaluate for pneumothorax?

internal jugular and subclavian central venous cannulation! portable chest film following central line placement to evaluate for pneumothorax and confirm catheter placement.

Developing rapport and trust is important in tx of the adolescent. When a parent raises concerns, assessment should include ______.

interviewing the patient alone.

Hip fractures due to falls are common in the elderly and are classified as either _____ or ______.

intracapsular (femoral head and neck) extracapsular (intertrochanteric or subtrochanteric). In elderly patients who are stable and ambulatory prior to the fracture, surgery within 48 hours is associated with lower mortality and a lower risk of pressure ulcers and pneumonia

How can you differentiate a baby with maternal rubella infection vs toxo infection?

intracranial calcification not seen in rubella RUBELLA infection is more likely to have SYMPTOMS: fever with maculopaular rash congenital rubella: manifests as congenital heart defects, eye abnormalities, and hearing impairment. Intracranial calcifications are not seen.

A commonly used muscarinic antagonist is _______, which causes bronchodilation and is typically used in patients with chronic obstructive pulmonary disease.

ipratropium

Primary adrenal insufficiency p/w 1. __ cortisol, 2. ___ ACTH, 3. ____ aldosterone, 4. ___ renin, 5. what electrolyte change of K, NA, SUGAR ? 6. hypotension

low cortisol, high ACTH, low aldosterone, high renin, hyperkalemia, hyponatremia, hypoglycemia and hypotension.

low erythropoietin epo level = _____ , whereas a high erythropoietin level suggests a secondary cause of polycythemia, such as _____ .

low epo = polycythemia vera high epo = chronic hypoxia or a hormone-producing neoplasm (commonly renal cell carcinoma).

Patients with DVT or PE who do not have cancer and are at ___ risk for bleeding complications are typically treated with_____months anticoagulation with an

low risk for bleeding complications are typically treated with >3 months anticoagulation with an oral factor Xa inhibitor such as rivaroxaban.

For patients with an underlying malignancy who develop DVT or PE what form of anticoagulation is most efficacious?

low-molecular-weight heparin (LMWH) is considered more efficacious than factor Xa inhibitors.

what cancers are common in Dermatomyositis? LOC dem up

lung ovary and colon

Goodpasture's syndrome affects the what 2 parts of the body?

lungs and kidneys which leads to nephritic syndrome.

rash in non-imunized child with rubella (think of starry night central star in vincent paint)

macular papular rash sreads centrally out caudally in child and adult who is not immunized

Hard cervical nodes, especially in older patients and in smokers, are suggestive of a what? And who should they be referred to?

malignancy. These patients should be referred to an otolaryngologist for fiberoptic examination of the oropharynx to look for a primary malignancy.

Vaginal delivery is indicated for dichorionic/diamniotic twins in a vertex/vertex presentation. s/p twin A (the first twin), twin B is in vertex presentation and has a normal fetal heart tracing, labor is managed -----?

managed expectantly.

What has supratemporal lense dislocation and which condition has inferior lense dislocation

marfans: supratemporal (due to zonular fibers malformation, also at high risk for aortic root disease and subsequent dissection) Homocystineuria: inferior

RBBB with right ventricular hypertrophy which often accompanies the S1Q3T3 pattern seen indicates what?

massive pulmonary embolization. This is typical in patients with right heart failure.

What vaccine can NOT be given during preg

measles-mumps-rubella MMR live attenuated NOT GIVEN DURING PREGNANCY

What is the best diagnostic (confirmatory) tool for someone with recurrent PNA, and what if is the next best step for someone with PNA reoccurrence (aka non confirmatory test)?

primary diagnostic tool Flexible bronchoscopy - HRCT scan is an alternative, but it does not provide confirmatory information). *The best diagnostic test for endobronchial obstructive lesions is flexible bronchoscopy. If the question had asked for the next best step in management, then the answer would have been CT scan.

"Salt and pepper" sign in skull; sub-periosteal bone resorption

primary hyperparathyroidism Serum Calcium high Serum Phosphate low Alkaline Phosphate high Parathyroid Hormone high

2 weeks of progressive motor weakness that can lead to paralysis 2-4 weeks of plateaued symptoms Slow, spontaneous recovery over months

progression of Guillain - Barré

3 part are the treatment for thyroid storm (hint PBS).

propylthiouracil Beta-blockade steroids (hint PBS).

The mainstay of treatment in anorexia nervosa is _____ and nutritional rehabilitation. Indications for hospitalization include (4 signs).

psychotherapy hospitalize if: 1. orthostatic hypotension, 2. syncope, bradycardia, and electrolyte abnormalities

uti in pt who is <24 months old needs what?

pt age <24 months with a febrile UTI should undergo renal and bladder ultrasound to evaluate for hydronephrosis and ureteral dilation to RO an underlying anatomic abnormality. Ultrasound done AFTER uti treatment to prevent FP from renal inflammation during infection.

cluster HA symproms (think of periocular injection)

men episodic headaches 1-8 x/day over weeks. 1. pain orbital, supraorbital, or temporal 2. unilateral 3. severe. Patients usually have restless agitation and autonomic symptoms of the ipsilateral trigeminal branch such as lacrimation, conjunctival injection, rhinorrhea, and nasal congestion.

First-line treatment for moderate asymptomatic toxicity (45-69 µg/dL) is

meso-2,3-dimercaptosuccinic acid (DMSA, succimer), PO. -side effects are few (eg, rash, neutropenia).

STATS: Tests for heterogeneity are useful when performing a _____.

meta analysis or comparing different trials

what are the expected bicarb levels in metabolic alkalosis vs acidosis

metabolic acidosis = pH of <7.35 on ABG and a low HCO3 level. - diarrhea, RTA, hyperchloremia, renal failure, lactic acidosis, and ketoacidosis. metabolic alkalosis is defined by a pH of >7.45 on ABG with ↑ HCO3. - vomiting, diuretic, antacid, and hyperaldosteronism.

primary acid-base abnormality associated with vomiting is ?

metabolic alkalosis.

Patients who do not have alcohol or gallstones as a cause of acute pancreatitis should be evaluated for what diseases? will they have a gap?

metabolic causes of the disease, including hypercalcemia and hypertriglyceridemia. HTG has a anion gap

Metabolic abnormality in SGLT2 (3 things)

metabolic: HLD Hypokalemia euglycemic dka randomz: amputation Low trauma fracture

Chlorhexidine bathing can help prevent recurrences of

methicillin-resistant Staphylococcus aureus

murmur of mild to moderate aortic stenosis

mild aortic stenosis: Early peaking of an AS murmur b/c relatively low left ventricular pressure (attained early in ventricular systole) is needed to overcome the valvular stenosis.

What is VPA used for?

mood disorder seizure disorder

Bacterial conjunctivitis comonly caused by what pathogen and what is the treatment? Preferred agent in contact lens wearers: fluoroquinolone drops

most common causes of bacterial conjunctivitis are Staphylococcus aureus, Streptococcus pneumoniae, Moraxella catarrhalis, and Haemophilus influenzae. Erythromycin ointment Polymyxin-trimethoprim drops (polytrim) Azithromycin drops (pea eye- unilateral

what is the typical granuloma in sarcoid and how can you tell if it is TB

non caseating bilateral granuloma while TB is often unilateral granuloma caseating (takes 2-10 weeks to calcify) if does not calcify they get miliary TB and die

What lymphoma are SLE pt at risk of ?

non hodgekins diffuse large B cell lymphoma

When is skeletal survey indicated in peds?

non-accidental trauma - abuse

Renal tubular acidosis causes WHAT TYPE OF acidosis?

non-anion gap (EG 10-14)

Scaphoid fractures are most commonly complicated by

nonunion and avascular necrosis. proximal fractures require longer immobilization (up to 12 weeks) for adequate healing.

HCW exposed to needle with HEPb who had adquate titer after vaccine but now has drop of titer needs what PEP?

nothing

Besides nephropathy and retinopathy what role does stict glycemic control play in those with DM (t1 and t2?

nothing, glycemic control only helps the microvascular complication but not macro (eg MI or stroke), or mortality

Flexible sigmoidoscopy can be offered to average risk pt and if normal repeat testing should be done how often?

q 5 years q10 years if done with annual fecal immunochemical test

when should hysterectomy be considered?

radical hysterectomy is needed in someone with abnormal pap and colpo evidencing early stage disease! Advance dx treated with chemoradiation.

What treatment is needed for displaced fractures of the distal humerus?

reduction (closed or open) with percutaneous pinning.

pud treatment that is only 10 days

omeprazole, amoxicillin, and clarithromycin (OAC) for 10 days

What is the root cause analysis?

origin and progression Data collection, ID of causal factors, ID root causes behind causal factors and recs. for preventing recurrence. 1. collect data 2. make causal flow chart 3. ID root cause 4. generate recs and implement changes 5. measure success of changes

orthostatic hypotension—systolic blood pressure drop >20 mm Hg, and increase in pulse when he moved from the decubitus to the standing position—associated with dizziness. And classically associated with the use of what 3 drugs?

orthostatic hypotension findings are classically associated with the use of diuretics, alpha blockers, or nitrates. DAN B fallin-all-over the damn place

During bone turnover, the decreased mineralization causes focal accumulation of new osteoid. manifests as _____ on xray known as "____";

osteomalacia pseudofractures radiolucent lines with sclerotic borders

Primary biliary cholangitis is associated with an increased risk for _______and warrants screening with ________.

osteopenia and osteoporosis, bone densometry

how does the penetrating artery become predisposed to form diverticula?

outpouching of colon wall at weakest point where vasa recta enters circular muscle layer of colon. mucosa and submucosa herniate through muscular layer and penetrating artery becomes more exposed and without muscle layer protection the mucosal layer erodes

What kidney disease can occur after impetigo?

pSGS can occur 3 weeks after BOTH strep infection of throat and skin

pyoderma gangrenosum is what?

painful associated with IBD and arthritidies

McNemar test compares the difference between 2

paired proportions; patients serve as their own control (eg, success/failure before and after treatment in the same subjects).

nondiabetic patients with moderate-to-severe hypothermia are hyperglycemic bc?

patients with moderate-to-severe hypothermia are hyperglycemic as insulin action stops at <30 C [86 F]. Warming these patients typically results in lowering of serum glucose. thus give iv insulin

How often does one need a dexa for post-menopausal bone loss eval?

patients with postmenopausal bone loss, a 2-year interval is generally appropriate.

What 3 skin conditions are seen in HEP C?

pct porphyria cutanea tarda Cutaneous leukocytoclastic vasculitis (palpable purpura) 2/2 cryoglobulinemia Lichen planus often is associated with advanced liver disease due to hepatitis C virus.

positive rapid ag test warrants treatment with what med and length of time what if anaphylactic rxn to 1st line ?

pen V or amx 10 days or ceflex 10 worse rxn azt x 5day

Postmenopausal osteoporosis is diagnosed as Osteopenia: T-score on DXA between Osteoporosis: T-score ____or confirmed _____.

penia: -1.0 to -2.5 porosis: ≤-2.5 or confirmed osteoperotic fracture !

what murmur should be expected in constrictive pericarditis

pericardial knock mid-diastolic

Patients who receive massive transfusions or those with liver or renal failure (or hypothermia, shock) can fail to adequately clear citrate how does this manifest and what is tx?

perioral tingling and numbness, twitching. -citrate chelates calcium thus treatment = IV calcium gluconate/chloride (may need few doses)

what type of parathyroid problem has high phosphate

secondary

Head imaging is needed when a neurologic problem is suggested by the history or physical examination. If there is evidence of ---, a head CT scan without contrast should be performed.

significant head trauma

What is the most important risk for AAA expansion? how is it diagnosed?

smoking cessation; dx 5.5cm on ultrasound if asymptomatic ct abdomen if symptomatic or confirmed AAA

Common risk factors for atherosclerosis are:

smoking, hypertension (HTN), diabetes, family history, high cholesterol, male gender, increased age, inactivity, and poor diet.

diplopia, restricted extra-ocular motility, painless proptosis, restrictive ophthalmoparesis and eyelid retraction has what somatic dysfnx

sphenoid and occiput (CN 2 exits optic canal) CN 346 via Superior orbital fissure thus related to sphenoid and temporal bones in thyroid eye disease initially evaluated by tsh

what develops in up to 25% of patients with Pancoast tumor during the course of the disease. Early recognition and appropriate therapy are imperative to preserve function and patient autonomy.

spinal cord compression

CML abdominal pain due to what nerve irritation

splenomegaly causing phrenic nerve irritation aka Kehr's sign with referred L shoulder pain (Philadelphia chromosome)

Stroke patients are discharged w _____, which are effective in preventing secondary stroke. Statins are also thought to have acute neuroprotective effects.

statins - preventing secondary stroke. - acute neuroprotective effects.

what should be stoped or corrected in suspected ACS

stop NSAID Benzo if cocain ACS (NO Beta blocker!) correct electrolytes

STATS: SMR standaridzed mortality is the ratio of observed to expected number of deaths thus what is SMR =1 indicate

study group death is what is expected SMR <1 is less than expected etc

STATS: Kaplan Meier survival curve is commonly used graphical representation of the probabilities that ______.

subjects survive in a study w/o an event. Comparison done by survival analysis. With x being time and y being cumulative survival.

Secondary Hyperparathyroidism

subperiosteal bone reaction Serum Calcium - low Serum Phosphate high Alkaline Phosphate high Parathyroid Hormone high

what nerve can be damaged in distal fib fractures and how does it manefest

superficial peronal --> loss of sensation over dorsum of foot

Following parathyroidectomy, serum calcium levels can fall (see below) and produce symptoms of hypocalcemia such as perioral numbness, muscle cramps, carpopedal spasm, positive Chvostek sign (ipsilateral contraction of facial muscles on tapping the angle of the jaw) and positive Trousseau's sign (rapid development of carpopedal spasm on occlusion of blood supply to the upper extremity). Severe hypocalcemia can cause mental status changes and seizures.

suppression is transient and most patients recover in few days.

The ____ nerve innervates the supraspinatus and infraspinatus muscles.

suprascapular

the empty can or Jobe's test and is considered the gold standard test for

supraspinatus function. The position described in the stem makes the supraspinatus the primary muscle opposing the downward force of the examiner. If there is only pain with this test and no weakness, that is indicative of tendinopathy rather than tear.

The long-term management of pheochromocytoma

surgical and adrenalectomy is the treatment of choice.

Rhinocerebral mucormycosis is most commonly seen in patients with diabetic ketoacidosis. Treatment requires

surgical debridement and intravenous liposomal amphotericin B.

§ When would succimer and EDTA be indicated ?

symptomatic lead poisoning

NSAIDs reduce prostaglandin

synthesis

spinal level of sternal notch?

t2

nipple dermatome

t4

VSD murmur location (hint: versed)

take some versed for a holographic thrill - 3rd 4th ICS VSD holosystolic murmur with palpable thrill in 3rd and 4th ICS

What is the goal BS in someone who has IDDM?

target pre-meal blood sugar goal in those with diabetes is between 80 and 120 mg/dL. If the pre-meal blood sugar is consistently less than 80 or more than 140 mg/dL gotta do something

___ number one cause of death in world ?

tb lol

URINARY SCHISTOSOMIASIS RISK FOR WHAT ?

terminal hematuria RENAL CARCINOMA CAUSES HEMATURIA SNAILS IN SKETCHY SWORDFISH SAHARAN AFRICA dx with urine sediment microscopy tx praziquant.

The goals include preventing further physical deterioration and maximizing quality of life. Other examples of ___ interventions include:*Cardiac or stroke rehabilitation programs*Chronic pain management programs*Patient support groups

tertiary

what level of prevention would explain the use of beta blockers to prevent remodeling after MI?

tertiary prevention as pt is symptomatic disease is present (cad) thus improve morbidity from disease (goal)

Abdominal epigastric discomfort in older patient should prompt evaluation with what tests?

tests: 1. RCA Myocardial Ischemia (Inferior) - EKG! 2. Mesenteric ischemia- lactic acid and VBG (venous blood gas) 3. DKA- VBG, beta hydroxybutyrate, BMP EKG! CBC- r/o anemia, wbc lactic acid and venous blood gas VBG - r/o mesenteric ischemia

What factors affect statistical power besides sample size?

the MAGNITUDE of the effect the study is looking to check - eg 8mmhg detection vs 5mmhg in bp requires smaller size of cohort since you are detecting a larger change.

low voltage ecg +afib 1 month after a bypass graft occurs in male his echo shows increased sized atria and septal defect, mild pericardial thickening jvp and hepatojugular reflux is noted on exam but his lungs are CLEAR

the pericardial thickening suggestive of pericardial calcification, JVP, edema, hepato-jug. reflux in someone with kussmal signs and no signs of stemi = R HEART FAILURE FROM PERICARDIAL CALCIFICAITONS.

Exchange transfusion, is indicated for neonates with what disease?

the removal of bilirubin and antibodies with replacement of red blood cells, severe hyperbilirubinemia (>20-25 mg/dL), worsening hyperbilirubinemia on phototherapy, or signs of bilirubin-induced neurologic dysfunction.

Wet Beriberi (eg, dilated cardiomyopathy, polyneuropathy) and Wernicke-Korsakoff syndrome (ie, neurocognitive impairment, oculomotor dysfunction, ataxia, encephalopathy, amnesia) - can be seen with what deficiency?

thiamine (vitamin B1) 1 brain 1 heart 1 cns

WHAT CONSENT needed in epiglottitis looking pt but hemodynamically stable kid with grandma and mom is at work

this is urgent thus call the mom to consent Informed consent from a parent or legal guardian should be obtained prior to non-emergency medical interventions in a minor. - routine care, a written consent by proxy should be signed by the parent or legal guardian specifying who may consent to the child's medical care. Verbal consent is often accepted in cases of urgent intervention, as in this case of a child with a peritonsillar abscess. Because this patient is hemodynamically stable and not in imminent danger, proceeding without consent is inappropriate; the mother should be contacted and legal consent should be obtained

petechia with no certain cause should raise concern for underlying ____

thrombocytopenia PETECHIA <.3MM

What is Budd-Chiari syndrome what would the SAAG be?

thrombosis of hepatic veins and supra-hepatic IVC post sinusoidal cause of portal hypertension SAAG >1.1

What causes elevated serum calcitonin?

thyroid nodule >1cm get FNA is only test that can ID if malignant (radioactive and US cannot tell you for sure) Once a solitary thyroid nodule is detected on Exam (aka >1cm pinky), FNA is the initial study of choice. radionuclide scan is done if fna is indeterminant!

EHRLICHIOSIS

tick-borne bacterial illness characterized by fever, nonspecific symptoms, nausea, vomiting, and cough. laboratory = leukopenia, thrombocytopenia, elevated aminotransferases).

What condition that affects the tracts across the horizontal aspect of the spinal cord at T-spine? AND occurs after viral infections. Progression is usually rapid.

transverse Myelitis can occur after immunosuppresion and presents with - urinary RETENSION - upward babinsky - thoracic pain in a band fashion -motor sensory weakness below lesion

Baby with chlamydia

treat with oral macrolyde

Uncomplicated cystitis in non pregnant

treated without testing with TMPSMX X3DAYS; nitrofurtoin x 5day. Durging pregnancy needs testing and treat with nitro?

professional ethics guidelines recommend what and how can you respond to a friend who wants drugs?

treatment of friends should be restricted to limited care in emergency situations when no other physician is available. "I would like to help you, but I am uncomfortable prescribing for someone I am not treating."

what 2 things cause ldh >1000

ttp and b12 deficit

Patient who presents with painful goiter following a viral sickness warrants what type of treatment?

tx nsaids! Subacute thyroiditis may be responsible for 15-20% of patients presenting with thyrotoxicosis and 10% of patients presenting with hypothyroidism.

migraine treatment during pregnancy?

tylenol 1st asa and nsaids if failed 1st line and pt in 2nd trimester sumatriptan (category c in prego) last line if failure of all other measures ergot = category x!

what is the main difference between a type 1, type 3 block vs type 2 block?

type 1 = prolonged pr >200msec, 5 tiny boxes that leg but all qsr, conducted type,3 has regular qrs but av node dissociation; both have NO DROPPED QRS. TYPE 2 BLOCK mobitz wenchybach has prolonged pr that legthens and eventualy drops a qrs. type 2 block 2 randomly drops the qrs but pr interval =stable

Anti-smooth-muscle antibodies are associated with ______.

type 1 autoimmune hepatitis, which typically causes elevation of hepatic transaminases. Alkaline phosphatase is usually normal or only mildly elevated.

Pt w/ a 1w of fever (often with temperature-pulse dissociation), --> abdominal pain and cutaneous "rose spots." By week 3, hepatosplenomegaly, intestinal bleeding, and intestinal perforation has what bacterial illness?

typhoid from salmonella

Wolff-Parkinson-White WPW syndrome is an --- cause of SCD. Resting ECG typically shows a --- PR interval with slurred upstroke and --- of the QRS complex.

uncommon cause of SCD SHORT PR WIDE QRS

OMM: What is the main motion of c2-c4 vs c5-c7 and what sides do sidebending and rotation occur on ?

upper c spine is rotation and other motion is same sides lower main motion is SB which other on same side

Clinical suspicion of necrotizing fasciitis warrants ______what treatment____ to prevent morbidity and mortality.

urgent, aggressive surgical exploration and debridement of the involved tissue AND IV BROAD spec antibiotic therapy and hemodynamic support should be provided. DO NOT wait for culture sensitivity labs etc tissue samples are obtained during surgery anyway to determine culture and sensitivity.

When is GGT elevated? What can GGT tell you about about alkaline phosphate

usually caused by cholestasis it is used to tell you if an elevated alk phos is due to bone or liver is GGT and alk phos are both high the liver is culprit

Factor V Leiden heterozygosity is associated with increased risk for ____ clots? and does not usually cause _____ thrombosis or emboli

venous thrombosis - deep venous - cerebral - mesenteric venous thrombosis NOT DEEP VEIN THROMBOSIS (DVT CAN LEAD TO PE WHICH IS TECHNICALLY ARTERIAL CLOT!?) Pulmonary embolism occurs when a clump of material, most often a blood clot, gets wedged into an artery in your lungs. These blood clots most commonly come from the deep veins of your legs, a condition known as deep vein thrombosis (DVT). arterial thrombosis/ emboli more likely with left atrial myxoma

what 3 liver pathology presents with ast in 1000s

viral, ischemic and drug induced liver injury

What vitamin deficit resembles scurvy?

vitamin C deficiency. Vitamin C deficiency results in scurvy, which is a condition associated with poor healing, hypertrophic bleeding gums, easy bruising, and deficient osteoid (mimicking rickets).

How does the fundus appear in retinal detachment?

vitreous hemorrhage and marked elevation of retina --> surgery asap!

What is the w/u and management for dyspepsia in <60 and ≥ 60 ?

w/u Workup Age ≥60: Upper endoscopy Age <60:Testing and treatment for H pyloriUpper endoscopy in high-risk patients (eg, overt GI bleeding, significant weight loss, >1 alarm symptom)

What is the J point on an EKG?

where QRS complex meets the ST segment

how is the diagnosis of acute pancreatitis confirmed initially in the setting of characteristic acute epigastric pain?

with an elevated level of serum amylase or lipase Advanced abdominal imaging (eg, MRI, CT scan) should be pursued if basic laboratory results and ultrasound fail to provide a diagnosis.

What would be expected in someone with subclinical hyperthyroidism s/p heart cath

worsening of hyperthyroidism due to iodinated dye administration!

STATS: What is an observational bias

wrong variable classification

what do you need to do for hypothyroid pt who is on levothyroxine that discovers she is pregnant?

you must adjust the dose up by 30% and check monthly to make sure you are keeping the TSH in low normal range

Hip resurfacing arthroplasty consists of trimming the femoral head and capping it with a metal covering as well as replacing damaged bone with a metal shell. It is most successful in -----?

young, active men. However, long-term effectiveness is less than that of joint replacement.

NORMAL CARDIAC ENZ V1-V4 ELEVATED, PULM CRACK 5W S/P STENT AND ON DAPT

persistent ST-segment elevations in the anterior leads makes ventricular aneurysm the most likely diagnosis. - confirmed with a chest radiograph: bulge in the left ventricle, and echocardiogram. - caused by replacement of the infarcted tissue with scar tissue, = decreases contractility = bulges outward, = systolic heart failure. -common weeks after an anterior wall infarct. Treatment is based on severity MILD: untreated with optimization of medical therapy blood pressure control. Large aneurysms aka cause hemodynamic compromise: surgically resected.

Difference btw Petechiae, Purpura, Ecchymoses? veronica

petechiae: .1mm - 3mm purpura: 3mm - 1cm Ecchymosis > 1cm

Which phase of CML is marked by neutrophil differentation impairment and difficulty in controlling leukocytes?

phase 2 (accelerated phase

Polyethylene glycol is used to remove

phenols and cresols from the skin in a 50:50 ratio.

Refeeding syndrome needs replacement of what nutrients (4)?

phosphate thiamine magnesium potassium

Mom with healthy baby >500cc blood loss in delivery (1k in c-section) failure to lactate and amenorrhea 6mo after delivery has what and needs what treatment?

pituitary gland necrosis (aka Sheehan syndrome, or postpartum hypopituitarism) due to postpartum hemorrhage and can occur late after delivery (aka months years etc) Needs life-long hormone replaced.

What placental anomaly is an absolute contraindication to C-section?

placenta previa

In someone with confirmed medullary thyroid cancer. Serum calcitonin is 300 pg/mL (normal 0-10 pg/mL). Genetic studies show a mutation in the RET proto-oncogene. Which of the following is the best next step in the management of this patient?

plasma free metanephrine always to see if they have pheochromocytoma

Developmental dysplasia of the hip

positive Barlow and Ortolani tests with hip dislocation, limited hip abduction, and asymmetric gluteal/inguinal folds.

The common peroneal (fibular) nerve runs ____ fibular head.

posterior the proximal Any dysfunction, especially posterior fibular head, will cause nerve entrapment.

An early pregnancy of undetermined location occurs when a pregnancy (eg in someone with early gestational bleed)

preg NOT visualized on ultrasound at a β-hCG level below the discriminatory zone (<1500 IU/L); β-hCG level is repeated to see if the increase is c/w a normal pregnancy (IF STABLE) - (>35% rise every 48 hours). - Ultrasound is repeated when β-hCG @>1500 IU/L

cross section tells you what?

prevalence of something

The mechanism of cromolyn sodium is to ______ and good for prevention!

prevent airway inflammation by inhibiting mast cell degranulation

what type of hyperparathyroidism has high calcium

primary

Can a low probability v/q scan rule out PE in general?

it all depends on PRETEST PROBABILITY using modified wells score! Pneumonic: if the criteria read by doc = 3 points nurse =1.5 family member can say: 1 pnt >4 high pretest: PE LIKELY ≤4 PE UNLIKELY

key rib in exhalation somatic dysfunction

key rib is the upper most rib aka holding all the group down and vice versa FOR INHALATION SD

congenital toxo vs toxo prophy?

key with IGG congenital egg with pyramids: treat Pyrimethamine, sulfadiazine, folate prophy - key- egg only - tmp/smx

If 75 M, with osteophytes and sclerosis/ joint space narrowing of bilateral hips presents to you and only complains of L hip pain what procedure is needed?

l hip arthroplasty only rather than BL

Lyme arthritis is the most common (stage) complication and typically affects ul knee! how is it definitively diagnosed?

late stage develops monoarticular or oligoarticular joint pain and swelling. Fever and other systemic symptoms are rare; joint aspiration generally reveals a leukocyte count of 20,000-50,000/mm³. Positive Lyme serology is diagnostic.

STATS: difference btw lead time bias and length time bias?

lead time bias - the screening is done sooner without impacting survival length time bias - the screening is done sooner catching more benign corse of disease rather than making the disease benign (eg doesn't measure detection time for someone who would have a bad case) length time = more benign

Methemoglobinemia causes a ____ shift of the curve, and the hemoglobin has a __( high vs low) __ affinity for o2.

leftward greater affinity for oxygen

child with multiple cafe au lait macules (CALMs) plus axillary or inguinal freckling needs what screening?

likely has NF1, autosomal dominant vision screen for glioma if abnormal then MRI of brain and orbit needed to determine tumor size

patients with chronic hypoparathyroidism serum calcium and urinary calcium (eg 7.8 and 680 urine respectively) need what drug tx?

long-term management of borderline low serum calcium and high urinary calcium in hypothyroidism thiazide diuretic will not only decrease the urinary calcium but also increase serum calcium levels.

Wilson disease is causes low serum ______ levels. It typically presents before age 35 with ______ and elevation in transaminases, and neuropsychiatric symptoms (eg, depression, movement disorders).

low ceruloplasmin due to abnormal copper transport regulation hepatomegaly (INTIALLY)

1- 6.11 months of schizophrenia is called?

schitzophreniFORM disorder

Is the BNP reliable is someone who is obese?

No obesity lowers BNP thus HFpEF exasterbation could have a low BNP theoretically if they are obese. Diagnosis of HFpEF is maid via CLINICAL findings along with CXR (maybe echo)

Can relative risk or risk ratio be calculated in a case control study?

No only odds ratio which is = ad/bc has to do with probability

When would you order an antimitochondrial antibody test? Should it be done in someone with normal alk phos?

No order an AMITO AB in primary biliary cirrhosis PBC and it would have ELEVATED ALK PHOS and normal ast (exact opposite of autoimmune hepatitis)

OMM: rule of 3

T1-3: spinous and tv at same leve T4-6: TV half up from spinous T7-10: TV 1 STEP up from spinous T11: 1/2 STEP T12: SAME

Drug that causes lithium toxicity? Slurred speech, confusion, tremor, and ataxia.

TAN thiazides (eg chlorthaladone), ACE, NSAIDs cause lithium serum levels ≥1.5 confirming toxicity. Thiazides increase NA excretion in distal tubule and cause volume depletion thus causeing increased proximal tubular reabsorption of NA and lithium reabsorbution

When can warfarin be initiated in a non-heparin anticoagulant from Hit 2

after plt count is ≥150k

Res ipsa loquitur implies

"The thing speaks for itself" negligence. Res ipsa loquitur also requires that the negligent event that occurred was solely under the defendant's and that the plaintiff (the patient) did not at all contribute to the cause.

Name the most important factor in determining the need for therapy in Ascending weakness with loss of DTR s/p diarrheal illness.

( Guillain - Barré ) peak inspiratory pressure measure is the most important factor in therapy with either intravenous immunoglobulins (IVIG) or plasmapheresis (plasma exchange). The PIP diminishes as the diaphragm is weakened - predicts respiratory failure

Describe Heerfordt's syndrome and what is it diagnostic of?

(uveoparotid fever) is another collection of clinical findings specific enough to diagnose sarcoidosis without a biopsy. These signs and symptoms include uveitis, parotid gland enlargement, and facial nerve palsy.

What is the risk factor for postpartum endometritis? When should it be suspected?

+1 =c-section (specifically when after labor onset) Others: chorio, GBS +, PROM, operative vaginal delivery P/w fever uterine fundal tenderness +purulent locia

Modified Wells criteria for pretest probability of pulmonary embolism hint: if the criteria read by doc = 3 points nurse =1.5 family member can say: 1 pnt >4 pe likely Total score ≤4 = PE unlikely >4 = PE likely

+3 points Clinical signs of DVT Alternate diagnosis less likely than PE +1.5 points Previous PE or DVT HR >100 Recent surgery or immobilization +1 point Hemoptysis Cancer

Subarachnoid hemorrhage is based on (what PE findings) and evidence of (what on imaging ?) . The most common cause is trauma, followed by aneurysmal bleeding. The treatment is supportive.

- Diagnosis of a subarachnoid hemorrhage is based on the appropriate clinical history of sudden "thunderclap" headache and - CT non/con: evidence of hyperdense blood filling the cortical sulci and surrounding cisterns.

what further test is needed in pt with elevated (normal) tsh, t3/t4, elevated alpha subunit

- GET MRI. tsh secreting pituitary adenoma - can also elevate other hormones Additional tests - Minimal or no suppression of TSH by exogenous thyroxine - Elevated sex hormone-binding globulin - Normal thyroid hormone receptor beta gene

Mom with chronic HTN should have what BP during pregnancy SBP >140 or DBP>90

- Initiate aspirin therapy - Initiate or continue antihypertensive treatment as indicated- Increased maternal and fetal surveillance- Delivery between 37 0/7 and 39 6/7 weeks

How is preeclampsia treated if severe features are present?

- Initiation of magnesium sulfate for seizure prophylaxis - Inpatient management for maternal and fetal monitoring with delivery at 34 0/7 weeks - If unstable, immediate delivery

How long should OM be treated and what is a test of cure?

6weeks, ESR and other serial inflam. markers

Hypertension in Pregnancy considered as Preeclampsia With Severe Features when

- SBP 160 mmHg or DBP >110 mmHg - Pulmonary edema - New-onset RUQ pain - New-onset HA unresponsive to med - Visual symptoms

ASD features

- asymptomatic UNTIL 40Yo - exercise intolerance, DOE, and fatigue. - Mild systolic ejection murmur at pulmonary area secondary to increased pulmonary blood flow Wide, fixed splitting of S2 across tricuspid valve area secondary to increased blood flow -RVF may be seen IN ADVANCED DX

ABPA diagnostic criteria

- brown mucus plugs eosinophils w. Asperg. -fleeting infiltrates - Aspergillus antigen skin testing, elevated total (eg, >417 IU/mL) - Aspergillus-specific IgE, serum precipitins to Aspergillus antigen, - central bronchiectasis on high-resolution CT, - peripheral eosinophilia.

Brugada syndrome and long QT syndrome are associated with an increased risk of SCD, but the SCD is not commonly --what-. - ECG findings for Brugada syndrome( --- ) or long QT syndrome (QTc >---msec in men or >--- msec in women)

- exertional. - EKG: right bundle branch block and ST-segment elevation in leads V1-V3 - LONG QTc > 450 (456 easier to rrmbr 7-- 470); 470 women

Depressed children and adolescents may display _____ rather than depressed mood. Initial evaluation of all children and adolescents with depression should include _____.

- irritability in adolescents vs sadness - careful suicide risk assessment.

Autoimmune hepatitis can cause -- (2 PE findings) -- in young F and can be chronic in nature and associated with autoimmune diseases along with elevated --- ( 2 lab values ) and normal ----.

- jaundice and - RUQ pain in a young woman 1. elevated indirect bilirubin, and 2. elevated liver enzymes and 3. normal alkaline phosphatase.

All patients with Medullary Thyroid Cancer require what ancillary tests?

- serum calcitonin - CEA, - neck ultrasound (evaluation for regional metastases), - genetic testing for germline RET mutations, and - evaluation for coexisting tumors (hyperparathyroidism, PCCs).

what anti-d ab titier is considered indicative of ALLOIMMUNIZATION

-300 µg anti-D immune globulin dose is administered at 28 weeks gestation and again <72 hours postpartum in an Rh(D)-negative mother with a Rh(D)-positive infant. Pregnancies C/B abruptio placentae/ HEMORRHAGE - prevents maternal sensitization only up to 30 mL of fetal blood.

BELL CURVE SD PERCENTAGES

0-1SD = 68 1-2 = 95 2-3 = 98 68, 95, 98

how many SD for 95%

2 SD from mean

2 months old baby can do what?

2 months: lifts head/chest when prone, tracks past midline, alert to sound, coos, recognizes parent, social smile

2 year old kid should be able to

2 word phrase 2 step command take of clothes stairs jump 6 cube towers

Class IV hypovolemic shock is associated with > ____ ml of blood loss, HR >___, and lethargy.

2,000 hr > 140

Venous Air embolism if suspected, lay them down! 1. attach a syringe to the hub of the catheter immediately to prevent further air entry and 2. attempt to _______. 3. Placing the patient on the_____side will theoretically prevent the air from entering the _____ heart, - reducing the risk of embolization into the ____ heart and arterial circulation. 4. Intubation if hemodynamic ∆'s

2. attempt to aspirate air through the catheter. 3. left side

permissible htn for ischemic stroke who did not get thrombolytic tx and was not due to embolic event?

220/120 mm Hg is generally permitted to allow appropriate perfusion of borderline ischemic regions

BEST INTITIAL TEST FOR ADDISON?

24 HOUR CORTISOL AM

The initial test of choice when suspecting Cushing syndrome is a ____ measurement. or ______ -dose dexamethasone

24-hour free cortisol measurement. Other options include a low-dose dexamethasone suppression test or a late-night serum/salivary cortisol testing.

WOULD REMOVAL OF A VILLOUS ADENOMATOUS POLYP be considered a secondary or tertiary problem if pt has a family history?

2NDARY as villous adenoma is. PRECOURSOR to disease tertiary if patient has SYMPTOMS (EG weight loss, rectal bleed) then treatment via excision would be tertiary treatment

How many levels of prevention exist? and what do they coorespond to?

3 levels primary removes potential risk factors secondary: screening in an asymptomatic or mild disease to prevent progression with the goal or reducing complication and treat early Disease state tertiary prevention: symptomatic disease with goal of preventing spread (eg rectal bleed with adenocar polypectomy)

how many phases of CML

3 phases including chronic, phase, accelerated phase, blast crisis which resembles acute leukemia

At ___ years: rides tricycle, climbs stairs with alternating feet (3 to 4 years), copies a circle, uses utensils, uses 3-word sentences, brushes teeth with help, washes/dries hands

3 years

3-5 years old can have ___ as cause of pNA, while >5yo commponly infected with ?

3. virus, strp pna, hib, pyogenes, areas 4. > 5yo: virus, Mycopla, strep pna, chlamidia pneumoniae, aureaus (post influenza only) , pyogenes

When is GBS checked in pregnancy, what if it is found before screening is IV penicillin treatment needed?

35-38 weeks and yes treat to prevent OB complications

§ . Cutaneous (skin) anthrax is treated with antibiotics taken by mouth, usually for ____ with ___are most often used.

7 to 10 days. Doxycycline and ciprofloxacin

kids age: hops, copies a cross, counts to 10, cooperative play

4 years: hops, copies a cross, counts to 10, cooperative play

Experts now use a reference level of ___ to identify children (1- 5yo) with blood lead levels that are much higher than most levels in children.

5 μg/dL t

How is carcinoid tumor diagnosed and managed?

5-HIAA is diagnostic Management: surgical resection if localized. Octreotide used for symptomatic control.

Headache red flags? Age >?

50

GDM 24-28 week if failed whats next?

50grm oral glucose challenge test check BS in 1hr <140 Need glucose tolerance test - give 100gms and check 1hr 2hr 3hr BS = if abnormal in >2 reading GTT you have gestational DM

What are the 5 reversible causes of asystole/pulseless electrical activity

5H's hypovolemia hypoxia hydrogen ions acidosis hypokalemia (hyperk) hypothermia 5T's Tension pneumothorax Tamponade Toxins (narcotics and benzos) Thrombosis (pulmonary or coronary) Trauma

how long do you need to do serial b hcg draws in someone with hydatiform mole?

6 months give them ocp

schizophrenia needs how many months of symptoms

6.66 month of symptoms schitzophrenia

6 months: 9 to 10 months bby

6m sits unassisted, transfers objects, raking grasp, babbles, stranger anxiety 9m crawls, pulls to stand, uses 3-finger pincer grasp, says mama/dada (nonspecific), waves bye-bye, plays pat-a-cake

laryngotracheitis croup develop in what age group

6mo- 3years

IDK

A 49-year-old man comes to the office for follow-up. A friend was recently diagnosed with gout, and the patient is anxious that he will also develop gout. The patient states that he had read "something about a diet called DASH that reduces the excess of uric acid in the blood and reduces the risk of gout." He would like to try the diet but asks if a less strict version would nevertheless help to reduce his chances of developing gout. Item 1 of 2 Based on the abstract, if this patient decided to adopt a strict version of the DASH diet (corresponding to the highest fifth intake ranking) as compared to a less strict version of the DASH diet (corresponding to the third fifth intake rating), what proportion of his reduction in risk of gout could be attributed to this intervention? A. 0.28 (42%) B. 0.50 (14%) C. 0.72 (30%) D. 1.00 (6%) E. 1.38 (6%) IncorrectCorrect answer A 42%Answered correctly 04 mins, 45 secsTime Spent 06/16/2020Last Updated Explanation The relative risk reduction (RRR) quantifies the proportion of risk reduction attributable to a specific intervention or exposure as compared to a control. RRR considers the risk for disease in the exposed/intervention group and the unexposed/control group as follows: RRR = (risk in unexposed − risk in exposed) / (risk in unexposed) In this case, the exposed/intervention group is the DASH diet in the highest fifth, and the unexposed/control group is the DASH diet in the third fifth; the risks are reported in person-years (incidence densities). The risk of gout among subjects adopting a DASH diet in the third fifth (unexposed/control group) is 354 / 196,510 = 0.0018, or 1.8 per 1,000 person-years; the risk of gout among subjects adopting a DASH diet in the highest fifth (exposed group) is 252 / 193,455 = 0.0013, or 1.3 per 1,000 person-years. Therefore, the RRR is calculated as: RRR = (1.8 per 1,000 person-years − 1.3 per 1,000 person-years) / 1.8 per 1,000 person-years RRR = 0.5 / 1.8 = ~0.28 (28%) Alternately, RRR is frequently calculated by subtracting the relative risk (RR) from 1: RRR = 1 − RR. RR is the risk of developing a disease (eg, gout) in the exposed group (eg, DASH diet in highest fifth) divided by the risk of developing the same disease in the unexposed group (eg, DASH diet in third fifth). In this case, RR = 0.0013 / 0.0018 = ~0.72 (72%) (Choice C). Therefore, RRR = 1 − 0.72 = 0.28. This means that, compared to patients who adopt a diet DASH diet in the third fifth, patients who adopt a DASH diet in the highest fifth reduce their risk of gout by 28%. (Choice B) The absolute risk reduction (ARR) is defined as the difference in risk between the unexposed and exposed groups. In this case, ARR = 1.8 per 1,000 person-years − 1.3 per 1,000 person-years = 0.5 per 1,000 person-years. (Choice E) If adopting a DASH diet in the third fifth is considered the exposure and adopting a DASH diet in the highest fifth is considered the reference, then RR = 0.0018 / 0.0013 = ~1.38. This means that a DASH diet in the third fifth increases the likelihood of gout by 38% when compared to a DASH diet in the highest fifth. Educational objective:Relative risk reduction may be used to determine which of 2 interventions is more beneficial to reduce the risk of disease based on incidence densities (ie, person-years data). Careful attention is required when identifying the exposed and unexposed groups between 2 interventions.

What is a common cause of stress urinary incontinence and how is it diagnosed?

A common cause of stress urinary incontinence urethral hypermobility. The Q-tip test is performed by placing cotton swab inside the urethra to measure the extent of urethra mobility during the Valsalva maneuver; a positive test has a >30-degree angle of movement. (3 unlucky number)

_____ can test the difference between 2 paired means; patients serve as their own control (eg, mean blood pressure before and after treatment in the same subjects).

A paired t-test

A blood sample should be drawn during the hypoglycemic episode evaluation involves what 4 measurements in the setting of true hypoglycemia if they fulfill Whipple's triad?

A serum assay to determine hypoglycemia cause: 1. oral hypoglycemic agents 2. serum levels of insulin, 3. C-peptide, and 4. proinsulin

________ abortion presents with vaginal bleeding, a closed cervix, and an intrauterine pregnancy with normal fetal cardiac activity.

A threatened ABORTION COMLEX A closed cervical os and a normal gestational sac will be seen. Expectant management and pelvic rest constitute the treatment regimen.

OMM: AA SD DX what is the primary motion and what vertebra does the atlas articulate with?

AA considered to be C1 motion on C2 and primary motion is ROTATION opposite to sidebend

In pt with CAD and prior MI need what therapy for secondary prevention of CV events?

ABCDE asprin beta blocker clopidegrel diuretic (ace) ECHO

In pt with either TB skin test or igra with symptoms of TB and cxr findings what is the way in which you diagnose active TB? (hint: AtFirstB -- MAC on that --- NAA)

AFB - rapid cheap most used up but can't give you a definitive answer, LOW sensitivity (false neg), poor differentiation MAC- takes 2-6 weeks to mac and its gold can tell you what drugs you need NAA( LIKE GHON TEST )--- super sensitive specific and wants results fast unlike AFB; naa can distinguish btw type

Elevation of ___ is associated with spina bifida (failure of the caudal neuropore closure)

AFP alpha feto protein elevation, also associated with pregestational DM, low folate

In someone with AK lesions what is the treatment if local and if wide spread. Why is treatment needed?

AK is risk for SQUAMOUS CELL CARCINOMA local treatment is with ablation (cryo) field therapy with 5FU

When can a on Willebrand factor-cleaving protease be ordered?

AKA ADAMSTS13 = $$$ expensive laboratory test. NEED TO FIRST CONFIRM THROMBOCYTOPENIA! AND PE SHOULD HAVE TTP signs e.g TTP classically presents with the pentad of altered mental status, fever, thrombocytopenia, renal failure, and hemolytic anemia.

PREECLAMPSIA OCCURES WHEN DURING PREGNANCY??

After 20weeks and up to 12 weeks post-partum

What is HITTS? And what does the American College of Chest Physicians want us to do about it?

After DC heparin (includes LMWH) products pt can be placed on danaparoid, lepirudin or argatroban, alone or in combination with warfarin to treat thrombotic risk

What dose of statin needed for 2ndary prevention of ascvd?

Age ≤75: High-intensity statin Age >75: Moderate-intensity statin

Common causes of postoperative hypoxemia that occur right after surgery?

Airway obstruction/edema: Stridor is common Often due to endotracheal intubation or pharyngeal muscle laxity bronchospasm: WHEEZING Scheduled nebulized bronchodilators residual anesthesia

dyspepsia alarm features

Alarm symptoms 1. Progressive dysphagia 2. Iron deficiency anemia 3. Odynophagia 4. Palpable mass or lymphadenopathy 5. Persistent vomiting 6. FH of GI malignancy

Alcohol use (abuse) screening - CAGE Single-item screening

Alcohol use (abuse) screening - CAGE - felt you should cut down on your drinking? - annoyed you by criticizing your drinking? - felt bad or guilty about your drinking? - ever taken a drink first thing in the morning (eye-opener) to steady your nerves? Single-item screening: How many times in the past year have you had 5 (4 for women) or more drinks in a day?

26yo recurrent asthma exacerbations, fever, lethargy, cough with production of brown mucus plugs. PE W. diffuse wheezing with adequate air entry. -high resolution CT scan shows central bronchiectasis and a right lower lobe infiltrate. Review of prior chest x-rays shows a lingular infiltrate and a left lower lobe infiltrate. Which of the following is the most likely cause of this patient's symptoms?

Allergic bronchopulmonary aspergillosis (ABPA) PATH: vigorous IgE and IgG immune-mediated hypersensitivity response to Aspergillus. EITIOLOGY: preexisting asthma or cystic fibrosis. CLINICAL: fleeting infiltrates(eg, transient infiltrates in different parts of the lungs). , recurrent asthma exacerbations, and central bronchiectasis. S/O: asthma exacerbations, fever, lethargy, cough

Medications that cause Urinary incontinence?

Alpha-adrenergic antagonist - urethral relaxation anticholinergic, opiates, cal block --> retention/overflow Diuretics excess production

Survival analysis is used to account not only for the number of events in both groups but also for what ?

Also for timing of events thru f/u period.

Psudodementia puts one at most risk for what if they are ≥65Yo?

Alzheimers and vascular dementia later on in life (NOT with other psych issues even anxiety is not a risk because late onset depression is not comorbid with anxiety!)

breast milk jaundice treatment and prognosis?

American Academy of Pediatrics recommends that infants with breast milk jaundice continue breastfeeding exclusively WITH FREQUENT F/U and monitoring prognosis is good with spontaneous resolution by age 3 months.

Anticholinergic TCA overdose effects zzq

Anticholinergic Dry mouth, blurred vision, dilated pupils Urinary retention, flushing, hyperthermia (not to be confused with heart muscle - N/A current)

What are the main side effects of TCAs?

Anticholinergic effects, orthostasis, cardiotoxicity in OD and are usually 2nd line in Depression treatment.

anti GBM antibiodies needs what treatment

Antiglomerular basement membrane disease (Goodpasture syndrome) is a rare condition characterized by acute glomerulonephritis and pulmonary alveolar hemorrhage. plasma exchange and immunosupression with corticosteroids and cyclophosphamide.

In someone with bradyarrhythmia symptoms of hypotension, dizziness, HF, syncope and STEMI that is not responsive to IV atropine what type of MI is it and what is the next step in treatment?

Anterior wall MI due to damage BELOW the AV node aka AV BLOCK (lad damaged which damages below the av node causing av node block thus av node cant transmit signal) will be unresponsive to atropine (as opposed to inferior wall MI with vagal tone inc) Treat with temporary transcutaneous then transvenous CARDIAC PACING then PCI

How would anemia of chronic disease due to inflammatory conditions such as rheumatoid arthritis managed?

Anti-TNF-α ( infliximab) agents improve anemia of chronic disease in patients with rheumatoid arthritis. (note Iron/ TIBC % matches iron level) I before T in alphabet = %

What happens if you give a baby with botulism abx?

Antibiotics (especially aminoglycosides) are avoided as they can cause colonic C botulinum to lyse, increasing toxin absorption.

Pt with days of cough, wheezing, shortness of breath and drop of PEF >20% likely has what in an asthmatic? What is the home management?

Asthma exacerbation 2/2 viral URI (also has nasal congestion, sore throat, myalgia) SABA albuterol followed by systemic CST (prednisone 40mg qd x 5-10days) if sx persist.

Renovascular htn PE exam, labs and imaging show what?

Asymmetric renal size (>1.5 cm), Abdominal bruit Labs: Unexplained rise in serum creatinine (>30%) after starting ACE inhibitors or ARBs Imaging: Unexplained atrophic kidney

Asthma is characterized by intermittent, reversible airway obstruction HOWEVER would PFT be normal in an asymptomatic asthmatic?

Asymptomatic pt have normal pulmonary function testing, but obstruction (>20% reduction in FEV1) can be provoked by methacholine challenge.

What are the post op pulm complications? What should be checked in routine preop lung function?

Atelectasis Bronchospasm Chronic lung dx exacerbation Infection Prolonged mech vent + resp failure Routine preop PFT or ABGs do NOT help predict or change post-op complications!

What 5 autoimmune conditions are associated with autoimmune hepatitis?

Autoimmune Thyroiditis Type 1 DM UC Celiac DX SLE

How does serotonin syndrome present?

Autonomic dysregulation, NM hyperactivity, and myoclonus

GERMAN ( vincent van g) MEASULES is also known as? hint Infants typically have low birth weight and microcephaly; purpuric lesions ("blueberry muffin" rash) occur in a minority of cases.

Congenital rubella syndrome presents with 1. sensorineural hearing loss, 2. patent ductus arteriosus, and 3. cataracts. 4. purpuric lesion: extramedulary hematopoeisis 5. microcephally

What type of hyperbilirubinemia is considered pathological? (hint: think of twinz)

Conjugated (bad news like conjoined twinz) hyperbilirubinemia is always pathologic w/u: with US of the hepatobiliary tree (aka gall bladder and liver) to evaluate for biliary atresia. - direct bilirubin is ↑ (>20% of total bilirubin)

What is one important cause of right heart failure which can cause edema, ascites, hepatic congestion and hepatomegally?

Constrictive pericarditis due to calcification can progress to cirrhosis (cardiac cirrhosis) treatment= supportive + nsaid refractory cases with pericardectomy (rare)

Why is IVC filter placed? IVC filter puts you at risk for what long term?

Contraindication to anticoagulation (eg stroke, active bleeding veracies, hemorrhage etc). Long term risk of Peripheral DVT although PE are prevented and embolisms prevented in renal artery due to IVC filter placement.

Diphtheria-tetanus-acellular pertussis immunization contraindication?

Contraindications: Encephalopathy after previous dose Anaphylaxis to vaccine component Components Diphtheria toxoid Tetanus toxoid Conjugated pertussis antigen (pertactin) 5 doses given at ages: 2, 4 & 6 months15-18 months4-6 years

What correlation value indicating a positive or negative direction of association btw 2 variables (eg margins -1, 1) can state a stronger association?

Correlation coefficient (r) values

Suspect secondary hypoadrenalism due to steroid withdrawal in postoperative patients who suddenly develop hypotension, shock, and hyperkalemia. Immediate treatment with _____.

Corticosteroids should be performed if this diagnosis is suspected, even if it has not been confirmed. Dexamethasone does not interfere with serum cortisol assay and, thus, is a good initial agent!

What pathogen leads to herpangina in children AGE 3-10? When is infection highest and what are symptoms?

Cox group A virus causing PAINFUL vesicles in posterior oropharynx. Infection common in summer and resolves in a week with supportive tx hand washing is best prevention

STATS: Cross section study is observational study in which a specific population of group is studied at one point What is a crossover study then?

Cross over study is an experimental study that subjects are exposed to different treatments or exposures sequentially and then later are crossed to serve as their own controls.

STATS: When are Hazard ratios useful?

Drug trials to analyze survival data or Drug trial looking at time to event data!

What are the potentially reversible causes of urinary incontinence in elderly? (hint: DIAPPERS)

DELERIUM INFECTION (UTI) Atrophic vag/urithritis Pharma (alpha block, diuretics) Psych. depression Excessive urination from DM or CHF ROM Restriction mobility (eg postop) Stool impaction

Dx: s/p surgery Sudden-onset pleuritic chest pain (WORSE WITH INHALE), combined with a loud P2 and pleural friction rub on physical examination. CXR evidence of pleural effusion VS: HR120, 99F, 92o2 sat

DIAGNOSIS: acute PE.

Plummer-Vinson syndrome is the triad of

DIE in your web of liiiiiies (low iron) iron deficiency anemia, dysphagia, and esophageal web.

Like asthma, chronic obstructive pulmonary disease demonstrates an obstructive pattern on PFT characterized by a reduced FEV1/FVC ratio what allows differentiation of asthma from COPD?

DLCO is also reduced in COPD DLCO IS LOW IN COPD (allowing for diagnostic differentiation from asthma) due to emphysematous destruction of the alveolar-pulmonary capillary membrane.

What level is hypoglycemia in diabetic and non-dm pt?

DM <60 healthy <45-50

What is the most significant predictor of adverse CV outcomes?

DM is considered a CHD risk equivlant

What is the MOST COMMON cause of carpal fracture as a result of FOOSH? How is it diagnosed?

DORSIFLEXED WRIST AND FOOSH INJURY = SCAPHOID FRACTURE Findings: decreased grip strength, ROM and tender snuffbox Diagnosis: XRAY full pronation and ulnar deviation, if not seen repeat in 1-2weeks Or bone scan in 3-5days Nondiplaced fractures treated with short arm thumb spicca cast (aka wrist splint) others to ortho. High risk for nonunion needs 12 week immobilization to heal!

what nerve provides sensation to 1st web interspace of toe?

DPN

The risk of DVT is highest ____ days following an ischemic stroke and is particularly high in patients with ___. In hemorrhagic stroke, ____ DVT prophylaxis ____ would generally be used initially.

DVT occurs 2-7 days s/p ischemic stroke with hemiparesis (up to 75% on the hemiparetic side). Hemorrhagic: needs prophy with mechanical (pneumatic compression)

How is Cerebral venous sinus thrombosis (CVST) diagnosed and treated in prego?

DX: MRI venography Tx: LMWH for recanalization of sinus obstructed by thrombus and to prevent further thrombus - hemorrhagic foci / infarctions should still be treated with ANTICOAGULATION!

What random preventative care tests should be done on someone with celiac's disease?

DXA scan due to Vit D deficiency and pneumococcal vaccine due to associated hyposplenism

When would it be dangerous to use a metoprolol or NDCCB

Danger with decompensated HF and hypo tension as they are negative ionotrops and can tank bp

What does a chronic etoh user who presents with prolonged INR, LE edema, abd distension with shifting dullness likely have?

Decompensated cirrhosis with ascities

______ are cx morbidity and mortality in patients with acute stroke _________ should be used for prophy in acute ischemic stroke.

Deep-vein thrombosis and subsequent pulmonary embolism are common causes of M & M in acute stroke, (particularly * dense hemiparesis) Low-dose heparin or LMWH is used for prophylaxis in acute ischemic stroke.

Long-acting insulin with the Formation of a soluble hexamer at the injection site allows a very long action

Degludec: 42+hrs

The management of gallstone pancreatitis depends on what? when is surgery needed during admission vs post hospitalization?

Depends on its severity. Patients with mild disease should undergo cholecystectomy within 7 days of clinical improvement. Those with more severe disease should delay cholecystectomy until the resolution of inflammation and complications.

STATS: What is a type 2 error dependent upon?

Depends on the power (cohort size) as power increases type 2 error decreases

What is Buproprion (Wellbutrin) used for?

Depression, smoking cessation some effect on ADHD. Bupropion, a norepinephrine dopamine reuptake inhibitor, is a first-line antidepressant that could be considered if the patient had persistent, intolerable side effects and/or failed an adequate trial of fluoxetine.

What long acting insulin analog is given BID ?

Detemir- BID administration

FROZEN shoulder occurs frequently in what pt?

Diabetics and treated with PT, treated with heat (frozen = heat) not synonymous with nerve impingement but often misdiagnosed as such.

Hyponatremia, hyperkalemia, and hyperchloremic metabolic acidosis on labs. and PE below should have what confirmatory testing? PE: + fatigue, weight loss, myalgias, SKIN: increased pigmentation, GU: decreased axillary and pubic hair

Diagnosis is confirmed with low morning plasma cortisol and elevated adrenocorticotropic hormone [ACTH] (often performed with an ACTH stimulation test). Chronic adrenal insufficiency should be suspected The most common etiology of this condition is primary adrenal insufficiency (Addison's disease).

What is the treatment algorithm for Graves? and which drug is preferred?

Diagnosis: TSH (low), T4 (high), radioactive uptake (related to t3 and tbg to dx TRUE HYPERTHYROIDISM causing increase TBG due to T4 binding causing displaced T3 to bind to the radioactive resin tracer! vs prego state) Start propranolol and a antithryroid drug such as methimazole 2. Radioactive ablation (contra-indicated in pregnancy and lactation) 3. Surgery: reserved for large goiters, obstructive symptoms, or cancer Generally Methimazole> > PTU because is it not hepatotox PTU in 1st trimester only

What therapy is a first-line treatment for borderline personality disorder?

Dialectical behavioR 1st line Pharmacotherapy may be used as an adjunct, but it does not treat the core disorder.

EKG of pericarditis?

Diffuse ST elevation + *PR elevation in lead aVR*

in someone with systolic HF or hypotension with Afib what should be given and why?

Digoxin is + ionotrop TREAT AFIB WITH ABCD- AMIODARONE BB CCB DIG

How do we monitor sacroiliitis disease? Can these people perform exercise? What is the life expectancy?

Disease progress with AS is monitored with ESR (acute phase reactants) and Xray 1. AP and lateral view of Lspine 2. Lateral view of Cspine 3. pelvic radiograph, including SI joint and hip Aerobic exercises improve overall functional status of pt with rheum dx and extension exercise is not harmful but prevents fusion! They have a normal life span.

Who needs to be treated for guillain barre?

Disease-modifying treatment with plasma exchange or intravenous immunoglobulin (IVIG) speeds recovery and reduces the need for mechanical ventilation. Patients with GBS should receive plasma exchange or IVIG if: Nonambulatory Within 4 weeks of symptom onset

What are some complicAtions of a supra-condylar fracture? What vasculature can be compromidsed?

Displaced supracondylar fracture causes NEUROVASCULAR ENTRAPMENT -Brachial artery and median nerve can cause decreased perfusion and weakness of hand FLEXION!

What is a major risk factor for colles fracture

Distal radial fracture risk inc with osteoperosis and should be evaluated for other injury such as ulnar styloid fracture, acute carpal tunnel s/o and scaphoid injury

What is the utility of cholinesterase inhibition>? Name them?

Donepizil, rivastigmine used for alzheimers to increase ACH in brain

What is the first line treatment for prolactinoma with levels as high as 500 prl?

Dopaminergic receptor agonists are 1-line treatment for prolactinomas, including large ones. dopaminergic receptor agonists generally leads to a decrease in tumor size within a few days.

What sensitivity can develop in tetracyclines?

Doxy= photosensitive, teratogen, stained tooth.

What could be determined in an old dude with knee effusion and sudden inflammatory symptoms who is being treated for heart disease?

Drug induced lupus! get an ESR!

Primary biliary cholangitis PBC (formerly primary biliary cirrhosis) ANTIBODY TEST ---->

Elevated alkaline phosphatase Positive AMA- ANTIMITOCHONDRIAL AB Liver biopsy if AMA negative may also have other autoimmune disorders (eg, CREST syndrome, type 1 diabetes mellitus

Spontaneous bacterial peritonitis SBP = complication of cirrhosis DX: >250 neutrophils/mm3 are found in the peritoneal fluid. therapy must be started immediately WITH WHAT 2 THINGS IV ____ has been associated with improved renal function and decreased mortality.

Empiric antibiotic START ASAP while waiting for cultures IV albumin associated improved renal function and decreased mortality.

Does enalapril cause orthostatic hypotension?

Enalapril is an antihypertensive drug that has not been associated with orthostatic hypotension as frequently as other diuretics. Side effects: renal failure, hyperkalemia, cough, and angioedema.

What management component is necessary in someone with hematemasis (active) after having placed 2 large bore IV's? What procedure would be contraindicated in this form of UGI bleed?

Endotracheal intubation (even if O2 sat is good) then upper endoscopy as it is both diagnostic and therapeutic for UGI bleed (varices) NG tube placement is contraindicated in variceal bleed

pediatric condition treated with intubation?

Endotrachial tube in epiglotitis (e =e treat) presents with fever stridor and no cough ! Intubation and intravenous antibiotic therapy are required for patients suffering from airway compromise secondary to epiglottitis.COMBANK Insight : Another clinical scenario of epiglottitis may present you with the following picture with the classic "thumb-print" sign.

Pt with RUQ pain, weight loss, recent immigration from mexico with avascular mass on RUQ imaging likely has what infection?

Entamoeba histolytica is an intestinal amoeba that can be associated with liver abscess. Amebiasis 2/2 E histolytica, a protozoan found worldwide but prevalent in developing countries, where barriers between human feces and food and water supplies are inadequate.

What is a visual difference between erysipelas vs cellulitis and are they caused by the same microbe?

Erysipelas has sharply demarcated borders and due to strep Pyogenes infection of the dermis --> Rapid progression with fever and chills While cellulitis caused by S pyogenes and MSSA is an infection of the deeper dermis and SQ fat --> thus ill-defined borders and flat; fever occurs gradually " she got cellulite thighs"

6 Indications for statin therapy in the secondary prevention of ASCVD? aka they have ascvd risk established.

Established ASCVD Acute coronary syndrome Stable angina Arterial revascularization (eg, CABG) Stroke, TIA, PAD

When should initial screening endoscopy be preformed in someone with gerd symptoms?

Eval done for barrets if chronic gerd or frequent symptoms with ≥2 of the following: age ≥50 male white hiatal hernia obesity tobbacco use 1st degree relative

Etiology multifocal atrial tachycardia?

Exacerbation of pulmonary disease (eg, COPD) Electrolyte disturbance (eg, hypokalemia) Catecholamine surge (eg, sepsis)

what 2 types of meds caus pancreatitis in DM

Exenatide (INJECTION) = GLP-1 agonist (acts as incretin mimetic) enhancing glucose-dependent insulin secretion. Side effects = gastrointestinal - pancreatitis Sitagliptin (PO) = DDP4 increases endogenous incretin also causes pancreatitis.

What are the toxin-mediated effects of diphtheria?

Exo toxin causes dose dependant damage to heart--> myocarditis which can progress to heart block and eventual HF neuritis kidney disease

This patient has ulcerative colitis, as suggested by progressive, bloody diarrhea, abdominal pain, and fatigue associated with signs of uveitis on examination. Ulcerative colitis is an inflammatory bowel disease that is always confined to the colon and rectum. Patients typically present with frequent diarrhea that may be bloody, tenesmus, and abdominal pain. Both common types of inflammatory bowel disease (Crohn's and ulcerative colitis) can be associated with extraintestinal manifestations such as uveitis.

Extraintestinal manifestations of ulcerative colitis include skin changes, arthralgias and arthritis, ocular involvement, and hepatobiliary diseases. This patient has uveitis, which is often insidious in onset and may be initially mistaken for dry eyes or allergies. Uveitis often presents with bilateral eye pain, conjunctival injection, and ciliary flush. It is associated with rheumatologic and systemic inflammatory diseases in addition to opportunistic infections. uc may also present with a mild precursor event of abdominal pain and diarrhea that self resolves months before the disease relapses. Ulcerative colitis does incur an elevated risk of colorectal adenocarcinoma and requires colonoscopy 8 years after symptom onset.

Lights criteria include what? and if criteria is met what does it mean?

Exudation caused by inflammation Pleura/serum protein >.5 Pleura/serum LDH >.6 PLEURAL LDH >60 (2/3 upper limit of normal serum LDH range 45-90)

TTP pentad AND TREATMENT

FAT RN Nasty Fever Torched His Kidneys Neurologic sx Fever Thrombocytopenia +anemia Hemolysis Kidney failure-- uremia dx: plt >30 and sCr >1.7 to 2.3 and adam ts13 <5%? PLASMAPHORESIS

BREAST FEED WITH HIV?

FORMULA IF IN US breast feed if in 3rd world with HIV on HAART

SBP is given prophy with what?

FQN in cirrotics during hospitalization given for 7days

What should be done after someone is diagnosed with osteopenia with DXA btw -1 to -2.5?

FRAX assessment to calculate fracture risk

OMM LAW 1 AND 2

FRYETTE LAW 1 AND 2 ONLY APPLY TO L AND T SPINE NOT C SPINE~!V F SAME DIRECTION N POINT OPPOSITE DIRECTION

What genetic condition increases risk for development of venous thrombosis in cerebral, mesentery deep veins>

FV leiden heterozygosity

Hemophilia A has which factor missing and reflected in what lab value?

Factor VIII PTT prolonged (intrinsic clotting cascade) X-linked recessive inheritance ( § male predominance)

androgenic steroids, glucocorticoids, chronic liver disease, and protein-losing states (eg, nephrotic syndrome) decrease need for thyroid hormone BY WHAT MECHANISM?

Factors that conversely decrease TBG thus less thyroid hormone needed to saturate sites - thyroid hormone is protein bound in plasma (mostly to tbg)

clinical diagnosis that is usually characterized by a triad of respiratory insufficiency, neurological impairment, and a petechial rash?

Fat embolism occurs 3 days s/p multiple fractures -petechial rash is most commonly found on the trunk; this is a late manifestation

What part of the iron panel tells you if microcytic anemia is due to iron vs thalassemia

Fe anemia = microcytic with low ferritin <15-30, small rdw (tiny rbc) alpha thalacemia = microcytic with HIGH ferritin and high rdw Partners of women with thalassemia minor should undergo hemoglobin analysis to assess fetal risk of inheriting thalassemia.

Blood pressure management is a common subject to address in the outpatient clinic. Blood pressure should be checked at every clinic visit.

Hypertension is a major contributor to cardiovascular disease and is more common with increasing age. In this patient with diabetes, her systolic blood pressure goal according to JNC 8 <140 mmHg/ <90 mmHg. SPRINT trial with some arguing for targets as low as <120/80. -Normal blood pressure in nondiabetic patients is defined as < 120/< 80 mmHg. Elevated blood pressure is 120-129/< 80 mmHg. - stage 1 is 130-139 mmHg systolic or 80-89 mmHg diastolic. stage 2 is ≥140 mmHg systolic or ≥90 mmHg diastolic. ddx: average based on ≥2 readings obtained on ≥2 occasions to estimate the individual's blood pressure. - patient presents with a blood pressure of >180/120 or with a hypertensive emergency. ---: Both the systolic and diastolic pressures are elevated, as normal diastolic blood pressure is < 80 mmHg. -Patients with hypertension should be treated to a goal blood pressure of <140/90 mmHg. Almost all societies agree with a target of <140/90 for the majority of patients.

What lipid levels are improved with metformin and thus good for obese DM patient with Hypertriglyceridemia?

Hypertriglyceridemia along with hepatic steatosis

What electrolyte derangements causes TDP

Hypokalemia, hypomagnesemia, and hypocalcemia (less common) increase the risk of developing recurrent episodes of TdP. No association is seen with hypernatremia.

Should IV abx be given to somone with hepatic encephalopathy for SBP prophy?

Hypovolemia and electrolyte abnormalities are more likely to be the precipitants of HE HOWEVER INFECTION CAN ALSO BE A CAUSE ABX CAN BE GIVEN UNLESS PT HAS: normal leukocyte count, no fever, and no abdominal pain.

What are the signs and symptoms of severe asthma exacerbation?

Hypoxemia, use of accessory muscles, reduced PEF >50% from baseline.

PYODERMA GANG.

IBD

What is responsible for 80% of occupational BL hand dermatitis and commonly due to use of irratants (eg detergent, solvent, oxidants) and occurs after repeated exposure?

ICD irritant contact dermatitis - puritic, erythematous scaling of skin --> chronic hyperkeratosis and fissuring eg on thin skin like hand dorsum, fingertips and webs. Occlusion of irritants under rubber gloves is typical cause

Increased FEV1/FVC occurs in what 2 diseases?

ILD (restrictive) and obesity hypoventilation s/o

TCA overdose can also cause acidemia, which can mildly increase

INC serum potassium due to cellular exchange of hydrogen and potassium.

PTH and thiazide have what in common?

INCREASES RENAL CALCIUM ABSORPTION especially if they have urinary calcium excretion and low serum ca despite supplementation in someone with hypoparathyroidism.

FFP indicated with inr

INR >/2

STAT: kappa statistic is quantitative measure of what concordance?

INTER RATER RELIABILITY +1 = agreement 0 = agreement due to chance alone -1= intentional disagreement >.08 excellent .41-60 fair .2-.4 minimal <.2 negligible improvement

What does palliative care do?

INterdiciplanary approach to prevent unneeded services in those with <6mo to live. However unlike hospice can allow for life prolonging measures

ISCHEMIC stroke pt who received thrombolysis should be managed more conservatively with blood pressure maintained at <_____ mm Hg for at least _____ to minimize risk of HEMORRHAGIC TRANSFORMATION

ISCHEMIC <185/105 24 hours (the precise lower limit of blood pressure is unclear)

What is the management for a pt with suspected UGI bleed and hg of 7.7, with melena?

IV crystaloid w PPI with planned endoscopy CLOSE MONITORING ONLY if pt is hemodynamically stable then restrictive RBC transfusion (eg hb<7 is done) No octreotide! as that is for UGI hemmorage No albumin infusion needed as it is not better than crystaloid infusion in acute gi bleeds

How is acute dystonic reaction treated in someone on an antipsychotic or metaclopramide?

IV diphenhydramine or bromocriptine can reverse dystonic reactions.

How is fulminant cdiff treated? Eg ileus, toxic megacolon?

IV metronidazole and high dose oral vancomycin; vancomycin may be administered rectally for ILEUS pt.

If variceal UGI bleed is suspected what medication should be administered while waiting endoscopy? What is the MOA What would be secondary prevention of rebleed if pt has undergone endoscopy? (hint think of altering BV)

IV octreotide lowers portal htn via decreasing splanchnic blood flow thus may actually stop bleed nonselective Beta blockers; nandolol and propranolol

How would osteoporosis be treated in roux en y pt?

IV zolendronic acid! Oral bisphosphonates are the preferred first-line treatment for osteoporosis. When oral bisphosphonates are contraindicated IV bisphosphonates are next line.

When does adolescent idiopathic scoliosis warrant further X-ray evaluation?

Idiopathic scoliosis is characterized by lateral curvature of the spine. a scoliometer angle of rotation ≥7 degrees requires x-ray of the spine.

when is NAAT of lesion on penis done?

If LAD present - get NAAT of the penile ulcers, which can be performed to diagnose painless lesions of lymphogranuloma venereum LGV (caused by L1, L2, and L3 serovars of Chlamydia trachomatis)

how can you evaluate if a pancreatic pseudocyte is necrotic

If a pancreatic pseudocyst or necrotic collection is suspected, a CT abdomen with IV contrast is the imaging test of choice to evaluate for the size of the collection, the maturity of the wall, and for debris or evidence of necrosis.

What does selection bias lead to and how may one assume a study is affected by it?

Making an association when one does not exist can occur in pt population that is underrepresentative eg a group with measured levels in one category has less participants (n is low)

Malunion is more commonly seen in what type of fracture?

Malunion: physeal fracture of the distal forearm in children - due to growth plate injury and arrest; - growth disparity between the radius and ulna and results in = wrist deformity.

PREGO W Clinical features NEEDS WHAT MGMT? LVEF <45% Onset: 36 weeks gestation - 5 months postpartum No other cause of heart failure

Management Peripartum cardiomyopathy Deliver based on maternal hemodynamic stability Standard systolic heart failure regimen Thromboembolism prophylaxis

The above patient has evidence of a congenital cardiac shunt. Right-to-left shunts cause cyanosis. Since the child is 10 years of age, it is likely that this recent cyanosis is secondary to a right-to-left shunt caused by Eisenmenger syndrome. Eisenmenger syndrome occurs when a long-standing left-to-right shunt reverses and becomes a right-to-left shunt, thus bypassing the lungs and providing the organs with unoxygenated blood. Signs and symptoms include cyanosis, high red blood cell count, clubbing of the fingers, syncope, and heart failure. The murmur of a ventricular septal defect is characterized as a harsh pansystolic murmur best auscultated at the left lower sternal border.

Management involves surgical repair or transplantation in severe cases.Answer A: An atrial septal defect will present with a loud S1 heart sound and a wide and fixed S2 heart sound.Answer B: A patent ductus arteriosus has a continuous machine-like murmur that is loudest in the second left intercostal space.Answer C: A congenital cardiac right-to-left shunt would most likely have caused the patient to become cyanotic at a much earlier age, making this answer unlikely. Tetralogy of Fallot typically presents in younger children who experience "tet" spells, which are characterized by a sudden, marked increase in cyanosis often followed by syncope. Older children often squat during a tet spell, which increases systemic vascular resistance and allows for temporary reversal of the shunt.Answer D: Transposition of the great vessels most commonly presents with cyanosis at birth.Bottom Line: The murmur of a ventricular septal defect is a harsh pansystolic murmur at the left lower sternal border. An atrial septal defect, ventricular septal defect, or patent ductus arteriosus can progress to Eisenmenger syndrome, which is reversal of a left-to-right shunt to a right-to-left shunt, causing cyanosis.COMBANK Insight: It is important to review the most common characteristics of different types of heart murmurs. COMLEX exams are now incorporating more audio files; therefore, it would be wise to be able to diagnose each murmur based on how it sounds.

management of TCA overdose?

Management: Supportive care & therapy Supplemental o2, intubation IV fluids Activated charcoal for patients within 2 hours of ingestion (unless ileus present) Intravenous sodium bicarbonate for QRS widening or ventricular arrhythmia

What does beta blocker do in pt with DM symptoms?

Masks hypoglycemic symptoms but should still be given in setting of recent MI

When do u order contrast ?

Mass, infection, inflammation !

Mitochondrial encephalomyopathy with lactic acidosis and stroke-like episodes (MELAS) inheritance pattern?

Maternal transmission with variable expression (severity)

What labs should be ordered for a 45yo hypothyroid female with mild bump is ast/alt a normal-is alk phos and biliruben, with normal hep and infectious labs but has hepatomegally? Her symptoms are arthitis, erythema nodosum, thyroiditis, pleurisy, pericarditis, anemia, and sicca syndrome

Measure an ana and antismooth muscle AB as she likely has autoimmune hep causing progressive parenchymal liver damage as pt likely has autoimmune hepatitis. the autoab are in homogeneous staning pattern = lupoid hepatitis and anti smooth mm ab against actin

ogilves sydrome is diagnosis of exclusion after what has been ruled out.

Mechanical obstruction must be ruled out. It occurs when everything signs symptoms and xray look like mech obstruction in Large bowel without it = ogilves s/o. Treat with d/c narco, fluids, ng rectal suction

HTN emergency bp control acheived with what meds?

Medication for HTN Emergency: 1. IV nitroprusside, 2. nicardipine, 3. clevidipine, 4. labetalol, and 5. fenoldopam.

What condition has UL hearing loss, periodic vertigo, and tinnitus?

Meniers triad of symptoms.

What is the Mentzer index used for?

Mentzer index (MCV/RBC) >13 is also suggestive of iron deficiency and can help differentiate it from B thalassemia

What are the 3 most effective ways to assess the adequacy of fluid resuscitation and the expected values?

Methods of assessing end organ perfusion: 1. Central venous pressure 2-6 mmHg 2. Cardiac output 4-8 3. Urine output .5- 1.0 mL/kg/hour for adults - 1 mL/kg/hour for children

___ are safe antihypertensives for use in pregnancy.

Methyldopa and labetalol hydralazine angiotensin-converting enzyme (ACE) inhibitors, angiotensin receptor blockers (ARBs), and beta-blockers can be teratogenic

_______ is occasionally used in patients with an inadequate response to loop diuretics.

Metolazone is a thiazide diuretic which blocks sodium reabsorption in the distal tubules and Metolazone is NOT indicated as this patient is euvolemic and has minimal peripheral edema.

What drug is contraindicated in patients who use cocaine because of the mechanism of action? veronica

Metoprolol is a selective beta-blocker, cocaine may allow unopposed alpha activation and exacerbate high blood pressure, causing hypertensive emergency and worsening end organ damage. Use non-selective BB like labetalol with alpha blocking properties

____ is an antibiotic prescribed for treatment of rosacea.

Metronidazole

Carcinoid tumors usually after spreading to what organ(s) p/w elevated 5-HIAA and elevated chromogranin A?

Mets to liver, carcinoids can metastasize to the lung, bone, skin, and other areas. Associated symptoms occur after carcinoid tumors metastasize.

STATS: if a test has more true positives and fewer false negative what is it?

More sensitive and can correctly dx more pt who have dx (eg HIGH TN, AND LOW FN)

MGMT of aortic dissection? think MEN

Morphine IV esmolol If >120sbp consider sodium nitroprusside Emergent surgical repair for ASCENDING dissection (Morphine, Esmolol, Nitro)

paget has wat electolyte abnormalities

Mosaic pattern: Alkaline Phosphate high Serum Calcium normal Serum Phosphate norm Parathyroid Hormone norm

What BP medication should be given in a pt with gout attacks? Which should be avoided?

Most diuretics (HCTZ and furosemide) decrease urate excretion and should be avoided! Losartan has a mild uricosuric effect and is 1st line for HTN in those with gout attack. Ace can also be used in those with htn and CKD regardless of crt level Avoid low dose aspirin, loop diuretics and thiazides in those with gout!

Pregnant women often develop gallstones due to pregnancy-related hormonal changes when is cholecystectomy needed

Most patients are asymptomatic and require no treatment. Those who develop symptoms of recurrent biliary colic or acute cholecystitis typically require cholecystectomy.

Why do hypothyroid patients need to up titrate their levothyroxine dose when taking estrogen or if on OCP?

Most thyroid hormone in blood is bound to plasma PROTEINS (mainly TBG) Up titration needed to offset the increase TBG thyroxine binding globulin produced by liver in hyperEstrogen states! more TBG means more thyroid hormone will be needed to saturate the binding sites!

____ is an autosomal dominant disorder caused by germline mutations involving the ______ proto-oncogene located on chromosome 10.

Multiple endocrine neoplasia type 2 (MEN 2) MEN 2A, MEN 2B, and familial medullary thyroid cancer (FMTC). RET PCGENE FMTC is a subset of MEN 2A associated with MTC only, without PCC or parathyroid disease.

What is the management in someone with high grade squamous intraepithelial lesion (HSIL) on Pap smear? What is the mgmt. if pt is >≥25 and finished having kids?

Must evaluate for either CIN 2/3 with Colposcopy ASAP! p Can consider a loop procedure (excision of transformation zone in endocervix) if ≥25 and done childbearing

What is the difference btw allergic rhinitis and non allergic rhinitis

NAR have less watery eye symptoms and PNDrip is without identifiable trigger are treated with intranasal glucocorticoids and intranasal antihistamines azelastine H1 antihistamines are sedating and not preferred first line treatment

OMM C SPINE NERVES EXIT

NERVE ROOTS OF C SPINE EXIT ABOVE THE VERTEBRAE C8 EXITS C7 - T1

CD4 count <300 in HIV and +FTABS + TREPONEMAL test rapid plasma reagin needs to have what specifically r/o? What is the treatment and what other stages are treated similarly? What are all other stages treated with and for how long if early secondary or <12 months, vs greater or unknown amount of time / gumma lesion?

NEUROSYPHILIS WITH LP IF ANY NEURO SX (headache, blurry vision etc. ) treatment is different for neurosyph. Treated with aqueous penicillin G similar to CONGENITAL syphilis Otherwise all other stages treated with IM bez penicillin x 1 dose ***if unknown, >12m or gumma x 3 dose ***$

if stroke pt hasnt had swallow eval how may meds be given

NG tube, iv

When is BBlocker or nitrate contraindicated in management of unstable angina and non stemi ?

NITRATES caution w. hypotension (RV infarct) BB contraindicated in heart failure and bradycardia

____ = 1/ARR

NNT is the number of patients who need to be treated in order to prevent one additional bad outcome; it is actually the inverse of absolute risk reduction.

Causes of Acute hypocalcemia (6) and diagnosis?

Neck surgery (parathyroidectomy) Pancreatitis Sepsis Tumor lysis syndrome Acute alkalosis Chelation: blood (citrate) transfusion, EDTA, foscarnet An ionized calcium level is required for diagnosis, as serum calcium = normal.

What commonly causes nec fasc IN HEALTHY and how does it present

Necrotizing fasciitis is often a polymicrobial infection in patients with poor peripheral circulation (DM).- type 1 less common Group A Streptococcus pyogenes is the most common cause of necrotizing fasciitis in healthy patients. (type 2 is most common) 1. pain out of proportion to other findings, 2. swelling, and 3. erythema. 4. Fever and 5. hypotension

What is the name of a rash that presents with glucagonoma and is super random?

Necrotizing migratory erythema which is in rare glucagonoma that is in mild diabetes with rash.

Which type of demential results in disrupted executive function and progression to dementia at 50-60Yo? Think robin williams was picked for it --?

Neurocognitive and degenerative disorder that is early onset with personality change and executive dysfunction and then later dementia = pick disease/ FTND frontotemporal neurocog. dx.

Why must one have initial brain MRI and baseline ECG followed by serial ecg for someone with tuberous sclerosis complex syndrome?

Neurologic impairment is the leading cause of death in patients with tuberous sclerosis complex. Uncontrollable seizures, aspiration pneumonia, and obstructive hydrocephalus can result from progressive central nervous system involvement.

What would prompt a diagnosis of SBP? What is used for treatment and why? What is the best indicator of mortality after 90days? hint: SICB got SBP

Neutrophils in peritoneal fluid >250 Give Prophylactic antibiotics and IV albumin to help prevent with Hepato-renal syndrome Meld score: INR, creatinine, bilirubin, sodium § mortality

What findings are considered diagnostic for ST-elevation myocardial infarction (STEMI)?

New ST elevation at the J point in >2 anatomically contiguous leads threshold with :>1 mm (0.1 mV) in all leads (except V2 and V3) in leads V2 and V3 ≥1.5 mm in women, ≥2 mm in men age ≥40, ≥2.5 mm in men age <40 New left bundle branch block with clinical presentation consistent with acute coronary syndrome (ACS)

What does a positive Nikolsky sign indicate?

Nikolsky sign = + when pressure on the base of a blister = sloughing of the skin seen in conditions such as: pemphigus vulgaris toxic epidermal necrolysis. staph scalded syndrome SJS/ TEN

Emperic treatment with what agents is used for pregnant and cystitis symptoms on exam?

Nitrofurantoin (fun fact causes Interstitial lung dx ILD) cephalexin amoxicillin clav #-7 days TMPSMX= BACTRIM ( only used in 2nd trimester rarely)

UTI antibiotics in pregnancy Recommended

Nitrofurantoin at 2nd trimester (avoid in 1st and right before birth as causes orofacial deformity and hemolytic anemia respectively) Amoxicillin Amoxicillin-clavulanate Cephalexin Fosfomycin

If someone has HSIL should an endometrial bx be done?

No a colopo must be done! Endometrial bx is done for evaluation of endometrial cancer

Should asa be discontinued prior to heart cath?

No continue in cath But DC 1 week prior in other surgeries with high bleeding risk.

What is LR Likelihood ratio is defined as what? How is it calculated?

Positive LR = sensitivity/ (1 - specificity) It is independent of disease prevalence. LR does not change with prevelance of disease.

If the central tendancy is mean, median and mode what is a positive? negatively skewed frequency order of MMM?

Positive begins with MODE> MED>MEAN (if ur mean ur last in positive skew) Mean < median< mode!

What does a positive ppd test tell you? What should be done after and is that enough for treatment?

Positive ppd doesnt tell you much you need to have coorelation with CXR to see if pt has ACTIVE dx Those with symptoms and cavitary infiltrate on cxr are thought to have ACTIVE DX and need microbial testing

Is pre-septal or post-septal cellulitis more dangerous? What imaging is needed?

Post septal or Orbital cellulitis is dangerous and associated with prooptosis and opthalmoplegia --> commonly due to SINUSITUS Ct of nasal and sinuses While pre-septal cellulitis is not as dangerous and caused by skin abrasion (s. aureas)

STATS: How can you calculate post test probability for a target disease using LR?

Post test odds = pretest odds x LR

PEP after rape if victim was vaccinated against hep B?

Postexposure prophylaxis for sexual assault treated empirically w/ Ceftriaxone, azithromycin, tenofovir-emtricitabine with raltegravir (carte) 1. chlamydia, 2. gonorrhea, and 3. trichomoniasis. 4. HIV prophylaxis is offered up to 3 days. 3 Multidrug regimen (eg, tenofovir-emtricitabine with raltegravir)

How does endometrial cancer present typically?

Postmenopausal bleed (angela 90day)

What is the most common cause of postpartum fever that p/w uterine fundal tenderness after 1 day c-section and what is the treatment?

Postpartum endometritis due to polymicrobial infection, treat with BSA with clinda and gentamicin until pt is afebril x24hrs. " cleanup for the gentelman"

H pylori testing is needed in someone <55yo with dyspepsia and what other symptoms?

Postprandial fullness, early satiation, epigastric burning that does not radiate!

Post partum with heavy bleed, edema, hyper-reflexive, pulmonary SOB o2 sat 92, HTN has what?

Pre-eclampsia: clonus (end organ damage) treated w MG to help reflexes during prenancy

What is the tx for G1P1 @ >20 WEEKS gestation: VS: SBP >140 mmHg (OR DBP>90 mmHg) - on 2 occasions at least 4 hours Labs: Proteinuria: >300 mg in 24 hours, P:C >0.3 mg/dL or 2+ on dipstick reading

Preeclampsia WITHOUT severe ft. - Increased maternal and fetal surveillance - Delivery at 37 0/7 weeks

What does Pemberton's test evaluate in someone with a goiter and NEW ONSET

Presence of facial plethora or JVD suggestive of thyroid as cause of obstructive symptoms --> thyroid gland infiltration of chronic lymphocytic infiltration (thyroid lymphoma)

What are the conditions and clinical clues that would prompt evaluation for secondary cause of hypertension?

Renal parenchymal disease: ↑ serum creatinine. Abnormal urinalysis (proteinuria, red blood cell casts) Renovascular disease: Severe hypertension with onset after age 55. Recurrent flash pulmonary edema. Rise in serum creatinine. Abdominal bruit Primary aldosteronism: Spontaneous or easily provoked hypokalemia Pheochromocytoma: Paroxysmal hypertension with tachycardia. Pounding headaches, palpitations, diaphoresis Cushing syndrome Cushingoid body habitus w/ Proximal muscle weakness and Hyperglycemia Hypothyroidism: Fatigue, dry skin, cold intolerance, Constipation, weight gain, bradycardia Primary hyperparathyroidism: Hypercalcemia, Kidney stones, Neuropsychiatric symptoms Coarctation of the aorta: Lateralizing hypertension, Brachial-femoral pulse delay

What evaluation should be done for pt with resistant hypertension and signs of diffuse atherosclerosis?

Renovascular hypertension get eval. with Renal Doppler ultrasonography first. Due to risk of contrast-induced nephropathy and nephrogenic systemic fibrosis when performing CT and gadolinium MR angiography, respectively, these should be avoided in patients with renal insufficiency.

Clinical clues to renovascular disease HTN-related symptoms?

Resistant HTN (uncontrolled despite 3-drug regimen) Malignant HTN (with end-organ damage) Onset of severe HTN (>180/120 mm Hg) after age 55 Severe HTN with diffuse atherosclerosis Recurrent flash pulmonary edema with severe HTN

What labs are needed to determining cause of normocytic/normochromic anemia

Reticulocyte count

If someone has peripheral vision loss then central and photopsia (flashing lights) and floaters everywhere what do they have and why?

Retinal detachment due to trauma, eye surgery, aging and myopia

patient with diabetic ketoacidosis developed fever, periorbital swelling, and a necrotic nasal turbinate, raising suspicion for

Rhinocerebral mucormycosis is most commonly seen in patients with diabetic ketoacidosis. Treatment requires surgical debridement and intravenous liposomal amphotericin B.

R prime wave in V1 and widened S wave in V6

Right bundle branch block can be benign or may occur in the setting of pulmonary hypertension or acute pulmonary embolism PE T wave inversion in leads II, III, and aVF is suggestive of the right heart strain that can ALSO occur with acute PE; BOTH ARE NON SPECIFIC

What are risk factors for Peripartum cardiomyopathy?

Risk factors Maternal age >30 Multiple gestation Preeclampsia, eclampsiA

SAAG <1.1 tells you what

SAAG less than 1.1 means NO PORTAL HYPERTENSION exudative fluid likely 2/2 1. peritoneal carcinomatosis 2. peritoneal TB 3. nephrotic s/o 4. pancreatitis 5. serositis

How can one determine if pt has a transudative or exudative effusion on tap? What is the main formula that is not outdated?

SAAG which has replaced the p/s calculation EXPLAINS IF PORTAL HTN IS PRESENT OR NOT SAAG >1.1 transudation (trans has more words so bigger number ) and <1.1 exudative Calculate by doing serum alb - ascitic albumen

What is given to pt in asthma if intermitant and what is added subsequently?

SABA prn Low dose ICS Medium dose ICS LABA High dose ICS Oral Corticosteroids Omalizumab

When is IV albumin needed?

SBP or large volume paracentesis

What would the PO4, Ca and Vit. D be in CKD that is longstanding? What would these electrolyte derangements result in?

SECONDARY HPT (2 ELEVATED P) HIGH P04 retention Low SERUM FREE CA LOW 1,25 DIHYDROXYV VD levels --> This leads to a compensatory rise in PTH (secondary HPT)

What are hints that would make you suspect psychological eitiology for erectile dysfunction?

Sudden onset spontaneous errection during non sexual event situational (eg only with partner)

What is the treatment in someone with sulfonylurea poisoning?

Sulfonylurea causes insulin overproduction thus first dextrose should be given followed by octreotide. Octreotide is a somatostatin analog that decreases insulin secretion in pt with severe sulfa overdose.

What condition presents in child with fever, crankiness followed by improvement of agitation with purulent ear drainage? Is palpation of pinnae sensitive ?

Suppurative otitis Media occurs with infection with nasal bacteria (GA strep) leading to bulging of TM and eventual rupture which relieve the pressure sensation in ear. The pinnae will be NONSENSITIVE Note: on exam of SOM TM may be said to be obsured by purulent discharge - do not be fooled for the TM is ruptured! in contrast to bacterial otitis externa (psudomonas aerginosa) where pinna is painful to touch and absence of purulence or fever.

What fracture occurs on a FOOSH and has limited ROM, swelling, elbow pain? TREATMENT?

Supracondylar fractures. An occult non-displaced fracture may be presumed if the FAT PAD displacement on x-ray and is treated with IMMOBILIZAITON!

What (which vit) deficiency in a patient who presents with bleeding gums and a hot dog/soda or tea/toast diet?

Suspect vitamin C deficiency in a patient who presents with bleeding gums and a hot dog/soda or tea/toast diet.

What is the diagnostic evaluation of stable CAD? What are the symptoms and risk factors for stable CAD?

Symptoms - able to exercise - stress test if negative RO CAD If unable - pharm test done and if positive cad present needing medical maangement w/ CA w revascularization in high risk pt

What would the joint aspiration need to show to be considered a septic joint? What abx should be given empirically against which pathogens?

Synovial fluid wbc >50k with neutrophil predominance indicates septic joint Empiric abx should target Staph A and streptococci with vanco.

can present with constitutional symptoms (eg, fever, fatigue) and polyarticular arthritis. However, its onset is generally gradual, with extensive systemic involvement

Systemic lupus erythematosus and skin findings such as photosensitivity and malar rash or discoid lesions. vs ra which is insidious

Danger signs indicating a potentially life-threatening cause of headache thus warrenting an MRI include the following:

Systemic symptoms (eg, fever) or illness (eg, cancer) Neurological signs/symptoms Onset is new Other associated conditions (eg, head trauma) Previous headaches with change in frequency, severity, or presentation

If the correlation coefficient ( r ) is not near the marginal values of (-1,+1) but rather closer to 0 (eg .33) what can be said of this association btw the 2 variables?

The closer the R value is to 0 the weaker the association thus if the value is a positive number but small then it is said to have a LOW predictive value

HTN emergency intial bp reduction should be rapidly lowered to ____ but no more than ____ what value?

The initial aim of treatment is to rapidly lower the diastolic pressure to 100 to 105 mmHg within 2 to 6 hours. The blood pressure should not be dropped by more than 25% of the presenting value.

intrinsic clotting cascade includes which factors?

The intrinsic pathway involves factors XII, XI, IX, VIII, X, V, II, and I. (PTT)

pt with 1. hyponatremia 2. hyperkalemia 3. mild hyperchloremic metabolic acidosis. 4. hypoglycemia The next step in evaluation of this patient is measurement of ----?

The next step: morning plasma cortisol with concurrent ACTH; a. low cortisol with high ACTH is diagnostic for primary adrenal insufficiency. b. if equivocal result get --> ACTH stimulation test c. OR get ACTH stimulation test may be ordered at the same time as the morning cortisol --- if hypoglycemic more likely to have chronic insufficiency!

STATS: if the SMR ratio is 1.75 in a statistical analysis what does that mean?

The observed number of death is 75% higher than expected Which is not the same as the crude or UNADJUSTED mortality rate

In the setting of likely pretest probability high with V/q: low- or moderate-probability and contraindication to CTA what is the management?

The presumed of acute PE is likely and anticoagulation should be initiated.

How many stages of pressure ulcers are there? What separates the worst form? Can you stage a pressure ulcer that is full thickness ?

There are 4 stages: 1. nonblanchable intact skin 2. shallow ulcer ---- deep ulcers require debridement of necrotic tissue : 3: deep to SQ fat full-thickness without fascial involvement 4: fascia and adjacent structural damage Full-thickness skin loss where base is covered with eschar is UNSTAGEABLE

Turners has what IQ

They are at increased risk of leaning disabilities warranting neurophychological test before school entry but they have normal IQ Coarctation of aorta bicuspid horseshoe kidney streak ovaries normal IQ

What is the difference btw cinacalcet and FHH regulation on PTH?

They have the opposite effect on calcium-sensing receptor which affects the PTH trigger Familial hypocalciuric hypercalcemia is an autosomal dominant disorder due to inactivating mutations of the calcium-sensing receptor. - mild hypercalcemia without any clinical findings and do not require any treatment.

In pt with suspected pulm. TB and abnormal CXR should have what testing for confirmation?

They need sputum acid fast bacillus smear, MAC, NAAT for microbial confirmation. TB skin test or INF gamma release assay CANNOT distinguish between active and latent TB

What diuretic can cause photosensitivity?

Thiazides are sulfonamides that cause photosensitivity rash and thus sun exposure should be limited and sunscreen used.

Tracheomalacia is a relatively common anomaly of the upper respiratory tract characterized by a dynamic collapse of the trachea during expiration, resulting in airway obstruction. Intrathoracic lesions typically present with recurrent harsh, barking, or croup-like cough, whereas extrathoracic lesions cause wheezing with expiratory stridor common if laryngomalacia is present.

This condition should be suspected in children with chronic wheezing.Answer A: Anaphylaxis is an acute allergic reaction characterized by respiratory compromise.Answer B: Bronchiolitis, or inflammation of the bronchioles, is a clinical syndrome characterized by wheezing, tachypnea, and hyperinflation of the lungs and is usually caused by RSV in young children.Answer C: Croup (laryngotracheitis) is a respiratory illness characterized by inspiratory stridor, a barking cough, and hoarseness. It typically occurs in children six months to three years of age and is caused by the parainfluenza virus. Croup would not be present from the time of birth through three months, as with this patient.Answer D: Foreign body aspiration is a common occurrence in children ages 1-3 and is characterized by the triad of wheeze, cough, and diminished breath sounds. Symptoms are typically acute in nature.Bottom Line: Consider tracheomalacia in a child with chronic wheezing.

Low levels of what ion causes carpopedal spasm?

This hypocalcemia is 2/2 the hypocarbia. The carpopedal spasm is NOT caused by primary hypocalcemia

What is the etiology behind new onset hypertensive urgency in someone who also has Raynaud and GERD

This person likely has scleroderma suffering renal crisis syndrome resulting in her hypertension. Capropril is used to treat the angiotensin induced vasoconstriction.

What medication is contraindicated in ACS due to non-STEMI or unstable angina with presence of s3, pulmonary edema, JVD?

This pt has acute decompensated heart failure that is likely from ischemic L ventricular dysfunciton. Beta Blockers are contraindicated since they decrease contractility and heart rate and make pulmonary edema worse, HOLD METOPROLOL until stable then low dose treatment is ok to resume.

Pt on glucocorticoids s/p renal txp with cough, Dyspnea, insidious onset fever, tachypnea with diffuse rales thought lungs crt 0.6 ELEVATED LDH what could have prevented this pt illness.

This pt on chronic immuno supression (or pt with AIDS CD4 <400) need TMPSMX +/- GC; to prevent PCP pneumocytis jiroveci penumonia. Imaging: ground glass diagnosis: induced sputum or BAC treatment: IV tmp-smx transitioned to oral tx once signs of clinical recovery! Pt with poor cell mediated immunity are succeptible and need prophylaxis

When is surgery indicated for a rapidly expanding aneurysm if the size is <5.5 cm?

Those that are increasing by .56cm /year or more since they are more likely to dissect

Initial testing to diagnose sicca syndrome

Though there is no set guideline for dx sjogrens Best initial test: Schimmers test with Ro- and La- antibodies, RF and ANA - confirming a secretory deficiency (e.g. Schirmer test) -- autoantibody screen: 1. Ro- and La- antibodies, 2. RF and 3. ANA Gold standard: labial salivary gland biopsy

What gestational age is threatened abortion considered? What is the difference btw threatened abortion or cervical insufficiency?

Threatened abortion <20ega The cervix is open in insufficiency and there is painless dialation and can occur <24 weeks

What is the TIMI score used for and what are the components (7)?

Thrombolysis in MI risk score: used for management of unstable angina or NSTEMI 1. ≥65yo 2. ≥ 3 CAD risk factors 3. CAD with > 50% occlusion 4. ASA use in past week 5. ≥2 anginal episodes (over 2 weeks) 6. elevated cardiac biomarkers 7. ST deviation >0.5mm on EKG admission 0-2 Low risk: stress test 3-7 mod-high risk: early coronary angio in 24hr

What findings on ECG most strongly indicates the need for tenecteplase in 30 mins of admission?

Thrombolytic therapy should be given to STEMI in rural facility where there is no access to PCI within 2 hrs! New ST elevation at the J point in >2 anatomically contiguous leads : >1 mm (0.1 mV) >1.5 mm in women, >2 mm in men age >40, and >2.5 mm in men age <40 in leads V2 and V3 New left bundle branch block LBBB w/ clinical presentation consistent with acute coronary syndrome

do angiodysplasias always bleed?

no only small percent among those with ESRD, aortic stenosis and VWD

Are CCB needed in ischemic stroke?

no. nicardipine in hemorrhagic only to keep sbp <140

____ prevents leukotriene synthesis by inhibiting 5-lipoxygenase.

Zileuton

When is the treatment for a-flutter contraindicated

a - flutter treatment: cardioversion or cardioablation unless: Pt. has new atrial flutter (or fibrillation) of duration >48 hours needs >3 weeks of anticoagulation prior.

Most patients age 13-65 benefit from sexually transmitted infection (STI) screening with

a 4-generation HIV test, regardless of the presence or absence of risk factors. Additional STI screening depends on many factors, including gender, sexual orientation, age, high-risk behavior (eg, encounters with sex workers), and symptoms.

Approximately 95% of pulmonary emboli arise from

a DVT in the proximal lower extremity.

Neonatal respiratory distress syndrome is caused by ____.

a lack of surfactant produced by type II pneumocytes to decrease the surface tension between alveoli.

Genetic testing for conditions such as familial adenomatous polyposis (FAP) is usually recommended for patients with

a stronger family history (eg, multiple relatives with cancer at a young age or at multiple sites).

what are the 2 main categories of normcytic anemia and how are they distinguished?

a. hemolysis b. hypoproliferative Get retic count to determine cause. If elevated --> hemolytic anemias get further labs with hemoglobin electrophoresis and Coombs' test.

Cocaine toxicity pathophys? What is 1st line treatment ? hint: downers

a1 and b1 adrenergic stimulation due to blocking of presynaptic NE reuptake in CNS leads to MI ACS stroke and aortic dissection 2/2 cocaine induced htn 1st line tx: benzo lorazepam diazepam

What pediatric hip condition presents with abducted flexed externally rotated hip? Vs which is restricted in abduciton?

abducted in infant with septic arthritis restricted abduction on dev. dysplasia (do B.O test)

What skin condition is seen with insulin resistance and GI malignancy?

acanthosis nigricans

common cause of DILI?

acetamenophen, VPA!!!!, AMOX clav, treat acetamenophen with nac, and vpa with L carnatine

Myasthenia gravis is an autoimmune disease caused by autoantibodies that attack the______ in the neuromuscular junction.

acetylcholine receptors

Contraindications to stress testing are ___

active ischemia, prinzmetals with ischemia on ekg, cardiac failure, or hemodynamically compromising valvular insufficiency.

Nitrates, diuretics, and opioids can reduce right ventricular (RV) preload and cause hypotension in patients with _____infarction. Such patients with evidence of low cardiac output and hypotension should be treated initially with

acute RV myocardial a bolus of intravenous fluids to improve RV preload. (VS INFERIOR INFARCT PULM EDEMA AVOID FLUIDS AFTER TAKING BP WITH NITRO)

Alzheimers patients lack what neurotransmitter?

acytylcholine that is why they are never given diphenhydramine (anticholinergic) but instead treated with an inhibitor of the enzyme that breaks down ACH (cholinesterases).

tachy treatment

adenosine sine wave

SMR represents an ___ measure of the overall mortality

adjusted: - Mortality is often adjusted for age (and less commonly for gender, race, or other factors) - used in occupational epidemiology

RTA 4 is characterized by what metabolic syndrome?

adrenal insufficiency hyperkalemia due to aldosterone deficiency

What CONSTITUTES an advanced adematous polyp?

advanced adenomatous polyps (size >1 cm, villous features, or high-grade dysplasia).

A stable patient who fails to arrive at the appointed time should be

advised to reschedule the appointment.

What key PE finding is highly specific for dx of lumbar spinal stenosis? SIT DOWN IF U HAVE STENOSIS

disappearance of pain is highly specific for LSS which presents in geriatric population as low back pain better with sitting and flexing (pushing grocery); worsens with extension. SIT WITH STENOSIS

pain that worsens with lumbar flexion

disk herniation flex forwared = herniation of anulus pulp and impinges on nerves root

Extracapsular fractures are at higher risk for ______ and present with what type of bruising?

displacement and usually have visible ecchymosis

Pt with NSTEMI need what treatment?

dual anti-platelet therapy, anticoagulation, BB, statins unless contraindicated

etiology and management of diverticular bleed?

due to arterial EROSION from mucosal outpouching dx: csp or tagged rbc scan. Treat: 1. iv volume replacement, 2. endoscopic tx or angio embolization 3. resection if bleed persists

where is calcium absorbed and why do we care

duodenum thus in gastric bypass its super important for them to get calcium supplement! or if they undergo thyroid surgery and remove the parathyroid they are going to get major osteoporosis

Bipolar II disorder is characterized by major depressive episodes

duration >2 weeks and hypomanic episodes (requiring a duration >4 days).

What is the most common neurological sign in wilsons disease?

dysarthria. Wilsons causes CU accumulation in brain and ensuing cognitive and gait dysfunction.

How is electrolyte repletion accoplished in re-feed syndrome (eg IV vs PO)?

empiric thiamine repletion monitor and repleate electolytes (eg Give phosphate) PRIOR to refeed. Caloric intake increased gradually with electrolyte monitoring. Oral phosphate therapy is preferred. IV predisposes to renal injury/ arrythmia and hypocalcemia

Sitagliptin is a dipeptidyl peptidase enzyme inhibitor (DPP-4) that enhances ___. It is taken PO with minimal hypoglycemia risk. Sitagliptin has what side effects commonly?

endogenous incretin actions. It may lower A1c levels 0.5-0.8 points. 1. HA, 2. nasopharyngitis, 3. URI 4. pancreatitis and so it may not be used in patients with previous pancreatitis.

What is required to document healing of gastric ulcers, and to rule out gastric cancer?

endoscopy This usually is performed 6 to 8 weeks after the initial diagnosis of PUD. Documentation of H. pylori cure with a noninvasive test, 1. urea breath test or 2. fecal antigen test, is appropriate in patients with complicated ulcers.

Thumb print sign on radio?

epiglottitis

Botulism is treated with ______ and ______. Patients with botulism usually have what deficits?

equine serum antitoxin and antibiotics. 1. symmetric descending weakness 2. starting in the cranial nerves 3. no sensory deficits (except blurred vision).

Cauda equina syndrome requires what emergent management

mri with l spine and NSGY eval surgical decompression is to reduce or eliminate pressure on the impacted nerves within 24 - 48 hours for improvement of sensory and motor deficits. CES is best treated with decompression by a lumbar laminectomy (and/or lumbar microdiscectomy) removal of all or part of the lamina (posterior part of the vertebra) to provide more space for the compressed spinal cord and/or nerve roots.

In someone with acute chest pain what should be the goal AFTER acs is ruled out

must differentiate btw cardiac and non -cardiac CP. Given symptoms and whether he is classified as anginal or non-anginal you must also take into account risk factors (sex age FH and htn as risk for ACS) if clinically significant CAD suspected --> stress test is best initial diagnostic intervention

Dengue fever name the common manifestation and management ? What syndrome can present that can cause shock?

myalgia break-bone fever, eye pain, rash, TCP <100k, bleed--> Shock, + tourniquet test - leukopenia, transaminitis Dengue hemorrhagic fever can cause capillary permeability leading to inc hemoconcentration pleural effusion + ascitis --> CIRCULATORY FAILURE Treat with supportive care

Trastuzumab-related cardiotoxicity is due to loss of

myocardial contractility (myocardial hibernation) leading to a decrease in LVEF. its reversible Trastuzumab should be withheld for 4 weeks if LVEF decreases by >16% from baseline, or - by 10%-15% from baseline to below the lower limits of normal. It should be completely discontinued if patients develop symptomatic heart failure.

carbamazepine use are significant adverse reactions include what?

neutropenia and bone marrow suppression due to carbamazepine use are significant adverse reactions. Elderly patients are also at risk of SIADH. Because of some mild anticholinergic effects, there is a risk of glaucoma, urinary retention or constipation

What is one major side effect of clozapine rendering it a last line for non treatment responding schizophrenia

neutropenia/ agranulocytosis

What should you worry about with low plt count is it bleed?

no not bleed but the 3 thrombotic causes of thrombocytopenia - thrombotic microangiopathy: atypical hus or ttp - antiphospholipid ab s/o - hit

does acetazolamide treat NPH?

no nph is treated with VP shunt after miller fischer is done

what paraneoplastic syndrome causes a high epo level?

renal cell carcinoma dx with epo level in someone who have high h/h Serum erythropoietin is a good initial diagnostic test for the evaluation of polycythemia and helps to guide the remainder of the work-up.

What is the RCRI index and what does it predict?

revised cardiac risk index preddictin the rate of cardiac death, arrest or nonfatal MI during surgery.

sternal angle of louis attaches to what rib and is what spinous process?

rib 2 T4

Angular cheilosis and stomatitis - nonspecific but can be seen in ____ deficiency.

riboflavin (vitamin B2) 2 lips

OMM: OA SD DX? what does it mean to have the right translation?

right translation = force from left to right = l sidebending FRrSl motion in relation to c1 has flexion and all motion in neutral type approx 50 % of flex and ext occurs at OA joint

patient WHO developed cardiogenic shock in the setting of acute inferior ST-elevation myocardial infarction (STEMI) after administration of nitroglycerine and morphine. This is most likely due to

right ventricular MI (RVMI). Patients typically present with hypotension or shock, jugular vein distension, and clear lung fields. RVMI is seen in 30%-50% of patients with acute inferior MI.

4 to 5 months old infant:

rolls front to back and back to front, grasps rattle, orients to voice, enjoys looking around, laughs

In cases of fetomaternal hemorrhage, the amount of fetomaternal transfusion is calculated FOR dose of anti-D immune globulin can be administered within 72-hourS. The screening that determines ____.

rosette test is a qualitative SCREEN

18mo old baby can do what?

run kick ball build 4 cube tower name stuff copy parents

Sodium bicarbonate can alkalinize the urine, which is its main action for treating

salicylate toxicity. Alkalinizing the urine enhances salicylate excretion by the kidney.

STATS: What is in common with sensitivity and postive ppv? what is a difference?

same = numerator different = sens not based upon prevalence

if the affect of a study is decreased in magnitude what must be changed to detect a smaller difference?

sample size must increase meaning n must increase

What type of lung disease manifests as a mixed restrictive and obstructive pattern eg low DLOC and low fev1 and fev1/fvc = 80%

sarcoidosis 1. Bilateral hilar lymphadenopathy +/- interstitial infiltrates 2. mixed restrictive and obstructive (reduced FEV1 and FEV1/FVC) on PFTs 3. Hypercalcemia and elevated ESR - DX: BX (eg, bronchoscopy with transbronchiAL: no treatment; those with symptoms or pulmonary function impairment receive 12-24 months of oral glucocorticoids

Major drug interactions of levothyroxine 2/2↑ TBG concentration (HINT: females love to be girls) vs. ↓ TBG concentration

↑ TBG concentration (f love to be girls) 1. Estrogen (oral), 2. tamoxifen, 3. raloxifene 4. Heroin, methadone ↓ TBG concentration (boys) Androgens, glucocorticoids Anabolic steroids Slow-release nicotinic acid

What lab abnormality is found in biliary atresia?

↑ conjugated hyperbilirubinemia (direct bilirubin >20% of total bilirubin) from progressive obstruction of the extrahepatic biliary tree - commonly occurs at age 2 years Vs. jaundice from breastfeeding in infants (2 weeks) p/w unconjugated hyperbilirubinemia

WHAT IS THE Thrombotic thrombocytopenic purpura Pathophysiology?

↓ ADAMTS13 activity → disseminated microvascular thrombosis Acquired (autoantibody) or hereditary

Major drug interactions THAT INC AND DEC levothyroxine - narrow therapeutic index, and bioavailability is sensitive to a number of dietary factors. IRON AND CA

↓ Levothyroxine absorption Bile acid-binding agents (eg, cholestyramine) Iron, calcium, aluminum hydroxide Proton pump inhibitors, sucralfate ↑ Thyroid hormone metabolism Rifampin Phenytoin Carbamazepine TBG = thyroxine-binding globulin.

thrombocytopenia threshold for platelet transfusion?

≤ 50k

When are nifphedapine (tocolytic) indicated for pre-term delivery? When is tocolytic and magnesium sulfate needed? When are antenatal CST needed (bethamethasone) needed?

≤34 weeks (after 34weeks toco is contraindicated) ≤32 week <37 weeks need bethamethasone to prevent neonatal resp. distress (TTN, bronchopulmonary dysplasia)

HIT requires 1. cessation of heparin, 2. laboratory testing to confirm the diagnosis, and 3. starting alternate anticoagulants (eg, direct thrombin inhibitors) even if thrombosis has not occurred. Warfarin is usually started after treatment with a non-heparin anticoagulant and platelet count recovery to____.

≥150,000/mm3.

centor criteria and what is the threshold for strep vs viral pharyngitis

≥3 bacterial less than = viral get rapid antigen test to confirm PRIOR to abx tonsillar exudates tender anterior cervical LAD fever absence of cough

What is considered severe lead toxicity? What is the treatment for symptomatic vs asymptomatic toxicity?

≥70 pb levels and acute encephalopathy = emergency --> hospitalization for chelation. 1. IM dimercaprol (crosses BBB) 2. IV calcium disodium edetate (EDTA) (decrease severity of encephalopathy). vs. Treat w/ Succimer if asymptomatic, moderate lead toxicity.


Related study sets

Study guide for science chapter 5 test

View Set

Chapter 04: Health of the Individual, Family, and Community

View Set

The Real World Chapter 8- Race & Ethnic Relations

View Set